eureka 2005

131
CONTEÚDO AOS LEITORES 2 XV OLIMPÍADA DE MATEMÁTICA DO CONE SUL 3 Enunciados e Soluções XLV OLIMPÍADA INTERNACIONAL DE MATEMÁTICA 13 Enunciados e Soluções ARTIGOS CENTRO DE MASSA E APLICAÇÕES À GEOMETRIA 29 Emanuel Carneiro & Frederico Girão SEQÜÊNCIA DE FIBONACCI 38 Cícero Thiago B. Magalhães COMO É QUE FAZ? 43 SOLUÇÕES DE PROBLEMAS PROPOSTOS 46 PROBLEMAS PROPOSTOS 60 COORDENADORES REGIONAIS 62

Upload: demi-de

Post on 22-Mar-2016

268 views

Category:

Documents


10 download

DESCRIPTION

SEQÜÊNCIA DE FIBONACCI 38 Cícero Thiago B. Magalhães COORDENADORES REGIONAIS 62 XV OLIMPÍADA DE MATEMÁTICA DO CONE SUL 3 Enunciados e Soluções XLV OLIMPÍADA INTERNACIONAL DE MATEMÁTICA 13 Enunciados e Soluções SOLUÇÕES DE PROBLEMAS PROPOSTOS 46 CENTRO DE MASSA E APLICAÇÕES À GEOMETRIA 29 Emanuel Carneiro & Frederico Girão CONTEÚDO ARTIGOS

TRANSCRIPT

Page 1: Eureka 2005

CONTEÚDO

AOS LEITORES 2

XV OLIMPÍADA DE MATEMÁTICA DO CONE SUL 3Enunciados e Soluções

XLV OLIMPÍADA INTERNACIONAL DE MATEMÁTICA 13Enunciados e Soluções

ARTIGOS

CENTRO DE MASSA E APLICAÇÕES À GEOMETRIA 29Emanuel Carneiro & Frederico Girão

SEQÜÊNCIA DE FIBONACCI 38Cícero Thiago B. Magalhães

COMO É QUE FAZ? 43

SOLUÇÕES DE PROBLEMAS PROPOSTOS 46

PROBLEMAS PROPOSTOS 60

COORDENADORES REGIONAIS 62

Page 2: Eureka 2005

Sociedade Brasileira de Matemática

EUREKA! N°21, 2005

2

AOS LEITORES

Chegamos a este número 21 com dois artigos: um de geometria e outrosobre a seqüência de Fibonacci. São as primeiras publicações dos respectivosautores na Eureka!. Esperamos que venham outras e que cada vez mais autorescontribuam com a revista. Apresentamos as soluções da Olimpíada do Cone Sul e daOlimpíada Internacional de 2004, competições nas quais as equipes brasileirastiveram muito bom desempenho. Agradecemos mais uma vez as contribuições dosleitores para a seção "Como é que faz?" e para a seção dos problemas propostos,com soluções e novos problemas, que fazem da Eureka! cada vez mais uma obra decriação coletiva.

Aproveitamos para registrar que foi criada em 2004 a Associação OlimpíadaBrasileira de Matemática – AOBM, uma pessoa jurídica destinada a ajudar asOlimpíadas de Matemática no Brasil a crescerem e se consolidarem.

Esperamos que a AOBM sirva como instrumento para maior integração eorganização da comunidade olímpica. Estimulamos nossos leitores a se associarem àAOBM, cujos sócios recebem gratuitamente a revista Eureka!. As informaçõessobre como se associar à AOBM podem ser encontradas na nossa página na internet:www.obm.org.br

Os editores

Page 3: Eureka 2005

Sociedade Brasileira de Matemática

EUREKA! N°21, 2005

3

XV OLIMPÍADA DE MATEMÁTICA DO CONE SULEnunciados e Soluções

PROBLEMA 1Maxi escolheu 3 dígitos e, fazendo todas as permutações possíveis, obteve 6números distintos, cada um com 3 dígitos. Se exatamente um dos números que Maxiobteve é um quadrado perfeito e exatamente três são primos, encontrar os 3 dígitosque Maxi escolheu.Dê todas as possibilidades para os 3 dígitos.

SOLUÇÃO DE LEANDRO FARIAS MAIA (FORTALEZA – CE)Sejam 1 2 3, ,x x x os dígitos escolhidos por Maxi. Note que 1 2 1 3 2 3, , x x x x x x≠ ≠ ≠pois as 6 reordenações: 1 2 3 3 2 1,...,x x x x x x são distintas.Agora faremos a lista de todos os números de 3 dígitos quadrados perfeitos:

2

2

2

2

2

2

2

2

2

2

10 100

11 121

12 144

13 169

14 196

15 225

16 256

17 289

18 324

19 361

==

==

==

==

==

2

2

2

2

2

2

2

2

2

2

20 400

21 441

22 484

23 529

24 576

25 625

26 676

27 729

28 784

29 841

==

==

==

==

==

2

2

30 900

31 961

=

=

Perceba que:• Os números que têm algum zero não satisfazem o enunciado: 2 2 210 ,20 ,30 .

• Reordenando 1, 6, 9 podemos obter 2 2 213 ,14 ,31 . Assim, 2 2 213 ,14 ,31não satisfazem o enunciado.• O número deverá apresentar no mínimo 2 dígitos ímpares, pois se tiver no máximoum, teremos (se tivermos) no máximo 2 números primos. Assim;

2 2 2 2 2 216 ,17 ,18 ,25 ,28 , 29 não satisfazem o enunciado.

• Os dígitos são distintos. Assim, 2 2 2 2 2 211 ,12 ,15 ,21 ,22 ,26não satisfazem o enunciado.

Page 4: Eureka 2005

Sociedade Brasileira de Matemática

EUREKA! N°21, 2005

4

Nos restam os números: 2 2 2 219 ,23 ,24 ,27 . Reordenando:219 361;136,163,316,361,613,631.=223 529;259, 295,529,592,925,952.=224 576;567,576,657,675,756,765.=227 729;279, 297,729,792,927,972.=

Perceba que os dígitos:• 1, 3, 6 satisfazem o enunciado, pois, 163, 613, 631 (apenas) são primos e 361(apenas) é quadrado perfeito.• 2, 5, 9 não satisfazem o enunciado, pois, 295, 592, 925, 952 são compostos e 529(apenas) é quadrado perfeito, assim teremos no máximo um primo.• 5, 6, 7 não satisfazem o enunciado pelo mesmo raciocínio acima: 675, 765, 756 e576 são compostos.• 2, 7, 9 não satisfazem o enunciado pois todas as suas reordenações são múltiplosde 9.Portanto, os dígitos que Maxi escolheu foram 1, 3, 6.

PROBLEMA 2Dada uma circunferência C e um ponto P exterior a ela, traçam-se por P as duastangentes à circunferência, sendo A e B os pontos de tangência.Toma-se um ponto Q sobre o menor arco AB de C. Seja M a interseção da reta AQcom a perpendicular a AQ traçada por P, e seja N a interseção da reta BQ com aperpendicular a BQ traçada por P.Demonstre que, ao variar Q no arco AB, todas as retas MN passam por um mesmoponto.

SOLUÇÃO DE LEANDRO FARIAS MAIA (FORTALEZA – CE)

Sejam ,QBAα = � QABβ = � e "D" um ponto sobre AB de modo que .PD AB⊥

Como AP é tangente a "C", então: .QBA QAP= = α� � Analogamente:.PBQ = β�

Agora veja que:i) PNDB é inscritível, pois

90 .PNB PDB= ° =� � Assim;NPD NBD= = α� � e .NDP NBP= = β� �

Page 5: Eureka 2005

Sociedade Brasileira de Matemática

EUREKA! N°21, 2005

5

D α B

β β

α

A

β α

N Q

X

M

P

β α

ii) PMDA é inscritível, pois 90 .PMA PDA= ° =� �

Assim; PDM PAM= = α� � e .MPD MAD= = β� � Portanto, de i) e ii) temosque• // .NPD MDP NP MD= = α ⇒� �

• // .NDP MPD PM ND= = β⇒� �

Logo, PMDN é paralelogramo, e então NM e PD se cruzam no ponto médio (*),

assim para qualquer "Q" e �AB (menor), MN passará por um ponto fixo que é o

ponto médio da altura .PDProva de (*):

β θ

M

θ α B A

D C α

β

// , .AB DC ACD CAB ABD BDC⇒ = =� � � �

// .BC AD ADB CBD⇒ =� �

Page 6: Eureka 2005

Sociedade Brasileira de Matemática

EUREKA! N°21, 2005

6

Assim, ( )OACD ACB LAA DC AB∆ ≡ ∆ ⇒ = e .AD BC=

( )MDC ABM ALA AM MC∆ ≡ ∆ ⇒ = e .BM MD=

PROBLEMA 3

Seja n um inteiro positivo. Chamamos Cn a quantidade de inteiros positivos x,menores que 10n, tais que a soma dos dígitos de 2x é menor que a soma dos dígitosde x.

Demonstre que Cn � ( )410 1

9n − .

SOLUÇÃO:

Se 0

10k

jj

j

m a=

= ⋅∑ , com {0,1,2,...,9}, ,ja j k∈ ∀ ≤ temos 0

2 (2 ) 10 .k

jj

j

m a=

= ⋅∑ Note

que, se {0,1, 2,3,4},a∈ 2 {0,2, 4,6,8}a∈ tem apenas um dígito, e, se

{5,6,7,8,9},a∈ 2 {10,12,14,16,18}a∈ tem dois dígitos, sendo o primeiro deles

igual a 1. Assim, na soma 0

(2 ) 10k

jj

j

a=

⋅∑ , não há "vai um", pois, se (2 ) 10iia ⋅ e

(2 ) 10 jja ⋅ têm dígitos não nulos na k-ésima, posição, com i < j, então k = j = i + 1,

sendo o dígito de (2 ) 10iia ⋅ igual a 1, nessa k-ésima posição, donde sua soma é

menor que 10 (pois o dígito corresponde de (2 ) 10 jja ⋅ é no máximo 8). Portanto, se

s(m) denota a soma dos dígitos de m, 0

( )k

jj

s m a=

= ∑ e 0

(2 ) (2 ).k

jj

s m s a=

= ∑ Agora, para

{0,1, 2,3,4},a∈ (2 ) 2 ,s a a= e, para {5,6,7,8,9}a∈ , (2 ) 2 9.s a a= − Portanto, para

0 9,a≤ ≤ (2 ) (2 (9 )) 2 2(9 ) 9 9 (9 ).s a s a a a a a+ ⋅ − = + − − = = + − Assim, se1

0

10n

jj

j

x a−

=

= ⋅∑ , (com 0 9,ja j≤ ≤ ∀ ) é um inteiro positivo menor que 10 ,n temos

1

0

(10 1) (9 ) 10 ,n

n jj

j

x a−

=

− − = − ⋅∑ e logo, (2 ) (2(10 1 ))ns x s x+ − − =

1 1 1 1

0 0 0 0

(2 ) (2 (9 )) ( (2 ) (2 (9 )) ( (9 ))n n n n

j j j j j jj j j j

s a s a s a s a a a− − − −

= = = =

= + ⋅ − = + ⋅ − = + − =∑ ∑ ∑ ∑

Page 7: Eureka 2005

Sociedade Brasileira de Matemática

EUREKA! N°21, 2005

7

1 1

0 0

(9 ) ( ) ((10 1) ).n n

nj j

j j

a a s x s x− −

= =

= + − = + − −∑ ∑ Assim, (2 ) ( )s x s x< se e somente se

(2(10 1 )) (10 1 ),n ns x s x− − > − − e (2 ) ( )s x s x= se e somente se

(2(10 1 )) (10 1 ).n ns x s x− − = − − Portanto, 10 1

,2

n

n

AC

− −= onde

#{1 10 | (2 ) ( )}.nA x s x s x= ≤ < = Note agora que ( ) (mod9)s x x≡ e

(2 ) 2 (mod 9)s x x≡ , donde, se (2 ) ( ),s x s x= então 2 (mod 9)x x≡ , e logo

0(mod 9).x ≡ Assim, 10 1

#{1 10 | 0(mod 9)} ,9

nnA x x

−≤ ≤ < ≡ = donde

4(10 1).

9n

nC ≥ −

PROBLEMA 4Arnaldo escolhe um inteiro a, a � �� � ������ �� ��� �� ������� b, b � �� �����

dizem, em segredo, o número que escolheram a Cernaldo, e este escreve em umquadro os números 5, 8 e 15, sendo um desses a soma a + b.Cernaldo toca uma campainha e Arnaldo e Bernaldo, individualmente, escrevem empapéis distintos se sabem ou não qual dos números no quadro é a soma de a e b, eentregam seus papéis para Cernaldo.Se em ambos os papéis está escrito NÃO, Cernaldo toca novamente a campainha, eo procedimento se repete.Sabe-se que Arnaldo e Bernaldo são sinceros e inteligentes.Qual é o número máximo de vezes que a campainha pode ser tocada até que umdeles escreva que sabe o valor da soma?

SOLUÇÃO DE TELMO LUIS CORREA JÚNIOR (SANTO ANDRÉ – SP)Se 9,a ≥ a campainha toca apenas 1 vez: Arnaldo sabe que 0,b ≥ logo a única somapossível é 15. O mesmo ocorre se 9,b ≥ com Bernaldo.Caso contrário, a campainha toca 2 vezes: com , 8a b ≤ não é possível decidir entreas somas dadas na primeira vez.Se a = 6, Arnaldo agora sabe que 8,b ≤ então a soma não pode ser 5 ou 15, apenas8. O mesmo ocorre se b = 6. Se {0,1,2,3,4,5,7,8},a∈ ainda não é possível decidirentre 5, 8 e 15. O mesmo ocorre para Bernaldo.Caso a campainha toque pela terceira vez , {0,1,2,3,4,5,7,8}a b∈ .

Page 8: Eureka 2005

Sociedade Brasileira de Matemática

EUREKA! N°21, 2005

8

Se a = 2, Arnaldo sabe que 6b ≠ e 8,b ≤ logo a única soma possível é 5. O mesmoocorre se b = 2. Caso 2a ≠ e 2b ≠ , ninguém escreve o SIM. De fato:Se {0,1,3,4,5},a∈ Arnaldo não pode decidir entre as somas 5 e 8.Se {7,8},a∈ Arnaldo não pode decidir entre as somas 8 e 15. O mesmo ocorre paraBernaldo.A campainha toca pela quarta vez, e ambos sabem que , {0,1,3,4,5,7,8}.a b∈Se a = 3, Arnaldo sabe que 2,b ≠ logo a soma deve ser 8. Se {0,1,4,5}a∈ não épossível decidir entre 5 e 8, se {7,8}a∈ , não é possível decidir entre 8 e 15. Omesmo ocorre com Bernaldo.A campainha toca pela quinta vez: ambos sabem que , {0,1,4,5,7,8}a b∈ .Se a = 5, Arnaldo sabe que 3b ≠ , logo a soma deve ser 5. Se {0,1,4}a∈ não épossível decidir entre 5 e 8, se {7,8}a∈ não é possível decidir entre 8 e 15.O mesmo para Bernaldo.A campainha toca: sexta vez - ambos sabem que , {0,1,4,7,8}.a b∈Se a = 0, Arnaldo sabe que 5b ≠ , logo a soma deve ser 8. Se {1,4},a∈ não épossível decidir entre 5 e 8, se {7,8},a∈ entre 8 e 15.O mesmo para Bernaldo.Sétima vez: ambos sabem que , {1,4,7,8}.a b∈ Se a = 8, Arnaldo sabe que a somadeve ser 15, se {1,4},a∈ não pode decidir entre 5 e 8, se a = 7, não pode decidirentre 8 e 15. O mesmo com Bernaldo.Oitava vez: ambos sabem que , {1,4,7}.a b∈ Se a = 7, Arnaldo sabe que a somadeve ser 8, se {1,4},a∈ não pode decidir entre 5 e 8.O mesmo para Bernaldo.Nona vez: ambos sabem que , {1,4}.a b∈ Arnaldo sabe que única soma possível sea = 1 é 5;se a = 4, a soma pode ser 5 ou 8. O mesmo para Bernaldo.A campainha toca pela décima vez: agora ambos sabem que a = b = 4, e determinama soma com segurança. Logo, a campainha pode ser tocada no máximo dez vezes.

PROBLEMA 5Utilizando triangulinhos eqüiláteros de papel, de lado 1, forma-se um triângulo

eqüilátero de lado 20042 . Desse triângulo retira-se o triangulinho de lado 1 cujocentro coincide com o centro do triângulo maior.Determine se é possível cobrir totalmente a superfície restante, sem superposiçõesnem buracos, dispondo-se somente de fichas em forma de trapézio isósceles, cadauma formada por três triangulinhos eqüiláteros de lado 1.

Page 9: Eureka 2005

Sociedade Brasileira de Matemática

EUREKA! N°21, 2005

9

SOLUÇÃO DE GABRIEL TAVARES BUJOKAS (SÃO PAULO – SP)

Vamos mostrar por indução que qualquer 1n ≡ (mod 9) é possível cobrir o triângulodividido em n partes cada lado. Como 20042 ( 1)≡ + (mod 9) já que 62 1n ≡ e 6 | 2004,

62 1n ≡ (mod 9) e 6 | 2004 para 20042n = é possível.Vamos mostrar que se para 1n ≡ (mod 9) é possível, então para n + 9 também é.Temos que 9 1n + ≡ (mod 9). Cobriremos os 3 lados do triângulo maior assim:Comece por um vértice A e cubra um lado a que este pertença com peças atérestarem 2 espaços (como 1n ≡ (mod 3), isto é possível). Agora preencha da mesmaforma a segunda linha, até restarem 4 espaços.Na terceira linha, deixe um espaço à esquerda e preencha até onde foi a segundalinha (como indicado na figura acima). E, acima das primeiras duas peças da terceiralinha, ponha mais uma (como na figura).Faça isso para todos os vértices. Observe que não há nem espaços nemsuperposições. Assim criamos uma "barba" e o meio é um triângulo de lado(n + 9) – 9 (muito fácil de ver, já que a borda tem "largura" 3). O centro do triângulogrande coincide com o do de lado n. Assim, por hipótese de indução, este triângulocentral pode ser preenchido, logo o triângulo grande também!. A base da indução én =1, caso em que não há nada a fazer. Assim, nossa afirmação está provada.

Page 10: Eureka 2005

Sociedade Brasileira de Matemática

EUREKA! N°21, 2005

10

PROBLEMA 6Sejam m, n inteiros positivos. Em um tabuleiro m × n, quadriculado emquadradinhos de lado 1, considere todos os caminhos que vão do vértice superiordireito ao inferior esquerdo, percorrendo as linhas do quadriculado exclusivamentenas direções ← e ↓.Define-se a área de um caminho como sendo a quantidade de quadradinhos dotabuleiro que há abaixo desse caminho. Seja p um primo tal que rp(m) + rp(n) ≥ p,onde rp(m) representa o resto da divisão de m por p e rp(n) representa o resto dadivisão de n por p.Em quantos caminhos a área é um múltiplo de p?

SOLUÇÃO:Para resolver este problema, usaremos técnicas de funções geratrizes descritas noartigo "Séries formais", de Eduardo Tengan, da Eureka! No. 11, pp. 34 – 39.Representaremos o tabuleiro por {1, 2,…,m} × {1, 2,…,n}.Desta forma, os quadradinhos são representados por pares (i, j), com1 ,1 .i m j n≤ ≤ ≤ ≤ Considere agora um caminho como no enunciado. Para

1 ,i m≤ ≤ seja ib o número de quadradinhos da forma (i, y) que estão abaixo docaminho. Como o caminho só desce ou anda para a esquerda, temos

1 2 30 ... mb b b b≤ ≤ ≤ ≤ ≤ (e, reciprocamente, tais ib determinam um caminho). A

área do caminho é 1

.m

ii

b=∑ Definamos 1mb n+ = e 0 0.b = Para 1 1,i m≤ ≤ + seja

1.i i iy b b −= − Temos 1

1 01

m

i ni

y b b n+

+== − =∑ , e, em geral, para

1,k m≤ + 01

.k

i k ki

y b b b== − =∑ Portanto, a área do caminho é

1

11 1 1 0

( 1 ) ( 1 ) .m m m m

i i i m ji i i j

A b m i y m i y j y+

+ −= = = =

= = + − ⋅ = + − ⋅ = ⋅∑ ∑ ∑ ∑

Assim, fazendo 1 ,j m jz y + −= temos 0

m

jj

z n=

=∑ e 0

.m

jj

j z A=

⋅ =∑Assim, o problema é equivalente a achar o número de soluções de

0 1 ... ,mz z z n+ + + = com 0iz ≥ para 0 1i m≤ ≤ + e 0

m

jj

j z=

⋅∑ múltiplo de p. Para

isso, considere a função 0

1 1 1 1( , ) ... .

1 1 1 1

m

k mk

F x yx y y xy x y=

= = ⋅ ⋅ ⋅− − − −∏ Como

Page 11: Eureka 2005

Sociedade Brasileira de Matemática

EUREKA! N°21, 2005

11

2,

,

11 ( ) ..., ( , ) ,

1k k r s

r skr s

x y x y F x y a x yx y ∈

= + + + =− ∑

onde ,r sa é o número de

soluções de 0 1 ... mz z z s+ + + = com 0, 0iz i≥ ∀ ≥ e 2 30

2 3 ... .m

i jj

z z z jz r=

+ + + = =∑Em particular, ,r na é o número de caminhos como no enunciado com área r.

Queremos então calcular ,|

,r np r

a∑ i.e., a soma dos ,r na para os r múltiplos de p.

Para isso, consideremos a soma 1

0

( , ),p

k

k

F y−

=∑ ξ onde 2 i pe= πξ é uma raiz p-ésima da

unidade (e logo 1

0

0,p

kr

k

=

=∑ξ sempre que r não é múltiplo de p). Temos

1 1 1

, , ,0 0 , , 0 ,

|

( , )p p p

k kr s kr s sr s r s r s

k k r s r s k r sp r

F y a y a y p a y− − −

= = ∈ ∈ = ∈

= ⋅ ⋅ = ⋅ ⋅ = ⋅ ⋅

∑ ∑ ∑ ∑ ∑ ∑

� � �

ξ ξ ξ (pois, se

p | r, 1

0

pkr

k

p−

=

=∑ξ ) . Em particular, o coeficiente de ny em 1

0

( , )p

k

k

F y−

=∑ ξ é p vezes o

número de caminhos como no enunciado cuja área é um múltiplo de p.

Para ( ) 2 11

10, , (1, ) (1 ...) ,

(1 )k m

mk F y F y y y

y+

+= = = = + + +−

ξ e o coeficiente de ny

em (1, )F y é o número de soluções de 1 2 1... mk k k n++ + + = , 0, ik i≥ ∀ , que é

.

m n

m

+

Por outro lado, para ( ) 1 10 0

1 1 11 1, ,

1

m mk

kr m krr r

k p F yy y y+ −

= =

≤ ≤ − = =− −∏ ∏ξξ ξ

.

Escrevendo ( ) ( 1) ( ( )),p pm qp r m q p p r m= + = + − − com 0 ( ) ,pr m p≤ < temos

( )( )

( )( )

( )( )

( 1) 1 ( 1) 1 ( 1) 1

1 1 1( 1) 1 111

1

0

1 (1 ) 11 1

, .( 1) ( 1) 1

q p q p q pkr kr kr

k r m r m r mq p qp qq p p

kr

r

y y yF y

y q p yy yy

+ − + − + −

= + = + = ++ − +− ++

=

− − −= ⋅ = =

+ − −−

∏ ∏ ∏

ξ ξ ξξ

ξ

Como o produto ( )( 1) 1

1

1q p

kr

r m

y+ −

= +

−∏ ξ tem ( ) 1pp r m− − termos, ao desenvolver esse

produto, obtemos termos da forma ,sa y⋅ com 0 ( ) 1.ps p r m≤ ≤ − − Por outro lado,

todas as potências de y em ( ) 121

11 ...

(1 )

qp p

p qy y

y

+

+ = + + +−

têm expoente múltiplo

Page 12: Eureka 2005

Sociedade Brasileira de Matemática

EUREKA! N°21, 2005

12

de p, donde os termos não-nulos em ( , )kF yξ têm o expoente de y congruente a s

módulo p, para algum s com 0 ( ) 1.ps p r m≤ ≤ − − Entretanto, ( ) ( ) ,p pr m r n p+ ≥

donde ( ) ( ) 1p pr n p r m> − − , e, como n é congruente a ( )pr n módulo p, nenhum

termo não nulo em ( ),kF yξ tem expoente n. Assim, o número procurado de

caminhos cuja área é um múltiplo de p é 1

m n

mp

+

.

���� �����

���259649512 1− � ���� �� � ����� �� ��������� ���� � �

���� ���� �������� � � �� ���� �� ��������� ����� �� ������� ������ � ��� ������ ����� ��������� �� �!

240365832 1,− 209960112 1− �134669172 1.−

" ����#���� ��� � �$ ���� % &��� ���� �� ���������������'� (�� ��� �� )*�+� � �� , �-�-��� � �-��! ��

��� ����� � ./�01� �� ��2� ����� �$ ���� ������� ������� ��������� ������3$�- ��-� ����#�� � �� � ������ ������� �������� � ��-��� � ���������4�2�5 � �566***������������ ���� ������ ��,���78���

Page 13: Eureka 2005

Sociedade Brasileira de Matemática

EUREKA! N°21, 2005

13

XLV OLIMPÍADA INTERNACIONAL DE MATEMÁTICAEnunciados e Soluções

Caros leitores: nossa equipe da IMO-2004 tinha seis alunos muitodiferentes, mas com uma característica em comum: garra. Todos eles adotaram atática de escrever tudo o que passava pelas suas cabeças, de modo a maximizar assuas pontuações. Assim, não podemos transcrever as suas soluções ipsis literis. Aocontrário, vamos colocar somente partes das soluções deles, com alguns comentáriossobre as suas idéias.

Carlos Yuzo Shine, vice-líder do Brasil na IMO-2004PRIMEIRO DIA

PROBLEMA 1

Seja ABC um triângulo acutângulo com AB ≠ AC. A circunferência de diâmetro BCintersecta os lados AB e AC nos pontos M e N, respectivamente. Seja O o ponto

médio do lado BC. As bissetrizes dos ângulos �BAC e �MON intersectam-se em R.Prove que as circunferências circunscritas aos triângulos BMR e CNR têm um pontoem comum que pertence ao lado BC.

SOLUÇÃO DE THIAGO COSTA LEITE SANTOS (SÃO PAULO - SP)

β

B K O

π – 2β π – 2γ γ C

Γ

γ

ΓC N

A

ϕ R ϕ

β

M

ΓB

α/2 α/2

π / 2– α π / 2– α

Page 14: Eureka 2005

Sociedade Brasileira de Matemática

EUREKA! N°21, 2005

14

Seja Γ a circunferência com diâmetro BC, ΓB o circuncírculo de BMR e ΓC ocircuncírculo de CNR. Suponhamos sem perda de generalidade que o circunraio deABC mede 1/2, de modo que BC = sen α, AC = sen β e AB = sen γ.

Aqui, Thiago nota (e prova) que o ponto comum às duas circunferências e o lado BCsó pode ser o pé da bissetriz:

Temos pot AΓ = AM ⋅ AB = AN ⋅ AC. Mas B

pot AΓ = AM ⋅ AB e C

pot AΓ = AN ⋅ AC,

logo A pertence ao eixo radical de ΓB e ΓC. Como R também pertence ao eixoradical, a reta AR é o eixo radical de ΓB e ΓC. Logo o ponto comum a ΓB e ΓC e BCsó pode ser a interseção de AR e BC, ou seja, a interseção da bissetriz de ∠BAC eBC.

Mais algumas considerações geométricas: como OM = ON (raios de Γ), ∠NOR =∠MOR (OR é bissetriz de ∠MON) e OR é comum, os triângulos MOR e NOR sãocongruentes (caso LAL), portanto ∠RNO = ∠RMO.

Nesse momento, Thiago percebe como poderia terminar o problema: seja K ainterseção da bissetriz de ∠BAC e BC. A idéia dele é provar que K pertence a BC,ΓB e ΓC. Que pertence a BC é óbvio. Para provar que pertence a ΓB, basta provar queB, M, R e K são concíclicos; e, de modo análogo, para provar que pertence a ΓC,basta provar que C, N, R e K são concíclicos.

Aqui está a formalização dele: se provarmos que ∠RNO = α/2, teríamos ∠RNC = γ+ α/2 e ∠RNC + ∠RKC = γ + α/2 + β + α/2 = π, portanto K pertence a ΓC.Analogamente (aqui você deve verificar que Thiago usa o fato que ele provou antesde que ∠RNO = ∠RMO), vamos ter K pertencente a ΓB, portanto ΓB e ΓC têm umponto comum em BC.

Deste modo, só falta provar que ∠RNO = α/2. Isso é um problema que costuma serresolvido de duas maneiras: (1) com “arrastão” (isto é, calculando todos os ângulosna figura) ou (2) “arrastão” seguido de contas (isto é, com trigonometria, geometriaanalítica ou complexos). Thiago resolveu adotar a opção (2), com trigonometria(quando o coordenador viu as contas, ele comentou, “estava indo tão bem até aqui,por que ele teve que fazer essas contas??”)

Page 15: Eureka 2005

Sociedade Brasileira de Matemática

EUREKA! N°21, 2005

15

O triângulo NBO é isósceles, logo ∠ONC = ∠OCN = γ e ∠NOC = π – 2γ.

Analogamente, ∠MOB = π – 2β e θ = ∠MOR = ∠NOR = ( 2 ) ( 2 )

2

π − π − β − π − γ =

2

π – α.

Seja ∠RNO = ϕ. A idéia é calcular alguma função trigonométrica de ϕ (geralmenteacaba sendo a cotangente, por causa do truque da cotangente – veja o artigoGeometria com Contas, Eureka! 17) e compará-la com a mesma funçãotrigonométrica de α/2 para provar que ϕ = α/2.

Mas por onde começar? Primeiro devemos procurar por triângulos que envolvam ϕ.Um deles vem da própria definição de ϕ: RNO. Outro bem interessante é o ARN,cujos ângulos envolvem tanto ϕ como α/2, além de ter RN em comum (umsegmento a menos para calcular, como você vai ver depois!). Então parece ser umaboa estratégia utilizar esses dois triângulos.

Lei dos senos em RNO:

( ) ( )2 2

RN NO

sen senπ π=

−α −α + ϕ(1)

Lei dos senos em ARN:

( )2 2

RN AN

sen senα α=

γ + ϕ−(2)

Veja que se dividirmos (1) por (2), RN se cancela, então não precisamos calcular

RN. Só precisamos de NO e AN. NO é fácil: é o raio 2 2

BC senα= . Além disso,

AN = AC – NC = sen β – NC. Só falta NC!

Pela lei dos senos em NOC,2 2 cos

cos2 2 2

NC OC sen sen sen senNC sen

sen sen sen sen

α γ α ⋅ γ γ= ⇔ = = = α γγ γ γ γ

logo AN = sen β – sen α cos γ = sen(α + γ) – sen α cos γ = sen α cos γ + sen γ cos α– sen α cos γ = sen γ cos α.

Vamos usar repetidas vezes o fato de que sen(π/2 – x) = cos x e as fórmulas deProstaférese (que os gregos tiveram que deduzir na coordenação! procure-as na

Page 16: Eureka 2005

Sociedade Brasileira de Matemática

EUREKA! N°21, 2005

16

Eureka! 17). Agora sim, dividindo (1) por (2) e substituindo NO e AN, obtemos anossa conta final:

⇔( ) ( )2 2 22

2cos cos cos( ) cos( ) cos

sen sen sensen sen

sen

α α αα

α

γ + ϕ− γ + ϕ − γ= ⇔ =α γ α ⋅ α −ϕ α −ϕ

⇔ ( )2 2cos( ) cossen sen α αγ α −ϕ = γ + ϕ− (*)

Parece loucura, mas Thiago abriu os senos e co-senos. Mas isso tinha um motivo:ele queria achar cotg ϕ (lembre-se: o segredo do sucesso nas contas é sempre teruma meta em mente!).

(*) ⇔ ( ) ( )2 2 2 2cos cos cos cos cos cossen sen sen sen sen senα α α αϕ α γ + ϕ γ α = ϕ γ − + ϕ γ −

⇔ ( )( ) ( )( )2 2 2 2cos cos cos cos cossen sen sen sen senα α α αϕ α γ − γ − = ϕ γ − − γ α⇔ cos ( ( ) ( ) ( ))sen sen sen senϕ α + γ + γ −α − γ − γ −α =

(cos cos( ) cos( ) cos( ))senϕ γ + γ −α − γ −α + γ +α

⇔( ) ( )( ) ( )

( )( )

2 2 2

22 2

2cos coscotg cotg

cos 22 cos

sen sen sen

sen sen sen

β+γ β−γ π−α

β−γ β+γ π−α

γ − β αϕ = = = =β − γ

Como cotg ϕ = cotg 2

α e ϕ e

2

α pertencem ao intervalo ]0; π[, temos ϕ =

2

α, o

que precisávamos.

PROBLEMA 2Determine todos os polinômios P(x) de coeficientes reais que satisfazem a igualdade

( ) ( ) ( ) 2 ( )− + − + − = + +P a b P b c P c a P a b c

para quaisquer números reais a, b, c, tais que ab + bc + ca = 0.

SOLUÇÃO DE FÁBIO DIAS MOREIRA (RIO DE JANEIRO - RJ)Note inicialmente queP(a – b) + P(b – c) + P(c – a) = 2P(a + b + c)P(b – a) + P(a – c) + P(c – b) = 2P(b + a + c) (*)P(b – a) + P(c – b) + P(a – c) = 2P(– a – b – c) (**)

Page 17: Eureka 2005

Sociedade Brasileira de Matemática

EUREKA! N°21, 2005

17

Desde que ab + bc + ca = 0 (a primeira equação é uma aplicação direta; a segundasegue se permutarmos a e b; a terceira se levarmos (a; b; c) em (– a; – b; – c), o queé permitido pois (– a)(– b) + (– b)(– c) + (– c)(– a) = ab + bc + ca = 0).

Igualando (*) e (**), P(a + b + c) = P(– a – b – c) desde que ab + bc + ca = 0. Masdado S real, se tomarmos a = S, b = 0 e c = 0, então a + b + c = S e ab + bc + ca = 0.Logo P(x) = P(–x) para todo x real, ou seja, P é par. Em particular, isto implica queo grau de P, gr(P), é par.

Agora vem a parte da solução que usa análise. Ele provou um lema geral bem útilpara o problema.

Note inicialmente que se P(x) = k para todo x real então 3k = 2k ⇔ k = 0 que, defato, é uma solução. Suponha agora que P não é constante, de grau n. Seja Qx(λ) =P(xλ) (aqui, Fábio usa uma idéia relacionada a homogeneidade). Fixado x, Qx é umpolinômio em λ de grau ≤ n (de fato, de grau n se 0x ≠ e de grau 0 se x = 0).

Lema. ( )

lim nxnn

QC x

λ→+∞

λ=

λ, sendo Cn o coeficiente líder de P(x).

Demonstração:Note que Qx(λ), Qx’(λ), …, Qx

(n –1)(λ), λn, λn –1, …, λ são todos polinômios, logo sãoderiváveis, e que para um desses polinômios qualquer, digamos R, temoslim | ( ) | Rλ→∞

λ = +∞ , pois todos eles têm grau maior ou igual a 1. Então, aplicando o

teorema de L’Hôpital repetidas vezes, temos ( )( ) ( )

lim lim!

nx x

n

Q Q

nλ→+∞ λ→+∞

λ λ=

λ. Mas se P(x)

= Cnxn + … + C1x + C0, Qx(λ) = Cnx

nλn + … + C1xλ + C0, logo Qx(n)(λ) = n!Cnx

n,logo o limite desejado é mesmo Cnx

n.

Agora, Fábio aproveita-se da homogeneidade da condição ab + bc + ca = 0: comoab + bc + ca = 0 ⇔ (λa)(λb) + (λb)(λc) + (λc)(λa) = 0 para todo λ real, logoP(λ(a – b)) + P(λ(b – c)) + P(λ(c – a)) = 2P(λ(a + b + c)) ⇔Qa–b(λ) + Qb–c(λ) + Qc–a(λ) = 2Qa+b+c(λ) para todos λ, a, b, c reais, ab + bc + ca = 0.

Como cada um dos limites ( )

lim xn

Qλ→+∞

λλ

, x ∈ {a – b; b – c; c – a}, n = gr(P) existe,

existe o limite da soma, que é a soma dos limites. Logo

Page 18: Eureka 2005

Sociedade Brasileira de Matemática

EUREKA! N°21, 2005

18

( ) ( ) ( ) ( )lim 2 lim

( ) ( ) ( ) 2 ( )

a b b c c a a b cn n

n n n nn n n n

Q Q Q Q

C a b C b c C c a C a b c

− − − + +

λ→∞ λ→∞

λ + λ + λ λ=

λ λ⇔ − + − + − = + +sendo a, b, c reais tais que ab + bc + ca = 0 e n = gr(P). Como Cn, coeficiente líderde P, é não-nulo por hipótese, (a – b)n + (b – c)n + (c – a)n = 2(a + b + c)n (***)Tome a = – 2, b = 3 e c = 6 (note que ab + bc + ca = 0). Então, substituindo em(***), (–5)n + (–3)n + 8n = 2 ⋅ 7n

Como n é par, obtemos 5n + 3n + 8n = 2 ⋅ 7n. Se n ≥ 7,7

8 1 18 7 7 1 7 1 7 2 7

7 7 7

nn n n n n = ≥ + > + ⋅ = ⋅

, absurdo.

Logo n ≤ 6. Mas n = 6 implica 0 ≡ 2 ⋅ 76 ≡ 36 + 56 + 86 ≡ 1 + 1 + 1 ≡ 3 (mod. 7),absurdo. Logo P(x) = Ax2 + B ou P(x) = Ax4 + Bx2 + C.

Fábio não precisava estudar o caso P(x) = Ax2 + B, pois isso é um caso particular docaso P(x) = Ax4 + Bx2 + C, mas ele estudou. Na hora de estudar o caso P(x) = Ax4 +Bx2 + C, ele viu que a conta seria grande. Mas nesse momento ele lembrou aspalavras que o prof. Luciano sempre diz: “tem que ter garra para ser campeão!” enão fraquejou, fazendo as contas:

Substitua a = b = c = 0 na equação do enunciado. Segue imediatamente que C = 0.Logo P(x) = Ax4 + Bx2. Substituindo (isso mesmo!) e tendo em mente queab + bc + ca = 0 e que “hay que hacer las cuentas, pero sin perder la simetríajamás”,(a – b)2[A(a – b)2 + B] + (b – c)2[A(b – c)2 + B] + (c – a)2[A(c – a)2 + B] == 2(a + b + c)2 [A(a + b + c)2 + B]⇔ A[(a – b)4 + (b – c)4 + (c – a)4] + B[(a – b)2 + (b – c)2 + (c – a)2]= 2A(a + b + c)4 + 2B(a + b + c)2

⇔ A[a4 – 4a3b + 6a2b2 – 4ab3 + b4 + b4 – 4b3c + 6b2c2 – 4bc3 + c4 + c4 – 4c3a + 6c2a2

– 4ca3 + a4] + B[a2 – 2ab + b2 + b2 – 2bc + c2 + c2 – 2ca + a2]= 2A(a2 + b2 + c2 + 2ab + 2bc + 2ca)2 + 2B(a2 + b2 + c2 + 2ab + 2bc + 2ca)⇔ 2A[a4 + b4 + c4 – 2ab(a2 + b2) – 2bc(b2 + c2) – 2ca(c2 + a2) + 3a2b2 + 3b2c2 +3c2a2] + 2B[a2 + b2 + c2] = 2A[a4 + b4 + c4 + 2a2b2 + 2b2c2 + 2c2a2] + 2B[a2 + b2 +c2] ⇔ A[a2b2 + b2c2 + c2a2 – 2ab(a2 + b2) – 2bc(b2 + c2) – 2ca(c2 + a2)] = 0⇔ A[a2b2 + b2c2 + c2a2 – 2a2(ab + bc + ca) + 2a2bc – 2b2(ab + bc + ca) +2ab2c – 2c2(ab + bc + ca) + 2abc2] = 0 ⇔ A[(ab + bc + ca)2 – 2abc(a + b + c) +2abc(a + b + c)] = 0 ⇔ 0 = 0.

Page 19: Eureka 2005

Sociedade Brasileira de Matemática

EUREKA! N°21, 2005

19

Como todas as passagens são equivalências, não há restrição sobre A e B. Portantoos únicos polinômios que satisfazem as condições do enunciado são os da formaP(x) = Ax4 + Bx2, A, B reais.

PROBLEMA 3Um gancho é uma figura formada por seis quadrados unitários como no seguintediagrama

ou qualquer uma das figuras obtidas desta aplicando rotações ou reflexões.Determine todos os retângulos m × n que podem ser cobertos comganchos de modo que:i) O retângulo é coberto sem buracos e sem sobreposições;ii) Nenhuma parte de nenhum gancho pode cobrir regiões fora do retângulo.

SOLUÇÃO OFICIALConsidere um preenchimento do tabuleiro m × n. Para cada gancho A, existe umúnico gancho B cobrindo o quadradinho “interno” de A com uma de suas“extremidades”. Além disso, o quadradinho “interno” de B deve ser coberto por umadas “extremidades” de A. Assim, num recobrimento, todos os ganchos formampares. Há apenas duas maneiras de formar tais pares. Em um caso, A e B formam umretângulo 3 × 4; no outro, a sua união é um octógono, com lados 3, 2, 1, 2, 3, 2, 1, 2.Então um tabuleiro m × n pode ser coberto por ganchos se e somente se pode sercoberto com os pares de 12 quadradinhos acima. Suponha que tal recobrimentoexista. Então mn é divisível por 12. Provaremos que m ou n é divisível por 4.Assuma por absurdo que isso não acontece; então m e n são ambos pares, pois mn édivisível por 4. Imagine o tabuleiro dividido em quadradinhos unitários, com linhase colunas rotuladas 1, 2, …, m e 1, 2, …, n, respectivamente. Escreva 1 noquadradinho (i, j) se exatamente um entre os números i e j é divisível por 4, e 2, se ie j são ambos divisíveis por 4. Como o número de quadrados em cada linha e colunaé par, a soma de todos os números escritos é par. Mas não é difícil verificar que umretângulo 3 × 4 sempre cobre números com soma 3 ou 7; e o outro tipo de parsempre cobre números com soma 5 ou 7. Conseqüentemente, o número de pares depeças é par. Assim, mn é divisível por 24 e, portanto, por 8, absurdo, poissupusemos que nem m nem n é múltiplo de 4.

Page 20: Eureka 2005

Sociedade Brasileira de Matemática

EUREKA! N°21, 2005

20

Note também que nem m nem n pode ser 1, 2 ou 5 (qualquer tentativa de colocar aspeças nesses casos falha). Concluímos então que se um recobrimento é possívelentão m ou n é divisível por 3, m ou n é divisível por 4 e m, n ∉ {1; 2; 5}.

Reciprocamente, se essas condições acima são verificadas, o recobrimento épossível (utilizando somente retângulos 3 × 4). Isso é imediato quando 3 divide m e4 divide n (ou vice-versa). Seja m divisível por 12 e n ∉ {1; 2; 5} (ou vice-versa).Então n pode ser representado como a soma de vários 3’s e vários 4’s. Então otabuleiro pode ser particionado em retângulos m × 3 e m × 4, que são fáceis decobrir, utilizando novamente retângulos 3 × 4.

SEGUNDO DIA

PROBLEMA 4Seja 3≥n um inteiro. Sejam 1 2, ,..., nt t t números reais positivos tais que

21 2

1 2

1 1 11 ( ... ) ... .

+ > + + + + + +

n

n

n t t tt t t

Mostre que ,i jt t e kt são as medidas dos lados de um triângulo para quaisquer i, j, k

com 1 .≤ < < ≤i j k n

SOLUÇÃO DE HENRY WEI CHENG HSU (SÃO PAULO - SP)Suponha tn ≥ tn – 1 ≥ … ≥ t2 ≥ t1 e seja tn = t1 + t2 + k.

Temos

2 1 2 11 2 1

1 2 1 1 2 1

1 1 1 1 1 11 ( ) 1n

n nn n n

t t tn t t t t

t t t t t t t−

−− −

+ + ++ > + + + + + + + + + + + +

�� � �

Por Chebyshev (na verdade, poderíamos usar Cauchy ou MA-MH…),

1 2 11 2 1

21 2 1

1 2 1

1 1 1

( ) 1

1 1 1

1 1 1( ) ( 1)

nn

nn

t t tt t t n

n n n

t t t nt t t

−−

−−

+ + + + + + − ≥

− − −

⇔ + + + + + + ≥ −

��

� �

Assim,

Page 21: Eureka 2005

Sociedade Brasileira de Matemática

EUREKA! N°21, 2005

21

2 2 1 2 1

1 2 1

1 1 11 ( 1) 1 n

nn n

t t tn n t

t t t t−

+ + ++ > − + + + + + +

��

⇔ 3 11 2

1 2 3 1

2 1 n n n n n

n n n

t t t t t tt tn

t t t t t t−

+ ++− > + + + + + +�

Como 2k n

n k

t t

t t+ ≥ , temos

1 2 1 2

1 2 1 2

2 1 2 6 5n n n n

n n

t t t tt t t tn n

t t t t t t

+ +− > − + + + ⇔ > + +

Sendo tn = t1 + t2 + k,

1 2 1 2 1 2 2 1

1 2 1 2 1 2 1 2 2 1

1 2

2 1 2 1 1 2

5 5 1 1 1

2

t t t t k t t k t tk k k

t t k t t t t k t t t t

t t k k k

t t t t t t k

+ + + + +> + + ⇔ > − + + + + + ++ + + +

⇔ > + + + −+ +

Mas 1 2

2 1

2t t

t t+ ≥ . Deste modo,

1 2 1 2 1 2

1 1 1 1 1 10 0

n

k kt t t t k t t t

> + − ⇔ > + − + +

Veja que 1 2

1 1 1

nt t t+ > pois t1 ≤ t2 ≤ tn. Logo k < 0 e tn < t1 + t2.

O resto é mais fácil: sejam 1 ≤ i < j ≤ n. Temos ti + tj ≥ t1 + t2 > tn ≥ tk para1 ≤ k ≤ n e, portanto, ti, tj e tk são sempre lados de um triângulo.

PROBLEMA 5

Num quadrilátero convexo ABCD a diagonal BD não é bissetriz do ângulo �ABC

nem do ângulo �CDA . Um ponto P no interior de ABCD satisfaz

∠ = ∠PBC DBA e .∠ = ∠PDC BDA

Prove que os vértices do quadrilátero ABCD pertencem a uma mesma circunferênciase e só se AP = CP.

Page 22: Eureka 2005

Sociedade Brasileira de Matemática

EUREKA! N°21, 2005

22

SOLUÇÃO DE RAFAEL DAIGO HIRAMA (CAMPINAS - SP)

B

A P

C

D

α1 α2 α1

β1

β2

β1

Primeiro ato: considerações gerais.Vamos determinar a figura a partir de quatro ângulos: sejam ∠ABD = α1, ∠BDA =β1, ∠CBD = α2, ∠CDB = β2. Então ∠DBP = α2 – α1 e ∠BDP = β2 – β1.

Veja que esses ângulos não dependem de P estar dentro do triângulo ABD (o queimplica ∠ABD > ∠DBC e ∠ADB > ∠BDC) ou P estar dentro do triângulo BCD (oque implica ∠ABD < ∠DBC e ∠ADB < ∠BDC) pois se ∠ABD = ∠PBC então∠ABD ± ∠DBP = ∠PBC ± ∠DBP (+ se P está dentro de BDC e – se P está dentrode ABD) ⇒ ∠ABP = ∠DBC = α2. Para os ângulos com vértice D é análogo.

Usando lei dos senos nos triângulos ABP, BCP, ADP e DCP, temos

2

AP AB

sen sen APB=

α ∠(1)

1

CP BC

sen sen BPC=

α ∠ (2)

2

AP AD

sen sen APD=

β ∠(3)

1

CP DC

sen sen DPC=

β ∠ (4)

De (1) e (4), temos

2

1

senAP AB sen DPC

CP DC sen sen APB

α ∠= ⋅ ⋅β ∠

(I)

Page 23: Eureka 2005

Sociedade Brasileira de Matemática

EUREKA! N°21, 2005

23

De (2) e (3), temos

2

1

senAP AD sen BPC

CP BC sen sen APD

β ∠= ⋅ ⋅α ∠

(II)

As equações (I) e (II) são fundamentais para a solução do problema, pois envolvemAP e CP, lados do quadrilátero ABCD e, além disso, observando a figura e essasequações, notamos algumas simetrias interessantes. Rafael começa a usá-las a partirde agora.

Vamos lá: sen∠APD = sen(∠APD + ∠BPC – ∠BPC) = sen(∠APD + ∠BPC) ⋅cos∠BPC – cos(∠APD + ∠BPC) ⋅ sen∠BPC.

Seja θ = ∠APD + ∠BPC = 360º – (∠APB + ∠CPD). Logo sen∠APD =senθ ⋅ cos∠BPC – cosθ ⋅ sen∠BPC (III) e sen∠APB = sen(∠APB + ∠CPD –∠CPD) = sen(∠APB + ∠CPD) ⋅ cos∠CPD – sen(∠APB + ∠CPD) ⋅ cos∠CPD =–senθ ⋅ cos∠CPD – cosθ ⋅ sen∠CPD (IV) (lembre-se de que sen(360º – x) = –sen xe cos(360º – x) = cos x).

Segundo ato: ABCD cíclico ⇒ AP = CP.Supondo que ABCD é cíclico (esta é a primeira vez que vamos usar este fato),

temos, pela lei dos senos estendida, que 1 2

AB CD

sen sen=

β α e

1 2

AD BC

sen sen=

α β.

Substituindo essas duas últimas igualdades e (III) e (IV) em (I) e (II),respectivamente, temos

(VI): cosPC sen APD sen

AP sen BPC tg BPC

∠ θ= = − θ∠ ∠

(V): cosPC sen APB sen

AP sen CPD tg CPD

∠ θ= = − − θ∠ ∠

Logo tg∠BPC = – tg ∠ CPD ou senθ = 0. A primeira possibilidade implica ∠BPC +∠CPD = 180º, mas aí B, P e C seriam colineares e então BD bissectaria os ângulos∠ABC e/ou ∠ADC.

Portanto senθ = 0 e então θ = 180º, pois θ = ∠APD + ∠BPC está entre 0 e 360º.

Então 180

cos180PC sen

AP tg BPC= −∠

� = 1, ou seja, AP = PC.

Page 24: Eureka 2005

Sociedade Brasileira de Matemática

EUREKA! N°21, 2005

24

Terceiro ato: AP = CP ⇒ ABCD cíclico.

De (II) e (I), temos 2

1

senAD sen APD

BC sen sen BPC

β ∠⋅ =α ∠

e 1

2

senCD sen CPD

AB sen sen APB

β ∠⋅ =α ∠

.

Multiplicando, temos 2 1

1 2

sen senAD CD sen APD sen CPD

BC sen AB sen sen BPC sen APB

β β ∠ ∠⋅ ⋅ ⋅ = ⋅α α ∠ ∠

.

Aplicando a lei dos senos em ABD e BCD temos 1 1

AD AB

sen sen=

α β e

2 2

CD BC

sen sen=

α β.

Assim, sen APD sen APB

sen BPC sen CPD

∠ ∠=∠ ∠

Mas, fazendo as mesmas contas que fizemos em (V) e em (VI) (veja que asigualdades à direita em (V) e (VI) não dependem de ABCD ser cíclico), vemos quesen APD sen APB

sen BPC sen CPD

∠ ∠=∠ ∠

= 1. E então 2

1

1senAD

BC sen

β× =α

⇔ 1 2

AD BC

sen sen=

α β. Sejam R1

e R2 os circunraios de ABD e BCD. Pela lei dos senos estendida,

1 21 2

2 2AD BC

R Rsen sen= = =

α β ⇒ R1 = R2.

Logo os triângulos ABD e BCD têm o mesmo circunraio. Só temos duaspossibilidades para um circuncírculo de raio fixado em um segmento fixado: umadas possibilidades é A, B, C e D serem concíclicos, que é o que queremos; a outra éa circunferência que passa por A, B e D ser a simétrica da que passa por B, C e D emrelação a BD.

B

C

D

A

α1α2

β1 β2

Page 25: Eureka 2005

Sociedade Brasileira de Matemática

EUREKA! N°21, 2005

25

Neste último caso vamos ter ∠BAD = ∠BCD. Como α1 + β1 + ∠BAD = 180º e α2 +β2 + ∠BCD = 180º, α1 + β1 = α2 + β2.

Mas veja que (α1 > α2 e β1 > β2) ou (α1 < α2 e β1 < β2) pois caso contrário, como Pestá no interior do quadrilátero ABCD, P deveria estar no interior de ABD e BCD aomesmo tempo, o que é impossível. Logo não é possível que α1 + β1 = α2 + β2, e oproblema acabou.

Enfim, uma solução muito parecida com uma trilogia grega, porém sem nenhumatragédia.

PROBLEMA 6Um inteiro positivo é dito alternante se, na sua representação decimal, quaisquerdois dígitos consecutivos têm paridade diferente.Determine todos os inteiros positivos n tais que n tem um múltiplo que é alternante.

SOLUÇÃO DE RAFAEL MARINI SILVA (RIO DE JANEIRO - RJ)O principal diferencial da solução do Marini é que, em vez de procurar exemplosparticulares para múltiplos alternantes de n, como a maioria dos estudantes fez, eledeu uma caracterização bastante interessante dos números alternantes. Vamos àsolução:

Seja T o conjunto dos números cuja representação decimal consiste em algarismostodos menores do que 5. Veja que todo número alternante a pode ser escrito em umadas duas seguintes formas:

se a é ímpar, a = 2 1 dígitos

101010...101n−

������� + 2k = 210 1

99

n − + 2k, k ∈ T, k com no máximo 2n

dígitos; (I)

se a é par, a = 2 dígitos

101010...1010n

������� + 2k = 210 1

99

n − ⋅ 10 + 2k, k ∈ T, k com no máximo

2n + 1 dígitos; (II)

Agora, Marini divide o problema em casos:

Caso 1: mdc(10; p) = 1.Aqui, Marini toma sim um número particular. Não é muito difícil encontrar umnúmero alternante nesse caso, se você conhece o teorema de Euler-Fermat.

Page 26: Eureka 2005

Sociedade Brasileira de Matemática

EUREKA! N°21, 2005

26

Tome k = 0 na forma (I) e n = ϕ (99p). Pelo teorema de Euler-Fermat, 10ϕ (99p) ≡ 1

(mod 99p) (veja que mdc(10; 99p) = 1) e portanto o número alternante 2 (99 )10 1

99

pϕ − é

múltiplo de p.

Caso 2: p é par, mas não é múltiplo de 5.Assim, p = 2e+1u, sendo u ímpar e não múltiplo de 5. O número alternante que

devemos encontrar é da forma (II). Assim, 2e+1u|210 1

99

n − ⋅ 10 + 2k, ⇔

2eu|210 1

99

n − ⋅ 5 + k, k ∈ T, k com no máximo 2n + 1 algarismos.

A nova dificuldade nesse caso é a potência de 2, ou seja, que 2e|210 -1

99

n

⋅ 5 + k. Para

n suficientemente grande (maior que e/2), temos 210 1

99

n − ⋅ 5 + k ≡ 0 (mod 2e) ⇔

k ≡ 5 ⋅ 99–1 (mod 2e). Marini resolveu, então, provar um lema um pouco mais geral,mas que faz muito sentido: o conjunto T contém uma boa parte dos númerosinteiros, então…

Lema. Fixados a e m naturais e {2,5}q∈ , existe t ∈ T tal que t ≡ a (mod qm).

Demonstração. Indução sobre m.

Base de indução: m = 1: imediato, pois T > {0, 1, 2, 3, 4} > {0, 1}.

Passo de indução: suponha o fato válido para m = k. Vamos provar para m = k + 1.

Suponha que 0 10 .kt≤ < Seja Xt = {t; t + 10k; …; t + (q – 1) ⋅ 10k}. Note que esseconjunto está contido em T (pois q ≤ 5) e que Xt contém todos os possíveis restoscongruentes a t módulo qk+1 (qk+1 não divide 10k e q divide 10). Da hipótese deindução, dado x natural existe t T∈ com t T∈ com 10kt < (se 10kt ≥ , trocamos tpelo número formado pelos seus k últimos dígitos que é congruente a t módulo 10k,e logo módulo qk) tal que x ≡ t (mod qk). Vendo módulo qk+1 temos que x écongruente a um elemento de Xt. Considerando a união de todos os conjuntos Xt,com t variando entre 0 e qk – 1, vemos que cobrimos todos os possíveis restos (defato, como Xt tem q elementos, a união tem qk+1 elementos) e portanto o lema estádemonstrado.

Page 27: Eureka 2005

Sociedade Brasileira de Matemática

EUREKA! N°21, 2005

27

Assim, aplicando o lema à congruência k ≡ 5 ⋅ 99–1 (mod 2e), vemos que podemosescolher k0 ∈ T que satisfaz essa congruência. Seja m o número de dígitos de k0.Tome s cópias de k0 (com alguns zeros à esquerda para que fique com m + e + 1

dígitos) e concatene-as, isto é, seja k = k0

1( 1)

0

10s

m e j

j

−+ +

=∑ = k0

( )11

10 1

10 1

sm e

m e

+ +

+ +

− −

. Veja que

k ainda é um elemento de T e que k ≡ 5 ⋅ 99–1 (mod 2e), pois 10m+e+1 ≡ 0 (mod 2e).Vamos fazer com que k seja múltiplo de u. Para isso, basta tomar s = ϕ((10m+e+1 – 1)⋅ u) que o resto segue de Euler-Fermat.

Retomando: o número alternante que procuramos é a = 210 1

99

n − ⋅ 10 + 2k. Já

encontramos k. Agora, basta escolher um n adequado par acertar o número dealgarismos de a. Tome n = ϕ(99u)v maior que a quantidade de dígitos de k. Pronto!Temos

a =210 1

99

n − ⋅ 10 + 2k = 2(

210 1

99

n − ⋅ 5 + k) ≡ 0 (mod 2e+1) (graças ao lema!)

a = 210 1

99

n − ⋅ 10 + 2k ≡ 0 + 0 ≡ 0 (mod u)

Logo a é múltiplo de p.

Caso 3: p é múltiplo de 5, mas é ímpar.Seja p = 5eu, u ímpar. Agora vamos procurar um número alternante da forma (I), ou

seja, da forma 210 1

99

n − + 2k, k ∈ T. Para n suficientemente grande,

210 1

99

n − + 2k ≡ 0 (mod 5e) ⇔ k ≡ 198–1 (mod. 5e)

Novamente pelo lema, existe 0k T∈ que satisfaz essa congruência. Assim como no

caso anterior, tomemos k = k0

11

0

10s

m e

j

−+ +

=∑ = k0

( )11

10 1

10 1

sm e

m e

+ +

+ +

− −

, com

s = ϕ((10m+e+1 – 1) ⋅ u) e n = ϕ(99u)v maior que a quantidade de dígitos de k. Demodo análogo ao caso anterior, conseguimos um alternante múltiplo de p.

Caso 4: p é par e múltiplo de 5, ou seja, p é múltiplo de 10.

Page 28: Eureka 2005

Sociedade Brasileira de Matemática

EUREKA! N°21, 2005

28

Neste caso, devemos tomar um alternante da forma (II), ou seja, a = 210 1

99

n − ⋅ 10 +

2k, k ∈ T. Seja p = 10u. Temos que 10u|210 1

99

n − ⋅ 10 + 2k ⇒ 5|k. Como k ∈ T, seu

último dígito só pode ser zero, de modo que 10|k. Seja k = 10w, w ∈ T. Temos então

que u|210 1

99

n − + 2w, w ∈ T. Como

210 1

99

n − + 2w é ímpar, u deve ser ímpar, e

caímos no caso 1 ou 3. Logo existe um múltiplo alternante de u só quando u é ímpar,ou seja, quando p não é múltiplo de 20.

Logo os inteiros positivos que não possuem múltiplo alternante são exatamente osmúltiplos de 20.

���� �����

��� �9�� �� �������8�� ��� �� ���� ��#� �������� �

-���� ,������ ������ �� ������ ����� /� ,���$�� ����� ���� � �� :�� .���� � ;������ ;�%��� �9<-������'� � ��� � � �� ! ��� ��2�� ���#�� �� � �� ��� < �- �� ������� ���� �� � �� �������7! ��$�� ��� ����- �� �� 4�� ��� =��� � ����$� �� �>�> �� �9�� �� ��,��� �� �������8����� �� ���� �� �?� ���� ,������ �� ������4�2� �� �� %�� ������� � ����� �! ��#-����' �� .����� ;� �� � �566��9�$���6�#�6�� ��);6�&�&��� � �� �� �9��� @�� �� :�A�� B�� �#�� ����� ��� �566�� ���� �*�� �������6C+��6������6���������,

Page 29: Eureka 2005

Sociedade Brasileira de Matemática

EUREKA! N°21, 2005

29

CENTRO DE MASSA E APLICAÇÕES À GEOMETRIAEmanuel Carneiro & Frederico Girão – UFC

Nível Avançado

1. INTRODUÇÃO

Chamaremos de sistema de massas um conjunto de n pontos 1 2, ,..., nP P P no plano,

sendo que ao ponto ( , )k k kP x y= está associada uma massa ,km ∈� de modo que

1 2 ... 0.nm m m+ + + ≠ Definiremos o centro de massa desse sistema como sendo o

ponto ( , )x y tal que:

1 1 2 2 1 1 2 2... ...; n n n nm x m x m x m y m y m y

x yM M

+ + + + + += = ,

Onde 1 2 ... nM m m m= + + + é a massa associada a ele.

Notação:Quando ao ponto (x, y) estiver associada uma massa m escreveremos (x, y)[m].

Observações:(i) Podemos interpretar fisicamente o centro de massa de um sistema como sendo oponto onde ele concentra toda sua massa. Em termos práticos, isso nos ajuda asimplificar, por exemplo, problemas de Dinâmica onde há aplicações de forças sobreo sistema.

(ii) Podemos considerar os pontos em n� . Neste caso, o cálculo do centro de

massa de um sistema é análogo.

(iii) Claramente o centro de massa é único.

2. PROPRIEDADES BÁSICAS

Proposição 1.Seja (x, y)[m] o centro de massa do sistema

1 1 1 1 2 2 2{( , )[ ], ( , )[ ], ..., ( , )[ ]},k k kS x y m x y m x y m= e seja (a, b)[N] o centro de

massa do sistema 2 1 1 1 2 2 2{( , )[ ], ( , )[ ],..., ( , )[ ]}.l l lS a b n a b n a b n= Então, se

Page 30: Eureka 2005

Sociedade Brasileira de Matemática

EUREKA! N°21, 2005

30

0,M N+ ≠ o centro de massa do sistema 1 2S S S= ∪ é o centro de massa do

sistema {( , )[ ], ( , )[ ]}.x y M a b N

Demonstração:

Por definição o centro de massa do sistema 1 2S S S= ∪ é o ponto

( , )[ ],X Y M N+ onde:

1 1k li i i j j j x a

m x n a M NX

M N M N= =+ +

= =+ +

∑ ∑

que é justamente a primeira coordenada do centro de massa do sistema{( , )[ ], ( , )[ ]}.x y M a b N Para a segunda coordenada é análogo.

A proposição acima nos dá um algoritmo para calcular o centro de massa de um

sistema com n pontos. Para isso tomamos dois pontos 1 1 1( , )[ ]x y m e

2 2 2( , )[ ]x y m quaisquer desse sistema e os substituímos pelo seu centro de massa

com a massa 1 2m m+ . Recaímos assim num sistema com n – 1 pontos econtinuamos o processo. Assim o cálculo de centros de massa resume-se apenas aocaso n = 2, que estudamos a seguir:

Centro de massa de um sistema com duas massas

O centro de massa ( , )[ ]x y M de um sistema 1 1 1 2 2 2{( , )[ ], ( , )[ ]}x y m x y m é

colinear com os pontos 1 1( , )x y e 2 2( , )x y pois

1 1

2 2 1 2 1 2 2 2 1 1

1

1

1

x y

x y x y xy x y xy x y x y

x y

= + + − − −

1 1 2 2 1 1 2 21 2 1 2

1 2 1 2

1 1 2 2 1 1 2 22 2 1 1

1 2 1 2

0

m x m x m y m yx y y x

m m m m

m x m x m y m yy x y x

m m m m

+ += + + + + + +− − − + + =

E além disso se chamamos 1 1( , ) ,x y A= 2 2( , )x y B= e ( , )x y G= vale que:

Page 31: Eureka 2005

Sociedade Brasileira de Matemática

EUREKA! N°21, 2005

31

1 2 0m AG m BG+ =���� ����

tal fato é deixado como exercício para o leitor.

Observação:Pela equação acima distinguimos alguns casos:

• As duas massas têm o mesmo sinal. Nesse caso o ponto G está entre A e B e vale

que 1 2 .m AG m BG=

• As duas massas têm sinais contrários. Nesse caso G está fora do segmento AB e

vale que 1 2m AG m BG= .

3. APLICAÇÕES À GEOMETRIAExemplo 1: Vamos tomar um triângulo ABC qualquer e pôr massas iguais em seustrês vértices, ou seja consideraremos o sistema [ ], [ ], [ ].A p B p C p Chamaremos deG o centro de massa desse sistema. Como encontrar o ponto G?(hummm…) denotaremos C.M. = centro de massa.

A[p]

B[p] C[p]M[2p]

NPG

Vamos usar a proposição da seção anterior. O.C.M. de [ ]B p e [ ]C p é o seu ponto

médio M. Podemos então trocar [ ]B p e [ ]C p por [2 ].M p Logo o ponto G será o

C.M. de [ ]A p e [2 ],M p que está sobre AM e divide AM na razão

2.

1

AG

GM=

Page 32: Eureka 2005

Sociedade Brasileira de Matemática

EUREKA! N°21, 2005

32

Sejam N e P os pontos médios de AC e AB. De modo análogo poderíamos terprovado que G BN∈ e que .G CP∈ Esta é uma demonstração diferente que astrês medianas concorrem em G, que é portanto o baricentro do triângulo. Alémdisso, segue do exposto acima que:

2

1

AG BG OG

GM GN GP= = =

Exemplo 2: Denote por a, b, c, os lados do triângulo ABC da maneira usual.Vamos pôr agora massas nos vértices do triângulo proporcionais aos lados opostos,

ou seja, considere o sistema [ ], [ ], [ ].A a B b C c Seja I o C.M. desse sistema.Você merece um prêmio se descobrir quem é I…

A[a]

B[b] A[a]

C[c]J[b+c]

L

K

I

a

bc

O raciocínio é igual ao do exemplo anterior. O C.M. de [ ]B b e [ ]C c é um ponto J

no lado BC tal que ,JB c

JC b= ou seja, J é o pé da bissetriz interna. Logo I será o

C.M. de [ ]A a e [ ].J b c+ Tiramos daí que I AJ∈ e que .AI b c

IJ a

+=

Sejam BL e CK bissetrizes internas. De modo análogo poderíamos ter provado queI BL∈ e que ,I CK∈ o que mostra que I é o incentro. As razões saem de graça:

; BI a c CI a b

IL b IK c

+ += =

Exemplo 3: Seja p o semiperímetro do triângulo. Agora uma novidade: o sistema de

massas será [ ], [ ], [ ].A p a B p b C p c− − − Seja N o C.M. desse sistema.Você realmente merece um prêmio se descobrir quem é o N.

Page 33: Eureka 2005

Sociedade Brasileira de Matemática

EUREKA! N°21, 2005

33

A[p – a]

B [p – b] C[p – c] X[a]

Y Z

N

p – c p – b

p – b p – c

p – a p – a

O C.M. de [ ] e [ ]B p b C p c− − é um ponto X sobre o lado BC tal que

,BX p c

CX p b

−=− donde concluímos que BX p c= − e que .CX p b= − Este

ponto X é onde o exincírculo relativo ao lado a toca este lado (como referência sobreeste fato podemos indicar [1]). Logo N será o C.M. de [ ]A p a− e

[ ] [ ].X p c p b X a− + − =

Portanto N AX∈ e .AN a

NX p a=− Se considerarmos os pontos Y e Z onde os

exincírculos relativos aos lados b e c tocam estes lados, respectivamente, podemosmostrar que N BY∈ e .N CZ∈ Conclusão: AX, BY e CZ sào concorrentes em Nque é chamado Ponto de Nagel do .ABC∆ Ora, ora, poderíamos saber disso usandoo teorema de Ceva (veja por exemplo [3]). Calma, o melhor ainda está por vir. Asrazões aqui são cortesias para nós:

; BN b CN c

NY p b NZ p c= =− −

O próximo resultado foi o que nos motivou a escrever este artigo. Ele mostra toda abeleza desta teoria, enquanto outros métodos são ineficazes. Para uma demonstração

Page 34: Eureka 2005

Sociedade Brasileira de Matemática

EUREKA! N°21, 2005

34

completa (e bastante extensa) do próximo resultado usando a geometria planaclássica, veja [2].

Teorema 3.1. No ABC∆ considere os pontos I, G e N como definidos acima.Vale então que I, G e N são colineares e ainda:

2

1

NG

GI=

Prova: Seja p o semiperímetro do triângulo. Considere um sistema de massas[ ], [ ], [ ].A p B p C p Já sabemos que o C.M. desse sistema é o baricentro G.

Fazendo uso da proposição 1, podemos dividir esse sistema em dois subsistemas

1 [ ], [ ], [ ]S A a B b C c= e 2 [ ], [ ], [ ].S A p a B p b C p c= − − − O C.M. de 1S é o

incentro I com massa [ ] [2 ],a b c p+ + = enquanto o C.M. de 2S é o ponto da

Nagel N com massa [ ] [ ].p a p b p c p− + − + − = Logo G será o C.M. de

[2 ], [ ]I p N p o que implica I, N, G colineares (com G entre I e N) e ainda pelaequação do momento linear:

2

1

NG

GI=

B[p] =[b + (p – b)] C[p] =[c + (p – c)]

H

GO

I[p]

N[2p]

x

2x2y

y

A[p] =[a + (p – a)]

Corolário 3.1.1. Em um triângulo qualquer ABC, sejam I,G,N como acima, O ocircuncentro e I o ortocentro. Então os pontos I, O, N, H formam um trapézio.

Page 35: Eureka 2005

Sociedade Brasileira de Matemática

EUREKA! N°21, 2005

35

Prova: Sabemos que H, G, O são colineares (reta de Euler) e que:

2

1

HG

GO=

Segue então do teorema anterior que IO é paralelo a NH, logo I, O, N, H formam umtrapézio, cujo encontro das diagonais é G.

Podemos aplicar estes métodos do centro de massa em problemas que envolvem oortocentro, o baricentro e os exincentros, para saber que massas devem estar nosvértices, veja o problema 1. Divirta-se resolvendo estes problemas. Vale usar tudo,mas experimente a sua mais nova arma.

4. PROBLEMAS RELACIONADOS

Problema 1

(a) Verifique que o sistema , ,cos cos cos

a b cA B C

A B C

tem como C.M. o

ortocentro do triângulo.

(b) Verifique que o sistema [ 2 ], [ 2 ], [ 2 ]A sen A B sen B C sen C tem como C.M. ocircuncentro.

(c) Prove que o C.M. do sistema [ ], [ ], [ ]A a B b C c− é o exincentro relativo ao ladoa. Verifique os análogos para os outros exincentros.

Problema 2

Sejam A, B, C, D pontos concíclicos. Sejam , , ,A B C DG G G G os baricentros dos

triângulos BCD, ACD, ABD e ABC. Prove que , , ,A B C DG G G G são concíclicos.

Problema 3Sejam ABCD um quadrilátero no espaço de forma que AB, BC, CD e DA sejamtangentes a uma esfera γ nos pontos X, Y, Z, W. Prove que estes pontos sãocoplanares.

Problema 4Sejam X, Y e Z os pontos onde o incírculo do triângulo ABC toca os lados BC, AC eAB, respectivamente. Mostre que o incentro de ABC∆ está sobre a reta que passapelos pontos médios de BC e AX. (veja uma solução em [5]).

Page 36: Eureka 2005

Sociedade Brasileira de Matemática

EUREKA! N°21, 2005

36

Problema 5Considere 6 pontos em uma dada circunferência. Tomamos três destes pontos e

marcamos seu baricentro 1.G Em seguida, marcamos o ortocentro 2H dos outros

três pontos e traçamos o segmento 1 2 .G H Mostre que todos os 620

3

=

possíveis

segmentos 1 2G H passam por um ponto fixo.

Problema 6Seja ABCD um quadrilátero convexo inscritível com os lados opostos AD e BC seencontrando em P, e AB e CD em Q. Prove que o quadrilátero EFGH, determinado

em ABCD pelas bissetrizes de �DPC e � ,CQB é um losango.

Problema 7

Seja PABC um tetraedro e sejam 1 1 1, ,A B C os pontos médios das arestas BC, AC e

AB, respectivamente. Seja α um plano paralelo à face ABC que intercepta as arestas

PA, PB, PC nos pontos 2 2 2, ,A B C respectivamente.

(a) Prove que 1 2 1 2 1 2, ,A A B B C C concorrem em um ponto D.

(b) Determine o lugar geométrico dos pontos D quando α varia.

Problema 8(a) Considere 4 pontos que formam um sistema ortocêntrico (cada um é o ortocentrodo triângulo formado pelos outros três). Ponha massas iguais nesses 4 pontos. Proveque o centro de massa é o centro do círculo dos nove pontos de cada um desses 4triângulos (veja [1] e o problema proposto No. 107 na página 61).

(b) (Beltrami) Prove que o C.M. do sistema formado pelo incentro e pelos trêsexincentros com massas iguais é o circuncentro.

Problema 9Seja ABCD um quadrilátero convexo inscritível com os lados opostos AD e BC se

encontrando em P, e AB e CD em Q. Prove que as bissetrizes dos ângulos �DPC e�CQB e a reta que une os pontos médios das diagonais do quadrilátero ABCD

(diagonal de Euler) concorrem.

Page 37: Eureka 2005

Sociedade Brasileira de Matemática

EUREKA! N°21, 2005

37

Problema 10(Banco IMO/97) No ABC∆ acutângulo, sejam AD, BE alturas e AP, BQ bissetrizesinternas. Sejam I e O o incentro e o circuncentro do triângulo ABC, respectivamente.Prove que os pontos D, E e I são colineares se e somente se P, Q e O são colineares.

Agradecimentos: A nosso amigo Carlos Shine pela primeira versão digitada destematerial, na Semana Olímpica de 2001, em Salvador - BA.

REFERÊNCIAS:[1] Coxeter, H.S.M.; Greitzer, S.L., Geometry Revisited, MAA, 1967.[2] Johnson, R.A., Advanced Euclidean Geometry, Dover Publications, 1960.[3] Castro, L.G.M., Introdução à Geometria Projetiva, Eureka!, vol 8, pp 16-27, 2000.[4] Honsberger, R., Mathematical Morsels, MAA, 1978.[5] Moreira, C.G.T., Wagner, E., 10 Olimpíadas Iberoamericanas de Matemática, OEI,1996.

���� �����

���783045728433 2 1⋅ + � ����

�� � ����� �� ��D���� ���� � � ��� ���� ������������ � ,� ����#�� �� ��6��6���&�/� �� �� �� ��&�� �! � �� ����� �� 1�������+�

%�� �� �� �� ���- ����� � �- �� 2 1nk ⋅ + � ����

���� � n∈� E $�2� � ����+�F ��� �3� ��'� ����G���

���� �� ����� �� �� ����� ������ �� ��DD� % �� � ���� ����� �� 1�������+� �������'� �#�� � �����! �� ��#� �� �! ������ �� 1�������+� � �!%��2�� ���<�� �� ������ ��-�� ��2�'5 &�&�� �������>�&>� ������ ���>>� �&���� ���D�� ������ DD&D> �������4�2�5 � �566***���$�� ����#�� ��� ���� ������,���78�� %���-���$� �#�� �� �2���� � ��$�� ������2�� ���'�

Page 38: Eureka 2005

Sociedade Brasileira de Matemática

EUREKA! N°21, 2005

38

SEQÜÊNCIA DE FIBONACCICícero Thiago B. Magalhães

Nível Avançado

INTRODUÇÃOO nome seqüência de Fibonacci, foi dado pelo matemático francês Edouard Lucas noséculo XIX. Porém, a seqüência surgiu de um problema que estava proposto na obra"Liber Abaci" de Leonardo de Pisa (1180 – 1250), conhecido como Fibonacci. Oproblema era o seguinte: "Um homem põe um casal de coelhos dentro de um cercado.Quantos pares de coelhos serão produzidos em um ano, se a natureza desses coelhos étal que a cada mês um casal gera um novo casal, que se torna fértil a partir do segundomês?" Depois de séculos de trabalho, é possível hoje citar uma quantidade enorme depropriedades da seqüência do número de coelhos existentes após n meses. O objetivodeste trabalho é apresentar algumas propriedades básicas desta seqüência.

DefiniçãoA seqüência de Fibonacci é definida da seguinte maneira:

1 2 1f f= = e 1 2n n nf f f− −= + , 2.n∀ >Por conveniência, algumas vezes usaremos 0 0.f =

Propriedades básicas

(I) Para todo 1 2 21: ... 1n nn f f f f +≥ + + + = −Prova: 1 31: 1n f f= = −Vamos supor 1q ≥ e 1 2 2... 1q qf f f f ++ + + = −

1 2 1 2 1 31: ... 1 1q q q q qn q f f f f f f f+ + + += + + + + + = − + = −

(II) Se 1m ≥ e 1,n > então 1 1n m n m n mf f f f f+ − += +Prova: Vamos fazer indução sobre m:

1 1 1 2 11: n n n n nm f f f f f f f+ − −= = + = + (verdadeira)

2 1 2 3 1 1 12 : 2 ( )n n n n n n n n n nm f f f f f f f f f f f f+ − − − += = + = + = + + = +(verdadeira)

Seja q > 2 e suponhamos a propriedade válida para todo , 2 ,k k q≤ < e para todon > 1. Esta suposição, mais o fato de que a propriedade vale também para k = 1, nosgarante:

Page 39: Eureka 2005

Sociedade Brasileira de Matemática

EUREKA! N°21, 2005

39

( 2) 1 2 1n q n q n qf f f f f+ − − − −= +

( 1) 1 1n q n q n qf f f f f+ − − −= +Somando membro a membro essas igualdades e levando em consideração a fórmula

recursiva que define ( ) :nf

1 1n q n q n qf f f f f+ − += +Ou seja, a fórmula vale também para q, sempre que n > 1. O princípio da induçãonos garante então que vale para todo 1m ≥ e qualquer n > 1.

(III) Dois números de Fibonacci consecutivos nf e 1nf + são primos entre si.A prova fica como exercício.

(IV) Se | ,m n então | .m nf f

Prova: Por hipótese n = mq, para algum .q∈� Usaremos indução sobre r.

Se q = 1, então m = n e é fácil ver que | .m nf f Seja 1q ≥ e admitamos que

| .m mqf fEntão, usando a propriedade (II):

( 1) 1 1m q mq m mq m mq mf f f f f f+ + − += = +

Como 1|m mq mf f f− e 1|m mq mf f f + (pois, pela hipótese de indução, fm divide

),mqf então mf divide a soma desses dois produtos. Ou seja: ( 1)| .m m qf f +

(V) Seja d = mdc(m, n), então mdc ( , ) .m n df f f=Prova: Indução em m + n. Se m = 1, mdc(m, n) = 1 e mdc(fm, fn) = mdc(1, fn) = 1 = f1.Se m + n = 2 é trivial. Se m = n não há o que provar.

Se 2 ,m n≤ < ( ) 1 1 ( , )n m n m m n m m n m m nf f f f f f mdc f f+ − − − − += = + ⇒ =( III )

1( , ) ( , ),m m n m m n mmdc f f f mdc f f− − −= que é igual, por hipótese de indução a

( , ) ( , ) .mdc m n m mdc m nf f− =Veja também a solução do problema proposto No. 92 na Eureka! 20, pp 55 – 57.

(VI) Seja 2 1,x x= + então, para n = 2, 3, … nós temos que

1.n

n nx f x f −= +

Prova: É trivial o caso n = 2. E se 1n

n nx f x f −= + para algum 2,n ≥ então1

1( )n nn nx x x f x f x+

−= ⋅ = +2

1n nf x f x−= +

Page 40: Eureka 2005

Sociedade Brasileira de Matemática

EUREKA! N°21, 2005

40

1( 1)n nf x f x−= + +

1( )n n nf f x f−= + +

1 ,n nf x f+= +como desejado!

Vejamos que as raízes de 2 1x x= + são os números 1 5

2+α = e

1 5.

2−β =

Então, para todo n = 2, 3, … nós temos

1n

n nf f −α = α +e

1.n

n nf f −β = β +

Subtraindo as duas últimas equações temos que ( ),n nnfα − β = α −β e notando

que 5α −β = , nós encontramos a fórmula de Binet

.n n

nfα − β=α −β

Problema 1

Seja 1 5

.2+α = Determine todos os n∈� tais que 2n nα − α seja inteiro.

Solução:

Note que α é raiz do polinômio 2( ) 1,p x x x= − − com isso e usando a expressão

da propriedade (V) temos que:2 2 2

1 1( ) .nn n n nn f f n f n f− −α − α = α + − α = − α +

Uma vez que α é irracional, segue da igualdade acima que 2n nα − α só será

inteiro quando 2 0,nf n− = e nosso problema equivale a determinar todos os

n∈� tais que 2 .nf n= Para tanto, observe que:

1 2 3 4 5 6 71, 1, 2, 3, 5, 8, 13,f f f f f f f= = = = = = =

8 9 10 11 12 13 1421, 34, 55, 89, 144, 233, 377.f f f f f f f= = = = = = =

Assim, 2 2 2

12 13 1412 e 13 , 14 .f f f= > > Por outro lado, se 2

nf n> e2

1 ( 1) ,nf n+ > + então2 2 2

2 1 ( 1) ( 2) ,n n nf f f n n n+ += + > + + > + desde que 3,n >

Page 41: Eureka 2005

Sociedade Brasileira de Matemática

EUREKA! N°21, 2005

41

donde segue por indução que 2nf n> para 13.n > Assim, as únicas soluções são

n = 1 e n = 12.

Problema 2:Sejam n e k dois inteiros positivos quaisquer. Então entre duas potências

consecutivas kn e 1kn + não podemos ter mais que n números de Finonacci.Sugestão: use a propriedade (I)

Problema 3:

Seja nf a seqüência de Fibonacci 1 2 1 1( 1, 1, ).n n nf f f f f+ −= = = + Calcule a série

2 1 1

.n

n n n

f

f f

= − +⋅∑

Problema 4:

Ache a, se a e b são números inteiros tais que 2 1x x− − é um fator de17 16 1.ax bx+ +

Fortemente ligada à seqüência de Fibonacci, e tão interessante quanto, é a seqüênciade Lucas que é definida da seguinte maneira:

1 1n n nL f f− += +

1 2 31, 3, 4,...L L L= = =Obs: é fácil perceber que de acordo com a definição da seqüência de Lucas temos

que 0 2.L =Usando a fórmula de Binet temos que:

1 1 1 1

1 1

n n n n

n n nL f f− − + +

− +α −β α −β= + = +α −β α −β

1 1 1.n n = α + α −β + β α −β α β

Como 1 5

2

+α = e 1 5

,2

−β = então 1

5+ α =α

e 1

5.+ β = −β

Portanto,

.n nnL = α + β

Problema 5:

Prove que 2 .n n nf f L=

Page 42: Eureka 2005

Sociedade Brasileira de Matemática

EUREKA! N°21, 2005

42

Problema 6:

Sejam 0 12, 1,L L= = e 2 1 ,n n nL L L+ += + para 0,n ≥ a seqüência de Lucas.

Prove que, para todo 1,m ≥

12 21

,k m

m

k

L f +

==∏

onde nf é a seqüência de Fibonacci.

Problema 7:

Achar o termo geral np se 0 1p = e 31 5 3 ,n n np p p+ = − para 0.n ≥

Problema 8:Todo natural pode ser unicamente escrito como soma de números de Fibonaccidistintos, de índices não consecutivos e maiores que 1. (Teorema de Zeckendorff).

Problema 9:

Sejam a e b inteiros positivos tais que 93b é divisível por 19a e 93a é divisível por19 .b Prove que 14 8( ) nfa b ++ é divisível por ( ) nfab para todo n > 1.

Problema 10:Prove que nenhum número de Fibonacci é potência de 7.

Problema 11:

Sejam ( )nf a seqüência de Fibonacci e, para todo inteiro positivo n,

2 22 .n n nV f f += +

Prove que, para todo inteiro positivo n, ,nV 1 ,nV + 2nV + são lados de um triângulo de

área 1.

2

REFERÊNCIAS:[1] Ross Honsberger, Mathematical Gems III, MAA, 1985.[2] Hygino H. Domingues, Fundamentos da Aritmética, Atual, 1991.

Page 43: Eureka 2005

Sociedade Brasileira de Matemática

EUREKA! N°21, 2005

43

COMO É QUE FAZ?

PROBLEMA 6PROPOSTO POR RAFAEL ALVES DA SILVA (TERESINA - PI) da Olimpíada Ibero-americana deMatemática de 1999.Seja n um inteiro maior que 10 tal que cada um dos seus dígitos pertence ao

conjunto {1,3, 7,9}.S = Prove que n tem algum divisor primo maior ou igual a 11.

Solução: Como o último dígito de n é ímpar, n é ímpar.Como o último dígito de n não é 0 nem 5, n não é múltiplo de 5. Assim, se n nãotivesse nenhum fator primo maior ou igual a 11, os únicos possíveis fatores primosde n seriam 3 e 7. Mostraremos por indução que todo número natural cujos únicos

fatores primos são 3 e 7 é da forma n = 20k + r, onde k é natural e {1,3, 7,9}r∈ , eportanto o dígito das dezenas de n deve ser par, e logo não pode pertencer a

{1,3,7,9}S = (e assim, se todos os dígitos de n pertencem a S, devemos tern < 10).Para isso, note que 1 = 20 ⋅ 0 + 1, e ainda

n = 20k + 1 ⇒ 3n = 20 ⋅ (3k) + 3 e 7n = 20 ⋅ (7k) + 7n = 20k + 3 ⇒ 3n = 20 ⋅ (3k) + 9 e 7n = 20 ⋅ (7k + 1) + 1n = 20k + 7 ⇒ 3n = 20 ⋅ (3k + 1) + 1 e 7n = 20 ⋅ (7k + 2) + 9n = 20k + 9 ⇒ 3n = 20 ⋅ (3k + 1) + 7 e 7n = 20 ⋅ (7k + 3) + 3,e nossa afirmação está provada.

PROBLEMA 7

PROPOSTO POR RAFAEL ALVES DA SILVA (TERESINA - PI) da Olimpíada da Bielorrúsia de2000.

Seja {1, 2,3,..., 40}.M = Ache o menor inteiro n para o qual é possível particionarM em M subconjuntos disjuntos tais que, sempre que a, b e c (não necessariamentedistintos) pertencem ao mesmo subconjunto, então a ≠ b + c.

Solução: Mostraremos que n = 4. Vamos ver como construir um exemplo de umapartição de n em 4 conjuntos de acordo com o enunciado, a partir de exemplosmenores: temos a partição {1} = {1}, com n = 1, para M = {1}; a partição{1, 2, 3, 4} = {1, 4} ∪ {2, 3}, com n = 2, para M = {1, 2, 3, 4}; a partição {1, 2, 3,4, 5, 6, 7, 8, 9, 10, 11, 12, 13} = {1, 4, 10, 13} ∪ {2, 3, 11, 12} ∪ {5, 6, 7, 8, 9} comn = 3, para M = {1, 2,…,13}; a idéia é, dada uma partição {1, 2,…, k} =

Page 44: Eureka 2005

Sociedade Brasileira de Matemática

EUREKA! N°21, 2005

44

1 2 ... nA A A∪ ∪ ∪ , obter uma partição 1 2 1{1, 2, ...,3 1} ... ,k kk B B B B ++ = ∪ ∪ ∪ ∪tomando 1 { 1, 2,...,2 1}kB k k k+ = + + + e, para , ( (2 1)).i i ii k B A A k≤ = ∪ + + Assim,

obtemos, para {1,2,...,40},M = {1,4,10,13,28,31,37,40} {2,3,11,12,29,30,38,39}M = ∪{5,6,7,8,9,32,33,34,35,36} {14,15,16,17,18,19,20,21,22,23,24,25,26,27},∪ ∪ e é fácil

verificar que esta partição satisfaz a condição do enunciado.Para mostrar que não é possível obter uma tal partição de M em apenas 3subconjuntos, suponha que .M A B C= ∪ ∪ Como M tem 40 elementos, algum dosconjuntos A, B ou C tem pelo menos 14 elementos. Digamos que A contenha os 14

elementos 1 2 14...x x x< < < . Os 13 elementos 14 1 14 2 14 13, ,..., x x x x x x− − − devem

pertencer a ,B C∪ pois, se 14 ,jx x y A− = ∈ então 14 ,jx x y= + com , jy x , 14 ,x A∈contradição. Assim, B ou C contém pelo menos 7 desses 13 elementos, digamos

1 2 7... ,y y y< < < onde 14 ,ii jy x x= − onde 7 6 1... .j j j< < < Suponhamos que esses

elementos pertencem a B. Temos então 3 2y y C− ∈ e 3 1 ,y y C− ∈ pois, se

3 ,jy y z B− = ∈ para algum 3{1,2}, ,jj y y z∈ = + com 3 , , ,jy y z B∈ absurdo,

e, 314 14( ) ( )

ij jx x x x− − − como 3 33 14 14( ) ( )

i ii j j j jy y x x x x x x− = − − − = − , se

3 ,iy y z A− = ∈ para algum {1,2},i∈ teríamos 3

,ij jx x z= + com

3,

ij jx x , z A∈ ,

absurdo. Vamos agora considerar o elemento 2 1 0.y y− > Como 1y e 2y pertencem

a B, 2 1y y− não pode pertencem a B. Como 2 1 3 1 3 2( ) ( ),y y y y y y− = − − − com

3 2 3 1( ), ( ) , y y y y C− − ∈ 2 1y y− não pode pertencer a C. Finalmente, como

2 1 1 22 1 14 14( ) ( ) ,j j j jy y x x x x x x− = − − − = − com 1j

x A∈ e 2 2 1, jx A y y∈ − não pode

pertencer a A, absurdo, pois 2 1 .y y M A B C− ∈ = ∪ ∪

Publicamos a seguir as soluções dos problemas 121 e 195 da seção "Olimpíadas ao redor domundo", por sugestão de Bruno, da Espanha.

121. (Rússia-2001) Os valores da função quadrática ( ) baxxxf ++= 2 para doisinteiros consecutivos são os quadrados de dois inteiros também consecutivos.Mostre que os valores da função quadrática são quadrados perfeitos para todos osinteiros coincide com o conjunto dos valores de g para os inteiros.Solução: Suponha 2( )f m k= e 2( 1) ( 1) ,f m k+ = + com m e k inteiros. Seja

( ) ( ).g x f x m= + O conjunto dos valores de f para os inteiros.

Page 45: Eureka 2005

Sociedade Brasileira de Matemática

EUREKA! N°21, 2005

45

Temos 2( )g x x cx d= + + para certos valores de c e d. Temos 2(0) ( )d g f m k= = =e 21 (1) ( 1) ( 1) ,c d g f m k+ + = = + = + donde 2d k= e 2 2( 1) 1 2 ,c k k k= + − − = ou

seja, 2 2 2( ) 2 ( ) ,g x x kx k x k= + + = + e logo g(x) é um quadrado perfeito para todo xinteiro.

195.(Eslovênia-2002) Sejam M o ponto médio da base AB do trapézio ABCD; E umponto interior ao segmento AC tal que BC e ME intersectam-se em F; G o ponto deinterseção de FD e AB; H o ponto de interseção de DE e AB. Mostre que M é oponto médio do segmento GH.

Solução: Podemos supor sem perda de generalidade, aplicando uma transformaçãoafim, que A = (0, 0), B = (1, 0), C = (1, 1), e logo D = (a, 1), para algum a < 1. Então

1,0

2M =

e E = (t, t), para algum (0,1).t∈ Como F pertence a BC, F = (1, y) para

algum y, e, supondo 1

,2

t ≠ como F pertence a ME, ( ) ( )1 12 2y t t= − , donde

2 1

ty

t=−

(caso 1

,2

t = BC e ME são paralelas e não se intersectam, logo não

existiria o ponto F). Como H pertence a AB, H = (x, 0) para um certo x. Como H

pertence a DE, 1

,t

t x a x=

− −donde (1 ) (1 ),t x t a− = − ou seja,

(1 ).

1

t ax

t

−=−

Finalmente, como G pertence a AB, G = (z, 0) para um certo z, e como G pertence a

FD, 1 (2 1)

1

t t

a z z

−=− −

, donde 1 (2 1)

,2 1 2 1

t t atz

t t

− − −⋅ =− −

ou seja, (2 1)

.1

at tz

t

− −=−

Assim, 1 1 1

,2 2 1 2

x z t

t

+ −= ⋅ =−

e logo ,2

G HM

+= ou seja, M é o ponto médio do

segmento GH.

Page 46: Eureka 2005

Sociedade Brasileira de Matemática

EUREKA! N°21, 2005

46

SOLUÇÕES DE PROBLEMAS PROPOSTOS� Publicamos aqui algumas das respostas enviadas por nossos leitores.

90) Prove que, para todo inteiro positivo n e para todo inteiro não nulo a, o

polinômio 1 2 ... 1n n nx ax ax ax− −+ + + + − é irredutível, i.e., não pode ser escrito comoo produto de dois polinômios não constantes com coeficientes inteiros.

SOLUÇÃO (baseada em um argumento de Artur Avila):

Seja 1 2( ) ... 1.n n naP x x ax ax ax− −= + + + + − Como

1( ) ,n

a aP x x Px− =−

( )aP x é

irredutível se e somente se ( )aP x− é irredutível. Assim, podemos supor sem perda de

generalidade que a é um inteiro positivo. Nesse caso, (0) 1aP = − e

(1) ( 1) 0, 2.aP n a n= − > ∀ ≥É claro que para n = 1 o polinômio ( ) 1aP x x= − é irredutível.

Para 2n≥ , deve existir ( )0,1∈α com ( ) 0.aP =αSeja 1 1( ) ( ) ( 1) ( 1)( 1) ( 1) ( 1) ( 1) 1.n n n n

a aQ x P x x x x ax x x a x a x− += ⋅ − = − − + − = + − − + +Como α é raiz de ( ) 0.aP =αSeja 1 1( ) ( ) ( 1) ( 1)( 1) ( 1) ( 1) ( 1) 1.n n n n

a aQ x P x x x x ax x x a x a x− += ⋅ − = − − + − = + − − + +Como α é raiz de ( ), aP x α também é raiz de ( ).aQ x Seja então

1 2 2 3 2( ) 1( ) ( 1) ( 1) ( 1) ... ( 1) .n n n n naQ x

R x x a x a x a x a xx

− − − −= = + + − + + − + + − + + + − −−

α α α α α α αα α

Temos (1) (1) (1 ) 0,R Q= − =α α e, se ≠β α é raiz de ( ),aP x β é raiz de ( )aQ x e de

( ).R x Mostraremos que 1>β para todo ∈�β , ≠β α tal que β é raiz de ( ).aP x

Como β é raiz de ( ),R x temos 1 2( 1) ( 1) ...n n na a− −+ + − + + − + +β α β α α β2 1

( 1) .n a−+ + − =α α βα

Se 1,≤β 1 2 2( 1) ( 1) ... ( 1)n n n na a a− − −+ + − + + − + + + −β α β α α β α α β1 2 1( 1) ... ( 1) 1 ( 1)1 ...

n n n n na a a− − −≤ + + − + + + − ≤ + + − + +β α β α α β α

Page 47: Eureka 2005

Sociedade Brasileira de Matemática

EUREKA! N°21, 2005

47

2 1 1 1 ( 1) 1 (1) ,n a R−+ + − ⋅ = + = =α α

α α α valendo a igualdade se, e somente se,

1,= =β β mas (1) 0,aP ≠ donde 1,≠β absurdo (na verdade, mostramos que

todas as raizes de ( )R x têm módulo maior ou igual a 1; em particular, α é raiz

simples de ( ),aP x caso contrário (0,1)∈α seria raiz de ( )).R x

Suponha agora que ( ) ( ) ( ),aP x f x g x= ⋅ onde f e g são polinômios não constantes

com coeficientes inteiros. Como o coeficiente constante de ( )aP x é –1, os

coeficientes constantes de f(x) e de g(x) pertencem a {–1, 1} (e podemos supor semperda de generalidade que seus coeficientes líderes são iguais a 1). Assim, o produtodas raízes de g(x) (e de f(x)) pertencem a {–1, 1}. Por outro lado, podemos suporsem perda de generalidade que α é raiz de ( ).f x Como todas as raízes de g(x) são

raízes de ( )aP x distintas de α , elas têm todas módulo maior que 1, e logo seu

produto não pode pertencer a {–1, 1}, absurdo.

95) "Resta-Um" é um jogo de tabuleiro na qual as peças ocupam um tabuleiroformando parte de um reticulado retangular (na verdade, existem variações emtabuleiros de reticulado triangular). O único movimento permitido consisteem tomar duas peças em casas adjacentes vizinhas a uma casa vazia, e fazer apeça mais distante da casa vazia pular sobre a outra peça, ocupando a casavazia. A peça pulada é retirada.

(esse movimento pode ser feito para a direita, para a esquerda, para cima oupara baixo).Agora imagine um tabuleiro que é um reticulado retangular infinito e uma retaque contém uma linha do reticulado, dividindo-o em dois lados. Todas as casasde um dos lados da linha estão vazias e cada casa do outro lado da linha podeou não ter uma peça.Quantas peças, no mínimo, precisamos para chegar a uma casa do lado vaziodo tabuleiro, a uma distância n da linha ? Abaixo indicamos uma casa adistância n, para n = 1,2,3,4,5.

Page 48: Eureka 2005

Sociedade Brasileira de Matemática

EUREKA! N°21, 2005

48

54321

… …… …… …… … … … … … …

SOLUÇÃO DE BERNARDO FREITAS PAULO DA COSTA (RIO DE JANEIRO – RJ)Vamos botar eixos no quadriculado, de acordo com a posição:

Y

X

"Fronteira"

Assim, a casa marcada com � é (0, 0), sua vizinha à esquerda é (1, 0) e acima é(0, 1). Supomos, sem perda de generalidade, que a peça final termina na casa (0, n)para n = 1, 2, 3, 4, 5. Usando isto, vemos que o movimento deveria "tender para ocentro e para cima".Com esta idéia, vamos associar energias às peças nos quadrados do tabuleiro,através da seguinte função:

( , ) abE a b = Φϕ onde 2

2

1, 1

1, 0 1

= + >Φ +Φ = < Φ <

ϕ ϕ ϕ

Page 49: Eureka 2005

Sociedade Brasileira de Matemática

EUREKA! N°21, 2005

49

Como as equações que os definem são simétricas, temos 1Φ =ϕ

Só para conferir:

1 51

1 5 1 522 21 5

02

+ >± += ⇒ =

− <ϕ ϕ

5 1(0,1)

1 5 5 122 21 5

02

− ∈− ± −Φ = ⇒Φ =

− − <

Assim, se fizermos um movimento

a – 2 a – 1 a a – 2 a

, a ≤ 0ou um movimento

a + 2 a + 1 a a + 2 a

, a ≥ 0na direção do centro, estaremos trocando uma energia 2 1a ab b+ +Φ + Φϕ ϕ por

abΦϕ , que são iguais por construção.Já se fizermos

a + 2 a + 1 a a + 2 a

, a > 0ou o contrário (com a < 0)

trocaremos energia com perdas, pois 2a a≤ + (para a > 0; se

a < 0 é 2a a≤ − ).

Para movimentos verticais, vemos que

Page 50: Eureka 2005

Sociedade Brasileira de Matemática

EUREKA! N°21, 2005

50

b + 2

b

b + 1

conserva energia ( )1 2a a ab b b+ +Φ + Φ = Φϕ ϕ ϕ e que

b + 2

b

b + 1

perde (pois 2b b+ >ϕ ϕ e 1 0b+ >ϕ ).Assim, a energia das peças no tabuleiro não aumenta.Vejamos a energia máxima do tabuleiro:ET = E (coluna central) + E (quadrantes negativos) = E(coluna central) +2E(1 quadrante negativo) =

00

1

2b

b a

E∞

=−∞ == Φ +∑ ∑ϕ (coluna em a)

0 0

1

2 ab b

b a b

=−∞ = =−∞

= + Φ ∑ ∑ ∑ϕ ϕ

0 1 0

2 ab b

b a b

+∞ ∞ ∞− −

= = =

= + Φ ∑ ∑ ∑ϕ ϕ

1

1 12

1 11 1

a

a

=

= + Φ − −

∑ϕ ϕ

Mas 22 2

1 1 1 11 1 1= + ⇒ = + ⇒ − =ϕ ϕ

ϕ ϕ ϕ ϕ

12 2

1 12

1 1a

a

== + Φ∑ϕ ϕ

2 2

1

21 2 1

1a

a

=

Φ = + Φ = + −Φ ∑ϕ ϕ

Page 51: Eureka 2005

Sociedade Brasileira de Matemática

EUREKA! N°21, 2005

51

Como 2 1,Φ +Φ = temos 21−Φ = Φ , e assim

( )2 2 2 22

2 21 1 (1 2 ) (1 )TEΦ = + = + = + = + + ΦΦ

ϕ ϕ ϕ ϕ ϕ ϕ ϕ

( )2 2 2 3 5= + = =ϕ ϕ ϕ ϕ ϕ ϕ

Ora, como a energia total do tabuleiro é 5ϕ , se tivermos um número finito demovimentos (para poder chegar em (0, 5) de verdade) teremos usado um conjunto

finito de peças e portanto de energia estritamente menor do que 5ϕ . Desta forma, éimpossível chegar a esta casa com movimentos "resta-um".

Decomponhamos nϕ para n = 1, 2, 3, 4 para sabermos o número de peças

necessárias. Vale lembrar as relações:

2 1

1 2

11 1

a a a

a a a

+ +

+ +

= +Φ = Φ +Φ = +Φ = +

ϕ ϕ ϕ

ϕϕ

( 2 1= +ϕ ϕ )

1Φ = ϕ1 1 2 peças:= +Φ→ϕ

"Fronteira"

OK

Altura 2: ( )2 1 1 1= + = + +Φϕ ϕ só temos 1 peça com energia 1

( )21= + Φ +Φ +Φ só temos 3 peças com energia Φ

só temos 5 peças com energia 2Φ4 peças: (em geral, para h ≥ 0, só temos 2h + 1 peças com energia hΦ )

"Fronteira"

Page 52: Eureka 2005

Sociedade Brasileira de Matemática

EUREKA! N°21, 2005

52

"Fronteira"

"Fronteira"

Altura 3: 3 2 2 1 3 2 1 2 (2)= + = + = + Φ = + Φ +ϕ ϕ ϕ ϕ2 21 2 (2 2 ) 1 3 2 ( )= + Φ + Φ+ Φ = + Φ + Φ + Φ

2 31 3 3 8 peças:= + Φ + Φ +Φ →

"Fronteira"

vai à (0, 2)

"Fronteira"

"Fronteira"

OK

Page 53: Eureka 2005

Sociedade Brasileira de Matemática

EUREKA! N°21, 2005

53

Vamos ao último caso (altura 4):4 3 2 22 3 2 5 3= + = + = + = + Φϕ ϕ ϕ ϕ ϕ ϕ

21 3 (4) 1 3 (4 4 )= + Φ + = + Φ + Φ + Φ

2 2 31 3 4 (4 4 )= + Φ + Φ + Φ + Φ

2 3 2 2 3 41 3 5 4 (3 ) 1 3 5 7 3= + Φ + Φ + Φ + Φ = + Φ + Φ + Φ + ΦTeoricamente, isto daria 1 + 3 + 5 +7 + 3 = 19 peças para chegar a altura 4.Entretanto, isto é impossível. Vejamos:Pintemos o tabuleiro com 3 cores em diagonal

B A C B AC B A C B AA C B A C B AB A C B A C B AC B A C B A C B A

C B A C B A C BC B A C B A C

C B A C B AC

"Fronteira"

Toda vez que realizarmos um movimento, tiramos uma peça de duas coresadjacentes e a terceira cor recebe uma peça a mais.Assim, invertemos a paridade dos 3 simultaneamente. Portanto, a paridade relativase mantém.As casas dentro da região indicada estarão todas ocupadas (suas energias somam

2 31 3 5 7+ Φ+ Φ + Φ ).A soma das cores é 5A + 7B + 4CSe a soma final é 1A + 0B + 0C, temos que a paridade de B e C é a mesma, e oposta

à paridade de A, logo das 43Φ que restam, temos uma quantidade par da cor A, umaquantidade ímpar de cor B e uma quantidade para da cor C.Assim, temos as seguintes possibilidades:2A + 1B + 0C0A + 3B + 0C0A + 1B + 2C

Mas só há 1 casa B valendo 4Φ logo a possibilidade OA + 3B + 0C está excluída.

Assim, concluímos que há 1 peça numa casa B, a única do nível 4Φ .Por simetria (ou seja, pintando o tabuleiro na outra direção : /// ),

Page 54: Eureka 2005

Sociedade Brasileira de Matemática

EUREKA! N°21, 2005

54

temos que deve haver também uma peça na casa simétrica a essa casa B do nível4Φ em relação à coluna central, a qual é uma casa C, e logo a solução é OA + 1B +

2C.Assim, temos a seguinte configuração:

X X? ?

X X?

"Fronteira"

As casas marcadas com X são A ou simétricas a casas A, e logo estão proibidas.As casas com • estão forçosamente ocupadas, e das casas com ?, uma, e apenas uma,será ocupada (para fazer 19).Isolemos a coluna central; sobra (pelo menos) um bloco consistindo apenas dasseguintes peças:

(ou do outro lado,por simetria éigual).

Ora, a peça � , para ser aproveitada, deve tomar uma peça que está a sua direita e irpara a coluna – 1.Entretanto, para terminarmos com peças apenas na coluna zero, as duas indicadaspelas setas devem tomar as peças que estão à direita (pois senão não conseguiríamosmandar todas as peças desse bloco para a coluna central usando apenas essas peças;se usássemos outras peças para isso perderíamos energia). Nesse caso, as peças quesobram do lado esquerdo ficam todas na altura 0, e a peça que estava originalmentena coluna 0 e na coluna – 1 não conseguiría chegar na coluna central sem que hajaperda de energia. Assim, não conseguimos (agora por impedimento de movimento,não de energia) chegar em (0, 4) com 19 peças.

Mas é possível com 20 (faço 3 4 5Φ =Φ +Φ ):

Page 55: Eureka 2005

Sociedade Brasileira de Matemática

EUREKA! N°21, 2005

55

"Fronteira" "Fronteira"

"Fronteira" "Fronteira"

"Fronteira" "Fronteira"

"Fronteira"

OK!!

96) No quadrilátero ABCD os ângulos A, C e D medem 100° e o ângulo ACB mede

40°. Demonstre que 2( ) .BC DA BC AB DA⋅ = + −

Page 56: Eureka 2005

Sociedade Brasileira de Matemática

EUREKA! N°21, 2005

56

SOLUÇÃO DE HEYTOR BRUNO E MARLON JÚNIOR (FORTALEZA – CE)A figura ilustra o problema:

A

D

C

B

80° 20°

60°

100°

60° 40°

ACD∆ (Lei dos Senos): 100

60AC sen

DA sen

°=°

ACB∆ (Lei dos Senos): 80

60BC sen

AC sen

°=°

2AC BCAC BC DA

DA AC⇒ = ⇒ = ⋅

Assim,

2 2 2( ) ( )BC DA BC AB DA AC BC AB DA⋅ = + − ⇔ = + − . Basta provar entãoque AC = BC + AB – DA.

ACD∆ (Lei dos Senos): 60

100AC sen

DAsen

°=°

ACB∆ (Lei dos Senos): 80

60AC sen

BCsen

°=°

e 40

60AC sen

ABsen

°=°

Daí, AC + DA = BC + AB ⇔ 60 80 40

100 sen60 sen60

AC sen AC sen AC senAC

sen

° ° °⇔ + = + ⇔° ° °

Page 57: Eureka 2005

Sociedade Brasileira de Matemática

EUREKA! N°21, 2005

57

100 60 80 40.

100 sen60sen sen sen sen

sen

°+ ° °+ °⇔ =° °

Usando o fato de que

2 cos ,2 2

p q p qsen p sen q sen

+ − + =

A equação acima é verdadeira se e somente se

2 80 cos 20 2 60 cos 20,

100 60sen sen

sen sen

° ° ° °=° °

o que é obviamente verdade.

99) Num triângulo, a razão entre os raios das circunferências circunscrita e inscrita é5

.2

Os lados do triângulo estão em progressão aritmética e sua área é

numericamente igual ao seu perímetro. Determine os lados do triângulo.

SOLUÇÃO DE KELLEM CORRÊA SANTOS (RIO DE JANEIRO – RJ)

A

B a + q

R

a

a – q

C

r

52 5

2R

R rr= ⇒ =

Perimetro de 3

2 ( ) ( ) 32a

ABC p a a q a q a p⇒ = + + + − = ⇒ =

pr = ( )( )

( )4

a a q a qA ABC

R

+ −=

Page 58: Eureka 2005

Sociedade Brasileira de Matemática

EUREKA! N°21, 2005

58

2 2 2 2 2 22 2 23 ( )

3 152 4 2 5

a a a q a q a qr r r a q

R R r

− − −= ⇒ = = ⇒ = − (1)

Como a área é numericamente igual ao perímetro, temos:2 2pr p r= ⇒ =

Por outro lado,

3( ) ( )( )( )

2 2 2 2

a a a aA ABC p p a p a q p a q q q

= − − − − + = ⋅ ⋅ − + = 2

213

2 4a

a q

= −

De (1), temos:2 2 2 2 2 2 2 2 2 215 15 2 60 60r a q a q a q q a= − ⇒ ⋅ = − ⇒ = − ⇒ = −

Logo 2 2

2 3( ) 3 60 3 60

2 4 2 4

a a a aA ABC a

= − + = −

Também podemos escrever:

3( ) 2 3

2a

A ABC pr a= = ⋅ =

Igualando:23

3 3 602 4

a aa

= −

2 2 2

2

22

3 36 3 36 3 60 60 12 60

4 3 4 4

348 16 4 8, pois 0.

4

a a a

aa a a

= − ⇒ = − ⇒ = − ⇒

⇒ = ⇒ = ⋅ ⇒ = >

De (1):2 2 2 215 60 64 2r a q q q= − ⇒ = − ⇒ = ±

Logo, os lados do triângulo ABC são 6, 8 e 10.

101) a) Sejam , ,i i ia b c reais positivos, para 1 3.i≤ ≤Prove que 3 3 3 3 3 3 3 3 3 3

1 2 3 1 2 3 1 2 3 1 1 1 2 2 2 3 3 3( )( )( ) ( ) .a a a b b b c c c a bc a b c a b c+ + + + + + ≥ + +b) Sejam , , , , ,a b c x y z reais positivos. Prove que

Page 59: Eureka 2005

Sociedade Brasileira de Matemática

EUREKA! N°21, 2005

59

3 3 3 3

2 2 2 2

( ).

( )

a b c a b c

x y z x y z

+ ++ + ≥+ +

SOLUÇÃO DE MARCELO RIBEIRO DE SOUZA e WALLACE MARTINS (RIO DE JANEIRO – RJ)a) Como esta é uma desigualdade homogênea, podemos supor

3 3 31 2 3( ) 1a a a+ + =3 3 31 2 3( ) 1b b b+ + =3 3 31 2 3( ) 1c c c+ + =

Com isto nossa desigualdade passa a ser:3

1 1 1 2 2 2 3 3 31 ( )a bc a b c a b c≥ + + . Isto é imediato de MA – MG:3 3 31 1 1

1 1 1

( )

3a b c

a b c+ + ≥

3 3 32 2 2

2 2 2

( )

3a b c

a b c+ + ≥

3 3 33 3 3

3 3 3

( )

3a b c

a b c+ +

Somando tudo e elevando ao cubo ficamos com:

( )31 1 1 2 2 2 3 3 31 a bc a b c a b c≥ + + cqd.

b) Este item decorre diretamente do anterior, tendo em vista que

33 33 3 3 3 3 31 1 1 1 1 1

3 3 3 3 3 32 2 23 3 3

a b cx y z x y z

x y z

+ + + + + + ≥ 3

1 1 1 1 1 1

3 3 3 3 3 32 2 2

3 3 3

a b cx x y y z z

x y z

⋅ ⋅ + ⋅ ⋅ + ⋅ ⋅ Portanto segue que

( )( )

33 3 3

22 2 2.

a b ca b c

x y z x y z

+ + + + ≥

+ +

Page 60: Eureka 2005

Sociedade Brasileira de Matemática

EUREKA! N°21, 2005

60

PROBLEMAS PROPOSTOS

� Convidamos o leitor a enviar soluções dos problemas propostos e sugestões de novosproblemas para os próximos números.

102) Você recebe x metros de arame para cercar um terreno na forma de umtriângulo pitagórico (os lados são números inteiros), com a condição de que amedida do cateto menor seja 24 metros. Qual deverá ser a medida do catetomaior e o comprimento do arame, a fim de que a área seja:a) máxima?b) mínima?

103) Sejam A e B matrizes 2 × 2 com elementos inteiros.Sabendo que A, A + B, A + 2B, A + 3B e A + 4B são invertíveis e que oselementos das respectivas inversas também são todos inteiros, mostre queA + 5B também é invertível e que os elementos da sua inversa também sãointeiros.

104) ABC é um triângulo. Mostre que existe um único ponto P de modo que:

( ) ( ) ( ) ( ) ( ) ( ) ( ) ( ) ( )2 2 2 2 2 2 2 2 2PA PB AB PB PC BC PC PA CA+ + = + + = + +

105) O baricentro do triângulo ABC é G. Denotamos por , ,a b cg g g as distâncias

desde G aos lados a, b e c respectivamente.Seja r o rádio da circunferência inscrita. Prove que:

a) 2 2 2

, , 3 3 3a b c

r r rg g g≥ ≥ ≥

b) 3a b cg g g

r

+ + ≥

106) Os polinômios 0 1 2( , , ), ( , , ), ( , , ),...P x y z P x y z P x y z são definidos por

0 ( , , ) 1P x y z = e 21( , , ) ( )( ) ( , , 1) ( , , ),m m mP x y z x z y z P x y z z P x y z+ = + + + −

0.m∀ ≥ Mostre que os polinômios ( , , ),mP x y z m∈� são simétricos em x, y,

z, i.e., ( , , ) ( , , ) ( , , ) ( , , ) ( , , ) ( , , ),m m m m m mP x y z P x z y P y x z P y z x P z x y P z y x= = = = =para quaisquer x, y, z.

Page 61: Eureka 2005

Sociedade Brasileira de Matemática

EUREKA! N°21, 2005

61

107) a) Dado um triângulo qualquer, prove que existe um círculo que passa pelospontos médios dos seus lados, pelos pés das suas alturas e pelos pontos médiosdos segmentos que unem o ortocentro aos vértices do triângulo (o chamado"círculo dos nove pontos").

b) Prove que, se X é o centro do círculo dos nove pontos de um triângulo, Ho seu ortocentro, O seu circuncentro e G seu baricentro, então

3 1.

2 2OX OG OH= ⋅ = ⋅���� ���� ����

Problema 102 proposto por Sebastião Vieira do Nascimento (Campina Grande – PB); Problema103 proposto por Carlos A. Gomes (Natal – RN); Problemas 104 e 106 propostos por WílsonCarlos da Silva Ramos (Belém – PA); Problema 105 selecionado da XXXV OlimpíadaMatemática Espanhola, fase nacional, 1999 e enviado por Bruno Salgueiro Fanego (Espanha).

Agradecemos também o envio das soluções e a colaboração de:

Carlos Alberto da Silva Victor Nilópolis – RJDaniel Lopes Alves de Medeiros Fortaleza – CEGabriel Ponce Por e-mailGeorges Cobiniano Sousa de Melo João Pessoa – PBGlauber Moreno Barbosa Rio de Janeiro – RJJônatas de Souza Júnior Recife – PERafael Silva Teresina – PIRaphael Constant da Costa Rio de Janeiro – RJ

Seguimos aguardando o envio de soluções dos problemas propostos Nos. 89, 97, 98 e 100.

Page 62: Eureka 2005

Sociedade Brasileira de Matemática

EUREKA! N°21, 2005

62

COORDENADORES REGIONAIS

Alberto Hassen Raad (UFJF) Juiz de Fora – MGAmérico López Gálvez (USP) Ribeirão Preto – SPAmarísio da Silva Araújo (UFV) Viçosa – MGAna Paula Bernardi da Silva (Universidade Católica de Brasília) Brasília – DFAntonio Carlos Nogueira (UFU) Uberlândia – MGAli Tahzibi (USP) São Carlos – SPBenedito Tadeu Vasconcelos Freire (UFRN) Natal – RNCarlos Frederico Borges Palmeira (PUC-Rio) Rio de Janeiro – RJClaus Haetinger (UNIVATES) Lajeado – RSCleonor Crescêncio das Neves (UTAM) Manaus – AMCláudio de Lima Vidal (UNESP) S.J. do Rio Preto – SPEdson Roberto Abe (Colégio Objetivo de Campinas) Campinas – SPÉlio Mega (Colégio Etapa) São Paulo – SPÉder Luiz Pereira de Andrade (UNESPAR/FECILCAM) Campo Mourão – PREudes Antonio da Costa (Univ. do Tocantins) Arraias – TOFlorêncio Ferreira Guimarães Filho (UFES) Vitória – ESIvanilde Fernandes Saad (UC. Dom Bosco) Campo Grande– MSJacqueline Fabiola Rojas Arancibia (UFPB) João Pessoa – PBJanice T. Reichert (UNOCHAPECÓ) Chapecó – SCJoão Benício de Melo Neto (UFPI) Teresina – PIJoão Francisco Melo Libonati (Grupo Educacional Ideal) Belém – PAJosé Carlos dos Santos Rodrigues (Unespar) Campo Mourão – PRJosé Cloves Saraiva (UFMA) São Luis – MAJosé Gaspar Ruas Filho (ICMC-USP) São Carlos – SPJosé Luiz Rosas Pinho (UFSC) Florianópolis – SCJosé Vieira Alves (UFPB) Campina Grande – PBKrerley Oliveira (UFAL) Maceió – ALLicio Hernandes Bezerra (UFSC) Florianópolis – SCLuzinalva Miranda de Amorim (UFBA) Salvador – BAMário Rocha Retamoso (UFRG) Rio Grande – RSMarcelo Rufino de Oliveira (Grupo Educacional Ideal) Belém – PAMarcelo Mendes (Colégio Farias Brito, Pré-vestibular) Fortaleza – CEMarilane de Fraga Sant'Ana FACOS Osório – RSPablo Rodrigo Ganassim (Liceu Terras do Engenho) Piracicaba – SPRamón Mendoza (UFPE) Recife – PERaúl Cintra de Negreiros Ribeiro (Colégio Anglo) Atibaia – SPRonaldo Alves Garcia (UFGO) Goiânia – GOReginaldo de Lima Pereira (Escola Técnica Federal de Roraima) Boa Vista – RRReinaldo Gen Ichiro Arakaki (UNIVAP) SJ dos Campos – SPRicardo Amorim (Centro Educacional Logos) Nova Iguaçu – RJSérgio Cláudio Ramos (IM-UFRGS) Porto Alegre – RSSeme Guevara Neto (UFMG) Belo Horizonte – MGTadeu Ferreira Gomes (UEBA) Juazeiro – BATomás Menéndez Rodrigues (U. Federal de Rondônia) Porto Velho – ROValdenberg Araújo da Silva (U. Federal de Sergipe) São Cristovão – SEValdeni Soliani Franco (U. Estadual de Maringá) Maringá – PRVânia Cristina Silva Rodrigues (U. Metodista de SP) S.B. do Campo – SPWagner Pereira Lopes (CEFET – GO) Jataí – GO

Page 63: Eureka 2005

CONTEÚDO

AOS LEITORES 2

XXVI OLIMPÍADA BRASILEIRA DE MATEMÁTICA 3Problemas e Soluções da Primeira Fase

XXVI OLIMPÍADA BRASILEIRA DE MATEMÁTICA 17Problemas e Soluções da Segunda Fase

XXVI OLIMPÍADA BRASILEIRA DE MATEMÁTICA 29Problemas e Soluções da Terceira Fase

XXVI OLIMPÍADA BRASILEIRA DE MATEMÁTICA 49Problemas e Soluções da Primeira Fase - Nível Universitário

XXVI OLIMPÍADA BRASILEIRA DE MATEMÁTICA 55Problemas e Soluções da Segunda Fase - Nível Universitário

XXVI OLIMPÍADA BRASILEIRA DE MATEMÁTICA 64Premiados

AGENDA OLÍMPICA 68

COORDENADORES REGIONAIS 69

Page 64: Eureka 2005

Sociedade Brasileira de Matemática

EUREKA! N°22, 2005

2

AOS LEITORES

O Programa Nacional de Olimpíadas de Matemática tem crescido muito nos últimosanos, contando, atualmente, com a adesão ao Programa de mais de 5.000 escolas públicase privadas de todo o Brasil, o que implica em uma participação na Olimpíada Brasileira deMatemática de cerca de 250.000 jovens estudantes e seus professores. Além disso, oPrograma Nacional de Olimpíadas de Matemática conta com a colaboração de professoresdo ensino básico de todo o Brasil e de professores universitários de mais de 80 instituiçõesde ensino superior. Eles participam de todas as atividades da Olimpíada Brasileira deMatemática, em atividades de coordenação, divulgação, treinamento de alunos,aperfeiçoamento de professores e aplicação das distintas fases da Olimpíada Brasileira deMatemática.

Em relação à promoção do ensino da Matemática em nível regional, temosalcançado resultados extremamente positivos: através do apoio a 22 Olimpíadas Regionaisconseguimos atingir um universo de cerca de 150.000 estudantes e seus professores, osquais são desafiados à resolução de problemas que estimulam o raciocínio e a criatividade.No que se refere à participação em competições internacionais, o Programa Nacional deOlimpíadas também tem muito a comemorar. Em 2005 os resultados são excepcionais:Excelente resultado na Olimpíada de Matemática do Cone Sul (2 medalhas de Ouro, 2medalhas de Prata); o primeiro estudante Latino-Americano premiado com Medalha de OuroEspecial (Grand First Prize) e duas outras medalhas de Ouro na Olimpíada Internacional paraEstudantes Universitários (IMC), mais uma Medalha de Ouro na Olimpíada Internacional deMatemática (IMO) e novamente quatro medalhas de ouro na Olimpíada Ibero-americana deMatemática.

Todos estes resultados nacionais e internacionais demonstram que, além deinfluenciar positivamente o ensino da Matemática em instituições de ensino fundamental,médio e superior de todo o país, conseguimos detectar jovens muito talentosos que sãoestimulados a seguir uma carreira científica, o que é fundamental para o crescimento daCiência e Tecnologia no Brasil.

Os editores

Page 65: Eureka 2005

Sociedade Brasileira de Matemática

EUREKA! N°22, 2005

3

XXVI OLIMPÍADA BRASILEIRA DE MATEMÁTICAProblemas e Soluções da Primeira Fase

PROBLEMAS – NÍVEL 1

1. Calcule o valor de 1997 + 2004 + 2996 + 4003.A) 10000 B) 11000 C) 10900 D) 12000 E) 13000

2. Qual dos números a seguir é ímpar?A) 7 × 8 B) 37 – 23 C) 9 × 36 D) 144 : 36 E) 17 × 61

3. Quanto é 26 + 26 + 26 + 26 – 44?A) 0 B) 2 C) 4 D) 42 E) 44

4. O 20% de 40 é igual aA) 5 B) 8 C) 10 D) 12 E) 20

5. Simplificando a fração 2004 2004

2004 2004 2004

++ +

, obtemos:

A) 2004 B) 113

355C)

1

2004D)

2

3E)

2

7

6. Os alunos de uma escola participaram de uma excursão, para a qual dois ônibusforam contratados. Quando os ônibus chegaram, 57 alunos entraram no primeiroônibus e apenas 31 no segundo. Quantos alunos devem passar do primeiro para osegundo ônibus para que a mesma quantidade de alunos seja transportada nosdois ônibus?A) 8 B) 13 C) 16 D) 26 E) 31

7. Uma professora tem 237 balas para dar a seus 31 alunos. Qual é o númeromínimo de balas a mais que ela precisa conseguir para que todos os alunosrecebam a mesma quantidade de balas, sem sobrar nenhuma para ela?A) 11 B) 20 C) 21 D) 31 E) 41

Page 66: Eureka 2005

Sociedade Brasileira de Matemática

EUREKA! N°22, 2005

4

8. Dezoito quadrados iguais são construídos e sombreados como mostra a figura.Qual fração da área total é sombreada?

A) 7

18B)

4

9C)

1

3D)

5

9E)

1

2

9. O preço de uma corrida de táxi é igual a R$2,50 ("bandeirada"), mais R$0,10 porcada 100 metros rodados. Tenho apenas R$10,00 no bolso. Logo tenho dinheiropara uma corrida de até:A) 2,5 km B) 5,0 km C) 7,5 km D) 10,0 km E) 12,5 km

10. Um arquiteto apresenta ao seu cliente cinco plantas diferentes para o projeto deajardinamento de um terreno retangular, onde as linhas cheias representam acerca que deve ser construída para proteger as flores. As regiões claras são todasretangulares e o tipo de cerca é o mesmo em todos os casos. Em qual dosprojetos o custo da construção da cerca será maior?

A) B) C) D) E)

11. 108 crianças da 5ª e 6ª séries vão fazer um passeio numa caverna. São formadosgrupos iguais com mais de 5 porém menos de 20 alunos. Com relação ao númerode estudantes por grupo, de quantas formas diferentes eles podem ser feitos?A) 2 B) 8 C) 5 D) 4 E) 3

12. desenho ao lado mostra o mapa de um país(imaginário) constituído por cinco estados. Deseja-se colorir esse mapa com as cores verde, azul eamarela, de modo que dois estados vizinhos nãopossuam a mesma cor. De quantas maneirasdiferentes o mapa pode ser pintado?

A) 12 B) 6 C) 10 D) 24 E) 120

Page 67: Eureka 2005

Sociedade Brasileira de Matemática

EUREKA! N°22, 2005

5

13. Um artesão começa a trabalhar às 8h e produz 6 braceletes a cada vinte minutos;seu auxiliar começa a trabalhar uma hora depois e produz 8 braceletes do mesmotipo a cada meia hora. O artesão pára de trabalhar às 12h mas avisa ao seuauxiliar que este deverá continuar trabalhando até produzir o mesmo que ele. Aque horas o auxiliar irá parar?A) 12h B) 12h30min C) 13h D) 13h30minE) 14h30min

14. algarismo das unidades do número 1 × 3 × 5 × … × 97 × 99 éA) 1 B) 3 C) 5 D) 7 E) 9

15. Dois quadrados, cada um com área 25cm2, são colocados lado a lado para formarum retângulo. Qual é o perímetro do retângulo?A) 30 cm B) 25 cm C) 50 cm D) 20 cm E) 15 cm

16. Se girarmos o pentágono regular, ao lado, de um ângulo de 252°,em torno do seu centro, no sentido horário, qual figura seráobtida?

A) B) C) D) E)

17. Os resultados de uma pesquisa das cores de cabelo de 1200 pessoas sãomostrados no gráfico abaixo.

preto 24% castanho 30%

loiro

ruivo 16%

Quantas dessas pessoas possuem o cabelo loiro?A) 60 B) 320 C) 360 D) 400 B E) 840

Page 68: Eureka 2005

Sociedade Brasileira de Matemática

EUREKA! N°22, 2005

6

18. Um cubo pode ser construído, a partir dos dois pedaços de papelão apresentadosem uma das alternativas a seguir, bastando apenas dobrar nas linhas tracejadas eunir nas linhas contínuas. Esses dois pedaços são:

A) B)

C)

E)

D)

19. Ao somar cinco números consecutivos em sua calculadora, Esmeralda encontrouum número de 4 algarismos: 2 0 0 *. O último algarismo não está nítido, pois ovisor da calculadora está arranhado, mas ela sabe que ele não é zero. Estealgarismo só pode ser:A) 5 B) 4 C) 3 D) 2 E) 9

20. Sobre uma mesa estão três caixas e três objetos, cada um em uma caixadiferente: uma moeda, um grampo e uma borracha. Sabe-se que• A caixa verde está à esquerda da caixa azul;• A moeda está à esquerda da borracha;• A caixa vermelha está à direita do grampo;• A borracha está à direita da caixa vermelha.

Em que caixa está a moeda? A) Na caixa vermelha. B) Na caixa verde.

C) Na caixa azul.D) As informações fornecidas são insuficientes para se dar uma resposta.E) As informações fornecidas são contraditórias.

Page 69: Eureka 2005

Sociedade Brasileira de Matemática

EUREKA! N°22, 2005

7

21. Um feirante vende batatas e, para pesar, utiliza uma balança de dois pratos, umpeso de 1 kg, um peso de 3 kg e um peso de 10 kg. Considere a seguinteafirmação: “Este feirante consegue pesar (com uma pesagem) n quilogramas debatatas”. Quantos valores positivos de n tornam essa afirmação verdadeira,supondo que ele pode colocar pesos nos dois pratos?A) 7 B) 10 C) 12 D)13 E)14

22. O mapa ao lado mostra um conjuntoresidencial onde as casas, numeradas, sãointerligadas por 23 ruelas. O vendedor ZéRuela, que mora na casa 8, planeja passar portodas as outras casas e retornar à sua,percorrendo o menor número possível deruelas. Ele deixará de caminhar por quantasruelas?

A) 15 B) 10 C) 13 D)12 E) 11

1 2 3 4

5

9

6

1011 12

7 8

23. O arranjo a seguir, composto por 32 hexágonos, foi montado com varetas, todascom comprimento igual ao lado do hexágono. Quantas varetas, no mínimo, sãonecessárias para montar o arranjo?

………

A) 113 B) 123 C) 122 D) 132 E) 152

24. Observe a figura:

Duas das figuras abaixo representam o objeto acima colocado em outras posições.I) II)

Page 70: Eureka 2005

Sociedade Brasileira de Matemática

EUREKA! N°22, 2005

8

III) IV)

Elas são:A) I e II B) I e IV C) II e IV D) I e III E) II e III

25. Entre 1986 e 1989, época em que vocês ainda não tinham nascido, a moeda dopaís era o cruzado (Cz$). Com a imensa inflação que tivemos, a moeda foimudada algumas vezes: tivemos o cruzado novo, o cruzeiro, o cruzeiro real e,finalmente, o real. A conversão entre o cruzado e o real é: 1 real = 2.750.000.000cruzadosImagine que a moeda não tivesse mudado e que João, que ganha hoje 640 reaispor mês, tivesse que receber seu salário em notas novas de 1 cruzado. Se umapilha de 100 notas novas tem 1,5 cm de altura, o salário em cruzados de Joãofaria uma pilha de altura:A) 26,4 km B) 264 km C) 26 400 km D) 264 000 kmE) 2 640 000 km

PROBLEMAS – NÍVEL 2

1. Veja o problema No. 3 do Nível 1

2. Se m e n são inteiros não negativos com m < n, definimos m ∇ n como a somados inteiros entre m e n, incluindo m e n. Por exemplo, 5 ∇ 8 = 5 + 6 + 7 + 8 =26.

O valor numérico de 64

2622

∇∇

é:

A) 4 B) 6 C) 8 D) 10 E) 12

3. Entre 1986 e 1989, época em que vocês ainda não tinham nascido, a moeda dopaís era o cruzado (Cz$). Com a imensa inflação que tivemos, a moeda foimudada algumas vezes: tivemos o cruzado novo, o cruzeiro, o cruzeiro real e,finalmente, o real. A conversão entre o cruzado e o real é:

Page 71: Eureka 2005

Sociedade Brasileira de Matemática

EUREKA! N°22, 2005

9

1 real = 2.750.000.000 cruzados

Imagine que a moeda não tivesse mudado e que João, que ganha hoje 640 reaispor mês, tivesse que receber seu salário em notas novas de 1 cruzado. Se umapilha de 100 notas novas tem 1,5 cm de altura, o salário em cruzados de Joãofaria uma pilha de altura:A) 26,4kmB) 264kmC) 26400kmD) 264000kmE) 2640000km

4. Veja o problema No. 23 do Nível 1.5. Veja o problema No. 14 do Nível 1.6. Veja o problema No. 16 do Nível 1.

7. Há 1002 balas de banana e 1002 balas de maçã numa caixa. Lara tira, sem olharo sabor, duas balas da caixa. Seja p a probabilidade de as duas balas serem domesmo sabor e seja q a probabilidade de as duas balas serem de saboresdiferentes. Quanto vale a diferença entre p e q?

A) 0 B) 2004

1 C)

2003

1 D)

2003

2E)

1001

1

8. O perímetro de um retângulo é 100 e a diagonal mede x. Qual é a área doretângulo?

A) 625 – x2 B) 625 – 2

2xC) 1250 –

2

2x

D) 250 – 2

2xE) 2500 –

2

2x

9. Veja o problema No. 19 do Nível 1.

10. Para quantos inteiros positivos m o número 2

20042 −m

é um inteiro positivo?

A) um B) dois C) três D) quatroE) mais do que quatro

11. Se x + y = 8 e xy = 15, qual é o valor de x2 + 6xy + y2?A) 64 B) 109 C) 120 D) 124 E) 154

Page 72: Eureka 2005

Sociedade Brasileira de Matemática

EUREKA! N°22, 2005

10

12. Dois espelhos formam um ângulo de 30o no ponto V. Um raio de luz, vindo deuma fonte S, é emitido paralelamente a um dos espelhos e é refletido pelo outroespelho no ponto A, como mostra a figura. Depois de uma certa quantidade dereflexões, o raio retorna a S. Se AS e AV têm 1 metro de comprimento, adistância percorrida pelo raio de luz, em metros, é

30°

A S

V

A) 2 B) 2 3+ C) 1 2 3+ + D) ( )2 1 3+

E) 5 3

13. Na figura, quanto vale x?A) 6° B) 12° C) 18°D) 20° E) 24°

3x

4x

5x

6x

2x

14. Se 2(22x) = 4x + 64, então x é igual a:A) – 2 B) – 1 C) 1 D) 2 E) 3

15. Qual é o maior valor da soma dos algarismos da soma dos algarismos de umnúmero de três algarismos?A) 7 B) 8 C) 9 D) 10 E) 11

16. Veja o problema No. 10 do Nível 1.

17. Um ponto P pertence ao interior de um quadrado com 10 cm de lado. Nomáximo, quantos pontos da borda do quadrado podem estar a uma distância de 6cm do ponto P?A) 1 B) 2 C) 4 D) 6 E) 8

Page 73: Eureka 2005

Sociedade Brasileira de Matemática

EUREKA! N°22, 2005

11

18. Veja o problema No. 18 do Nível 1.

19. No triângulo PQR, a altura PF divide o lado QR em dois segmentos de medidasQF = 9 e RF = 5. Se PR = 13, qual é a medida de PQ?A) 5 B) 10 C) 15 D) 20 E) 25

20. Veja o problema No. 20 do Nível 1.

21. No desenho ao lado, o quadrilátero ABCD é umquadrado de lado 3 cm e os triângulos ABF eAED são ambos equiláteros. Qual é a área daregião destacada?

A) 2 cm2

B) 1,5 cm2

A) 3 cm2

D) 4,5 cm2

E) 2,5 cm2

A

B

C

DF

E

22. Uma folha quadrada foi cortada em 42 quadrados menores, dos quais um temárea maior do que 1 cm2 e os demais têm área de 1 cm2. Qual é a medida dolado da folha?A) 6 cm B) 12 cm C) 21 cm D) 19 cm E) 20 cm

23. Eu planejava fazer um curral quadrado, com uma certa área, usando uma certaquantidade de cerca de arame farpado. Descobri, porém, que tenho 10% amenos de cerca do que esperava. Por esta razão, a área cercada será:A) 5% menor B) 10% menor C) 19% menor D) 20% menorE) 25% menor

24. Veja o problema No. 13 do Nível 1.

20. Esmeralda, a digitadora, tentou digitar um número de seis algarismos, mas osdois algarismos 1 não apareceram (a tecla devia estar com defeito). O queapareceu foi 2004. Quantos são os números de seis algarismos que ela pode tertentado digitar?A) 4 B) 8 C) 10 D) 15 E) 20

Page 74: Eureka 2005

Sociedade Brasileira de Matemática

EUREKA! N°22, 2005

12

PROBLEMAS – NÍVEL 3

1. A função f é dada pela tabela a seguir.

1 2 3 4 5f(x) 4 1 3 5 2

Por exemplo, f(2) = 1. Quanto vale 2004 vezes

( (...( ( (4))...))f f f f�������

?

A) 1 B) 2 C) 3 D) 4 E) 5

2. Seja AB um segmento de comprimento 26, e sejam C e D pontos sobre osegmento AB tais que AC = 1 e AD = 8. Sejam E e F pontos sobre umasemicircunferência de diâmetro AB, sendo EC e FD perpendiculares a AB.Quanto mede o segmento EF?

A) 5 B) 25 C) 7 D) 27 E) 12

3. As alturas de um triângulo medem 12, 15 e 20. O maior ângulo interno dotriângulo medeA) 72o B) 75o C) 90o D) 108o E) 120o

4. Esmeralda, a digitadora, tentou digitar um número de seis algarismos, mas osdois algarismos 1 não apareceram (a tecla devia estar com defeito). O queapareceu foi 2004. Quantos são os números de seis algarismos que ela pode tertentado digitar?A) 4 B) 8 C) 10 D) 15 E) 20

5. O produto dos números que aparecem nas alternativas incorretas dessa questão éum cubo perfeito.Assinale a alternativa correta.A) 4 B) 8 C) 18 D) 54 E) 192

6. Qual é o menor inteiro positivo n para o qual qualquer subconjunto de nelementos de {1,2,3,…,20} contém dois números cuja diferença é 8?A) 2 B) 8 C) 12 D) 13 E) 15

Page 75: Eureka 2005

Sociedade Brasileira de Matemática

EUREKA! N°22, 2005

13

7. Sejam

2001

1001

5

3

3

2

1

1 2222

++++= �a

e

2003

1001

7

3

5

2

3

1 2222

++++= �b .

Qual é o inteiro mais próximo de a – b?A) 500 B) 501 C) 999 D) 1000 E) 1001

8. Uma ampulheta é formada por dois cones idênticos. Inicialmente, o conesuperior está cheio de areia e o cone inferior está vazio. A areia flui do conesuperior para o inferior com vazão constante. O cone superior se esvazia emexatamente uma hora e meia. Quanto tempo demora até que a altura da areia nocone inferior seja metade da altura da areia no cone superior?

A) 30min B) 10h C) 1h03min20s D) 1h10min12s E) 1h14min30s

9. A função real f, definida nos inteiros, satisfaz f(n) – (n + 1)f(2 – n) = (n + 3)2,para todo n inteiro. Quanto vale f(0)?A) –17 B) 0 C) 1 D) 2 E) 9

10. Com três algarismos distintos a, b e c, é possível formar 6 números de doisalgarismos distintos. Quantos conjuntos {a, b, c} são tais que a soma dos 6números formados é 484?A) Um B) Dois C) Três D) QuatroE) Mais que quatro

11. Dois cubos têm faces pintadas de ocre ou magenta. O primeiro cubo tem cincofaces ocres e uma face magenta. Quando os dois cubos são lançados, aprobabilidade de as faces viradas para cima dos dois cubos serem da mesma cor(sim, ocre e magenta são cores!) é 1/2. Quantas faces ocres tem o segundo cubo?A) 1 B) 2 C) 3 D) 4 E) 5

12. Veja o problema No. 10 do Nível 2.13. Veja o problema No. 12 do Nível 2.

Page 76: Eureka 2005

Sociedade Brasileira de Matemática

EUREKA! N°22, 2005

14

14. Para n inteiro positivo, definimos n! (lê-se “n fatorial”) o produto de todos osinteiros positivos menores ou iguais a n. Por exemplo, 6! = 1 ⋅ 2 ⋅ 3 ⋅ 4 ⋅ 5 ⋅ 6.Se n! = 215 ⋅ 36 ⋅ 53 ⋅ 72 ⋅ 11 ⋅ 13, então n é igual aA) 13 B) 14 C) 15 D) 16 E) 17

15. Constrói-se o quadrado ABXY sobre o lado AB do heptágono regular ABCDEFG,

exteriormente ao heptágono. Determine a medida do ângulo ˆBXC , emradianos.

A) 7

πB)

3

7

πC)

14

πD)

3

14

πE)

3

28

π

16. O conjunto das raízes reais da equação 2 1 2 1 2x x x x+ − + − − = éA) {1} B) {1, 2} C) [1, 2] D) ]1, 2[ E) {2}

17. No desenho ao lado, os segmentos e AB CD sãoperpendiculares ao segmento BC . Sabendo que oponto M pertence ao segmento AD e que otriângulo BMC é retângulo não isósceles, qual é aárea do triângulo ABM ?

A) 1 B) 6

5 C)

7

5 D)

8

5E)

9

5

M

A

B

2

6 C

4

D

18. Veja o problema No. 3 do Nível 2

19. O dono de uma loja empilhou vários blocos medindo 0,8m x 0,8m x 0,8m nocanto da loja e encostados numa parede de vidro que dá para a rua, conformemostra a figura abaixo.Quantos blocos no máximo, uma pessoa de 1,80m de altura que está do lado defora da loja pode enxergar?

Obs. Consideramos que uma pessoa pode enxergar uma caixa se consegue veruma pequena região de área positiva de sua superfície.

Page 77: Eureka 2005

Sociedade Brasileira de Matemática

EUREKA! N°22, 2005

15

A) 13 B) 14 C) 15 D) 16 E) 17

20. Veja o problema No. 4 do Nível 2.

21. Numa prova para uma sala com 30 alunos, a média aritmética das 10 pioresnotas é 3 e a média aritmética das 10 melhores notas é 9. O menor valor possívele o maior valor possível para a média da sala são, respectivamente:A) 6 e 7 B) 5 e 7 C) 4 e 6 D) 3 e 9 E) 4 e 8

22. Veja o problema No. 20 do Nível 1.23. Veja o Problema No. 25 do Nível 2.

24. Esmeralda escreveu (corretamente!) todos os números de 1 a 999, um atrás dooutro:

12345678910111213… 997998999. Quantas vezes aparece o agrupamento “21”, nesta ordem? A) 11 B) 21 C) 31 D) 41 E) 51

25. Um feirante vende batatas e, para pesar, utiliza uma balança de dois pratos, umpeso de 1 kg, um peso de 3 kg e um peso de 10 kg. Considere a seguinteafirmação: “Este feirante consegue pesar (com uma pesagem) n quilogramas debatatas”. Quantos valores positivos de n tornam essa afirmação verdadeira,supondo que ele pode colocar pesos nos dois pratos?A) 7 B) 10 C) 12 D)13 E)14

Page 78: Eureka 2005

Sociedade Brasileira de Matemática

EUREKA! N°22, 2005

16

GABARITO

NÍVEL 1 (5a. e 6a. séries)

1) B 6) B 11) D 16) B 21) D2) E 7) A 12) B 17) C 22) E3) A 8) B 13) D 18) E 23) B4) B 9) C 14) C 19) A 24) C5) D 10) C 15) A 20) A 25) D

NÍVEL 2 (7a. e 8a. séries)

1) A 6) B 11) D 16) C 21) D2) C 7) C 12) B 17) E 22) C3) D 8) C 13) C 18) E 23) C4) B 9) A 14) E 19) C 24) D5) C 10) B 15) D 20) A 25) D

NÍVEL 3 (Ensino Médio)

1) D 6) D 11) C 16) C 21) B2) D 7) B 12) B 17) B 22) A3) C 8) C 13) B 18) E 23) D4) D 9) A 14) D 19) B 24) C5) D 10) B 15) E 20) B 25) D

Page 79: Eureka 2005

Sociedade Brasileira de Matemática

EUREKA! N°22, 2005

17

XXVI OLIMPÍADA BRASILEIRA DE MATEMÁTICAProblemas e Soluções da Segunda Fase

PROBLEMAS – NÍVEL 1 PARTE A(Cada problema vale 3 pontos)

01. O número 1000…02 tem 20 zeros. Qual é a soma dos algarismos do número queobtemos como quociente quando dividimos esse número por 3?

02. A soma de dois números primos a e b é 34 e a soma dos primos a e c é 33.Quanto vale a + b + c?

03. No desenho, os quadriláteros ABCD, EFAGe IAJH são retângulos e H é ponto médio deAE.Calcule a razão entre a área do retângulo ABCDe o triângulo AHI.

A I

J H

G E

F B

D C

04. Dizemos que um número natural é teimoso se, ao ser elevado a qualquerexpoente inteiro positivo, termina com o mesmo algarismo. Por exemplo, 10 éteimoso, pois 2 3 410 ,10 ,10 ,..., são números que também terminam em zero. Quantosnúmeros naturais teimosos de três algarismos existem?

05. Qual é o maior número natural menor que 100 cuja soma dos divisores positivosé ímpar?

06. Na multiplicação a seguir, a, b e c são algarismos:

1

3

* * *

* * *

1 0 1

a b

b

c c

×

Calcule a + b + c.

Page 80: Eureka 2005

Sociedade Brasileira de Matemática

EUREKA! N°22, 2005

18

07. Esmeralda, de olhos vendados, retira cartões de uma urna contendo inicialmente100 cartões numerados de 1 a 100, cada um com um número diferente. Qual é onúmero mínimo de cartões que Esmeralda deve retirar para ter certeza de que onúmero do cartão seja um múltiplo de 4?

08. De quantos modos podemos sombrear quatro casas do tabuleiro 4 4× abaixo demodo que em cada linha e em cada coluna exista uma única casa sombreada?

09. Juntando cubinhos de mesmo volume mas feitos de materiais diferentes - cadacubo branco pesando 1 grama e cada cubo cinza pesando 2 gramas - formou-se umbloco retangular, conforme mostrado na figura abaixo. Qual é a massa, em gramas,desse bloco?

10. Na população de uma espécie rara de 1000 aves da floresta amazônica, 98%tinham cauda de cor verde. Após uma misteriosa epidemia que matou parte das avescom cauda verde, esta porcentagem caiu para 95%. Quantas aves foram eliminadascom a epidemia?

Page 81: Eureka 2005

Sociedade Brasileira de Matemática

EUREKA! N°22, 2005

19

PROBLEMAS – NÍVEL 1 PARTE B(Cada problema vale 10 pontos)

PROBLEMA 1:No desenho abaixo, o triângulo ABC é retângulo e os lados do polígono (regiãoescura) são paralelos ou coincidem com algum dos catetos do triângulo.

x

5 1 0

2

A

B CCalcule x de modo que a área do polígono seja igual à do triângulo.

PROBLEMA 2:Esmeralda, a digitadora, construiu uma tabela com 100 linhas e 100 colunas,preenchendo uma casa com 1, se o número da linha da casa divide o número dacoluna e com 0, caso contrário. Assim, por exemplo, a casa da linha 2 e da coluna 4foi preenchida com 1, porque 2 divide 4 e a casa na linha 3 e da coluna 7 foipreenchida com 0.

1

1 2 3 4 5 6 99 100 …

2

3

4

100

1 1 1 1 1 1 1 1

0 1 0 1 0 1 0 1

0 0 1 0 0 1 1 0

0 0 0 1

a) Qual a soma dos números escritos na linha 5?b) Qual a soma dos números da coluna 60?

PROBLEMA 3:a) É possível dividir o conjunto {12, 22,…,72} em dois grupos A e B de modo que a

soma dos elementos de A seja igual à soma dos elementos de B? Justifique.

b) É possível dividir o conjunto {12, 22, 32,…,92} em dois grupos C e D de modoque a soma dos elementos de C seja igual à soma dos elementos de D?Justifique.

Page 82: Eureka 2005

Sociedade Brasileira de Matemática

EUREKA! N°22, 2005

20

PROBLEMAS – NÍVEL 2 PARTE A(Cada problema vale 3 pontos)

01. Veja o problema No. 6 do Nível 1.02. Veja o problema No. 8 do Nível 1.

03. Qual é a soma dos algarismos do número 2004 2002 1998 1996 36 ?× × × +

04. Veja o problema No. 1 da Parte B do Nível 1.

05. Um polígono com 20 lados é chamado icoságono. Unindo-se três dos vértices deum icoságono regular obtemos triângulos. Quantos são triângulos retângulos?

PROBLEMAS – NÍVEL 2 PARTE B(Cada problema vale 10 pontos)

PROBLEMA 1:(a) É possível dividir o conjunto {12, 22,…,72} em dois grupos A e B de modo que a

soma dos elementos de A seja igual à soma dos elementos de B? Justifique.(b) É possível dividir o conjunto {12, 22, 32,…,92} em dois grupos C e D de modo

que a soma dos elementos de C seja igual à soma dos elementos de D?Justifique.

PROBLEMA 2:(a) Simplifique a expressão

(b) Certa calculadora tem duas teclas especiais: A e B. A tecla A transforma o

número x que está no visor em 1

x. A tecla B transforma o número x que está no

visor em 1 − x .Pedro tem um número no visor e aperta sucessivamente, de forma alternada, as duasteclas:

A, B, A, B, ….Após 1000 operações, o visor mostrava o número 2004. Que número Pedro tinhainicialmente no visor?

Page 83: Eureka 2005

Sociedade Brasileira de Matemática

EUREKA! N°22, 2005

21

PROBLEMA 3:Uma folha de papel retangular ABCD foidobrada de modo que o vértice B foi levado noponto B’ sobre o lado AD. A dobra é EF, com Esobre AB e F sobre CD.

Sabe-se que AE = 8, BE = 17 e C F = 3.

(a) Calcule a medida do segmento AB’.(b) Calcule a medida do lado AD.

PROBLEMA 4:Um número de 4 algarismos a b c d é chamado de legal quando a soma dos númerosformados pelos dois primeiros e pelos dois últimos algarismos é igual ao númeroformado pelos algarismos centrais (ou seja, ab + cd = bc). Por exemplo, 2307 é umnúmero legal pois 23 + 07 = 30.

(a) Qual é o menor número legal maior do que 2307?(b) Quantos são os números legais de 4 algarismos?

PROBLEMAS – NÍVEL 3

PROBLEMA 1:

Cada um dos números 1 2 2004, , ...,x x x pode ser igual a 2 1− ou a 2 1+ .Quantos valores inteiros distintos a soma2004

2 1 2 1 2 3 4 5 6 2003 2 0041

...k kk

x x x x x x x x x x−=

= + + + +∑ pode assumir?

PROBLEMA 2Seja ABCD um trapézio retângulo de bases AB e CD, com ângulos retos em A e D.Dado que a diagonal menor BD é perpendicular ao lado BC, determine o menor valor

possível para a razão .C D

AD

Page 84: Eureka 2005

Sociedade Brasileira de Matemática

EUREKA! N°22, 2005

22

PROBLEMA 3:Os doze alunos de uma turma de olimpíada saíam para jogar futebol todos os diasapós a aula de matemática, formando dois times de 6 jogadores cada e jogando entresi. A cada dia eles formavam dois times diferentes dos times formados em diasanteriores. Ao final do ano, eles verificaram que cada 5 alunos haviam jogado juntosnum mesmo time exatamente uma vez. Quantos times diferentes foram formados aolongo do ano?

PROBLEMA 4:Determine todas as soluções da equação 1 22 1 ,nn m−⋅ + = com n e m naturais.

PROBLEMA 5:Dizemos que um número inteiro positivo é sinistro quando a soma de seus fatoresprimos é igual à soma dos expoentes de sua decomposição em fatores primos.Encontre todos os números sinistros de quatro algarismos.

PROBLEMA 6:Sejam H, I e O o ortocentro, o incentro e o circuncentro do triângulo ABC,respectivamente. A reta CI corta o circuncírculo de ABC no ponto L, distinto de C.Sabe-se que AB = IL e AH = OH. Determine os ângulos do triângulo ABC.

Page 85: Eureka 2005

Sociedade Brasileira de Matemática

EUREKA! N°22, 2005

23

SOLUÇÕES – SEGUNDA FASE – NÍVEL 1 – PARTE A

Problema 01 02 03 04 05 06 07 08 09 10Resposta 064 036 032 360 098 010 076 024 262 600

SOLUÇÕES – SEGUNDA FASE – NÍVEL 1 – PARTE B

SOLUÇÃO DO PROBLEMA 1:O polígono consiste na reunião de dois retângulos: um deles tem largura 10 e altura 2e o outro tem largura 5 e altura 2+x ; o triângulo tem catetos de medidas 15 e

2+x . Como a área do polígono é igual à área do triângulo, temos

630530152010402

)2(15)2(5210 =⇔=⇔+=++⇔+=++⋅ xxxx

xx

SOLUÇÃO DO PROBLEMA 2:a) Cada linha apresenta 1 nas colunas cujos números são múltiplos do número dalinha. Assim, a linha 5 tem 1 nas colunas 5, 10, 15, etc. Até 100, existem 20múltiplos de 5, logo a soma dos números na linha 5 é igual a 20.b) Cada coluna apresenta 1 no cruzamento com as linhas cujos números sãodivisores do número da coluna. Assim, a soma dos números da coluna 60 é igual ao

número de divisores de 60. Como 53260 2 ××= , concluímos que 60 tem 3.2.2 =12 divisores. Logo, a soma dos números da coluna 60 é 12.

SOLUÇÃO DO PROBLEMA 3:a) A soma total dos elementos é

2 2 2 2 2 2 21 2 3 4 5 6 7 1 4 9 16 25 36 49 140+ + + + + + = + + + + + + = .Logo, cada um dos grupos deve conter elementos que somem 70. Examinando asparcelas, vemos que 49 + 1 + 4 +16 = 70. Assim podemos escrever, por exemplo, A= {12, 22, 42, 72} e B = {32, 52, 62}.b) Como

12 + 22 + 32 + 42 + 52 + 62 + 72 + 82 + 92 = 140 + 64 + 81 = 285é ímpar, é impossível dividir em dois grupos de mesma soma.

SOLUÇÕES – SEGUNDA FASE – NÍVEL 2 – PARTE A

Problema 01 02 03 04 05Resposta 010 024 048 006 180

Page 86: Eureka 2005

Sociedade Brasileira de Matemática

EUREKA! N°22, 2005

24

SOLUÇÕES – SEGUNDA FASE – NÍVEL 2 – PARTE B

SOLUÇÃO DO PROBLEMA 1:Ver o problema 3 – Parte B do Nível 1

SOLUÇÃO DO PROBLEMA 2:

a) 1 1 1 1 1

1 1 1 1 1 1 11 1 1 11 1 1

1 1 1 11

xx x

x x xx x x

x x

−− = − = − = − = − = + − =− − −− − −− − −−

1 1 20032004 2004 2004 2004 1

1 1 2004x x

x xx x

− − −⇔ = ⇔ = ⇔ − = − ⇔ =− −

SOLUÇÃO DO PROBLEMA 3:a) A partir da dobra da folha podemos ver que B'E = BE = 17, e como AE = 8,

aplicando o teorema de Pitágoras temos 2 2 2 2´ ´ 17 8 15AB B E AE= − = − = .

SOLUÇÃO DO PROBLEMA 4:Como 10 10 10 10 9( )ab cd bc a b c d b c a d b c+ = ⇔ + + + = + ⇔ + = − , ou seja,10a d+ é o número de dois algarismo a d , e é um múltiplo de 9.

a) Mantendo a = 2, temos d = 7. Além disso, 10 2 7 9( ) 3b c b c⋅ + = − ⇔ − = . Omenor valor de b que podemos escolher, após 3, é 4, e nesse caso, c = 1. O númeroprocurado é, então, 2417.

b) Uma vez que escolhemos cb − , a e d estão determinados: a é o algarismo dasdezenas de 9( )b c− , e d, o das unidades. Além disso, 9( ) 10 2b c b c− ≥ ⇔ − ≥ .

Se 2b c− = , ( ) ( ) ( ) ( ) ( ){ }, 2,0 ; 3,1 ; 4,2 ;...; 9,0b c ∈ , um total de 8 possibilidades. Da

mesma forma, vemos que se 3b c− = , ( ) ( ) ( ) ( ) ( ){ }, 3,0 ; 4,1 ; 5,2 ;...; 9,6b c ∈ , há um

total de 7 possibilidades. Para 4b c− = , ( ) ( ) ( ) ( ) ( ){ }, 4,0 ; 5,1 ; 6,2 ;...; 9,5b c ∈ , 6

possibilidades, 5b c− = ,

( ) ( ) ( ) ( ) ( ){ }, 5,0 ; 6,1 ; 7,2 ;...; 9,4b c ∈ , 5 possibilidades, 6b c− = ,

( ) ( ) ( ) ( ) ( ){ }, 6,0 ; 7,1 ; 8,2 ; 9,3b c ∈ , 4 possibilidades, 7b c− = ,

( ) ( ) ( ) ( ){ }, 7,0 ; 8,1 ; 9,2b c ∈ , 3 possibilidades, 8b c− = , ( ) ( ) ( ){ }, 8,0 ; 9,1b c ∈ , 2

possibilidades e, finalmente, para 9b c− = , ( ) ( ), 9,0b c = , 1 possibilidade.

Há, portanto, um total de 8 7 6 5 4 3 2 1 36+ + + + + + + = números legais.

Page 87: Eureka 2005

Sociedade Brasileira de Matemática

EUREKA! N°22, 2005

25

SOLUÇÕES – SEGUNDA FASE – NÍVEL 3

SOLUÇÃO DO PROBLEMA 1:

Os possíveis produtos 2 1 2k kx x− são ( 2 1)( 2 1) 3 2 2,− − = −

( 2 1)( 2 1) 3 2 2+ + = + e ( 2 1)( 2 1) 1.− + = Suponha que a produtos são

iguais a 3 2 2,− b produtos são iguais a 3 2 2+ e 1002 a b− − produtos são iguaisa 1.

A soma é igual a

(3 2 2) (3 2 2) 1002 1002 2 2 2( ) 2.a b a b a b b a− + + + − − = + + + −

Assim, para que a soma seja inteira, devemos ter a = b. Logo a soma é igual a1002 4 .a+

Como a varia de 0 a 501 (pois a + b não pode ser maior que 1002), a soma podeassumir 502 valores inteiros.

SOLUÇÃO DO PROBLEMA 2:

Seja � .ABD BDC α= = Então cos

BDDC

α= e sen AD BD α= , donde

1 2cos 2. sen sen cos sen2

BDDC

AD BDα

α α α α= = = ≥

A igualdade ocorre quando sen 2 1,α = ou seja, quando 45 .α = °

Page 88: Eureka 2005

Sociedade Brasileira de Matemática

EUREKA! N°22, 2005

26

SEGUNDA SOLUÇÃO DO PROBLEMA 2:

Sejam H a projeção de B sobre DC, DH = m, HC = n e BH = h. ABDH é então umretângulo, donde AD = BH = h.

Como o triângulo CBD é retângulo, temos 2 .h mn= Logo,

.DC DC m n m n

AD BH h mn

+ += = =

Mas sabemos que 2( ) 0,m n− ≥ donde 2 .m n mn+ ≥ A igualdade ocorrequando m = n.Segue que

2. DC m n

AD mn

+= ≥

SOLUÇÃO DO PROBLEMA 3:Para cada grupo de 5 alunos, existe um único time formado que os contém. Logo,

contamos 7925

89101112

5

12=

⋅⋅⋅⋅=

!

times para cada 5 alunos escolhidos. Por

outro lado, em cada time de 6 jogadores, temos 65

6=

modos de escolhermos

cinco jogadores, ou seja, existem 6 grupos de 5 jogadores que geram o mesmo time

na nossa primeira contagem. Logo, o total de times formados é igual a 1326

792 = .

SEGUNDA SOLUÇÃO DO PROBLEMA 3:

6

12 maneiras de escolher 6 dentre 12 alunos. Além disso, fixados 5 alunos, há 7

maneiras de montar um time com esses 5 alunos mais outro aluno. Assim,considerando que cada 5 alunos jogaram juntos num mesmo time exatamente umavez, o total de maneiras de escolher 6 dentre 12 alunos é igual a 7 vezes o número de

Page 89: Eureka 2005

Sociedade Brasileira de Matemática

EUREKA! N°22, 2005

27

maneiras de formar os times ao longo do ano. Logo o número de maneiras de formar

os times ao longo do ano é 1321234567

7891011126

12

7

1 =⋅⋅⋅⋅⋅⋅⋅⋅⋅⋅⋅=

.

SOLUÇÃO DO PROBLEMA 4:

A equação é equivalente a 12 ( 1)( 1).nn m m−⋅ = − + Suponha n > 3. Temos m ímpar,

digamos m = 2k +1. A equação fica então 32 ( 1).nn k k−⋅ = + Portanto, 32n− divide k

ou k + 1, pois k ou k + 1 (o que for ímpar) divide n. Assim, 31 2nk −+ ≥ e

,k n≤ donde 321 −≥+ nn .

Mostremos, por indução, que 321 −<+ nn para n > 5. Para n = 6 (base de indução),temos 6 + 1 = 7 e 26 – 3 = 8. Supondo que a desigualdade é válida para n = k,provemos que a mesma é válida para n = k + 1 (passo indutivo). De fato, temos k + 1< 2k – 3 ⇔ 2(k + 1) < 2k – 2. Como k + 2 < 2(k + 1), temos k + 2 < 2k – 2, completando ademonstração.

Assim, basta testar 50 ≤≤ n . Portanto as soluções são (m; n) = (1;0) e (m; n) =(9;5).

SOLUÇÃO DO PROBLEMA 5:

Seja 1 21 2 ... ,k

kn p p pαα α= (com 1 2 ... kp p p< < < primos) um número sinistro. Como

2ip ≥ para todo i, 1 2 ...2 .kn α α α+ + +≥

Como n tem 4 algarismos, 10000,n < donde 1 2 ...2 10000k nα α α+ + + ≤ < , e logo

1 2 ... 13.kα α α+ + + ≤Se um dos fatores primos fosse maior ou igual a 11, a soma dos fatores primos seria

11≥ , donde 1 2 ... 11kα α α+ + + ≥ e 102 11 10000n ≥ ⋅ > , absurdo.

Assim, os únicos fatores primos possíveis são 2, 3, 5 e 7. Como 53 1000,< se a soma

dos expoentes for 5,≤ o número deve ser 55 3125= . A soma pode ser também igual

a 7, donde o número pode ser 4 32 5 2000,⋅ = ou 3 42 5 5000⋅ = (note que77 10000> ).

Não pode ser igual a 8 3 5= + , pois 73 5 10000.⋅ >Pode ser igual a 9 = 7 + 2, podendo o número ser igual a 82 7 1792⋅ = ou

7 22 7 6272.⋅ =

Page 90: Eureka 2005

Sociedade Brasileira de Matemática

EUREKA! N°22, 2005

28

Pode ser igual a 10 = 2 + 3 + 5, podendo o número ser igual a82 3 5 3840,⋅ ⋅ = 7 22 3 5 5760,⋅ ⋅ = 7 22 3 5 9600⋅ ⋅ = ou 6 32 3 5 8640⋅ ⋅ = (note que os

fatores primos não podem ser 3 e 7, pois 93 7 10000).⋅ >A soma não pode ser 11, nem 12 (pois 102 3 7⋅ ⋅ e 115 7⋅ são maiores que 10000)nem 13. Assim os números sinistros de quatro algarismos são 55 3125= ,

4 32 5 2000,⋅ = 3 42 5 5000,⋅ = 82 7 1792,⋅ = 7 22 7 6272,⋅ =82 3 5 3840,⋅ ⋅ = 7 22 3 5 5760,⋅ ⋅ = 7 22 3 5 9600⋅ ⋅ = e 6 32 3 5 8640.⋅ ⋅ =

SOLUÇÃO DO PROBLEMA 6:Sendo α, β e γ as medidas dos ângulos internos nos vértices A, B e C,

respectivamente, temos m(∠ IBL) = m(∠ BIL) 2

α + β= . Logo BL = IL, e como BL

= ΑL e IL = AB, concluímos que o triângulo ABL é equilátero, logo o arco AB mede60o e, portanto, m(∠ ACB) = 120o.

O quadrilátero CXHY é inscritível, onde X e Y são os pés das alturas traçadas de A eB. Logo ∠ AHB mede 180o − 120o = 60o. Como m(∠ AOB) = 120o, concluímos que oquadrilátero OAHB é inscritível. (Isto também pode ser provado, por exemplo,utilizando-se a propriedade de que o simétrico de H em relação a AB pertence aocircuncírculo de ABC).Isto implica que m(∠ AHO) = m(∠ ABO) = 30o, e como OH = AH, temos m(∠ AOH) =m(∠ OAH) = 75o.Finalmente, temos m(∠ BAC) = m(∠ OAH) − m(∠ OAB) − m(∠ XAH) = 75o − 30o −30o = 15o; e m(∠ ABC) = 180o − 120o − 15o = 45o.

O

A B

C

H

L

X

Y

Page 91: Eureka 2005

Sociedade Brasileira de Matemática

EUREKA! N°22, 2005

29

XXVI OLIMPÍADA BRASILEIRA DE MATEMÁTICAProblemas e Soluções da Terceira Fase

PROBLEMAS – NÍVEL 1

PROBLEMA 1:Encontre todos os números naturais n de três algarismos que possuem todas aspropriedades abaixo:

• n é ímpar;• n é um quadrado perfeito;• A soma dos quadrados dos algarismos de n é um quadrado perfeito.

PROBLEMA 2:Com quatro triângulos eqüiláterosde lado 1 é possível formar umapeça, no formato de um triânguloeqüilátero de lado 2, como mostraa figura ao lado.

Imagine que você tenha muitos triângulos eqüiláteros de lado 1 de três tipos:brancos, pretos e cinzas para formar peças como no exemplo acima. Duas peçasassim formadas são consideradas iguais quando podemos obter uma delas girando aoutra, conforme ilustrado abaixo, à esquerda.

Par de peças iguais Par de peças diferentesQuantas peças diferentes podem ser formadas nas condições apresentadas?

PROBLEMA 3:Dizemos que um número natural é composto quando pode ser escrito como produtode dois números naturais maiores que 1. Assim, por exemplo, 91 é composto porquepodemos escrever 91 = 7 × 13.Mostre que o número

Page 92: Eureka 2005

Sociedade Brasileira de Matemática

EUREKA! N°22, 2005

30

12220042

+

+

é composto.

PROBLEMA 4:Arnaldo e Bernaldo disputam um jogo num tabuleiro 2 × n:

As peças do jogo são dominós 2 × 1. Inicialmente Arnaldo coloca um dominócobrindo exatamente duas casas do tabuleiro, na horizontal ou na vertical. Osjogadores se revezam colocando uma peça no tabuleiro, na horizontal ou na vertical,sempre cobrindo exatamente duas casas do tabuleiro. Não é permitido colocar umapeça sobre outra já colocada anteriormente.Quem não conseguir colocar uma peça no tabuleiro perde.

Qual dos dois jogadores tem uma estratégia vencedora, ou seja, uma estratégia que oleva à vitória quaisquer que sejam as jogadas de seu adversário, para:(a) n = 2004?(b) n = 2005?

PROBLEMA 5:Considere o polígono P de 6 lados.

Page 93: Eureka 2005

Sociedade Brasileira de Matemática

EUREKA! N°22, 2005

31

Com cópias de P, podemos cobrir todo o plano, sem sobreposições, como mostradoa seguir.

Existe um polígono de 13 lados com o qual é possível cobrir todo o plano com suascópias, sem sobreposições? Caso seja possível, apresente um polígono. Caso nãoseja, diga o porquê.

PROBLEMAS – NÍVEL 2

PROBLEMA 1:Na figura, ABC e DAE são triângulos isósceles (AB = AC = AD = DE) e os ângulosBAC e ADE medem 36°.

a) Utilizando propriedades geométricas, calcule a medida do ângulo CDE ˆ .b) Sabendo que BC = 2, calcule a medida do segmento DC.c) Calcule a medida do segmento AC.

Page 94: Eureka 2005

Sociedade Brasileira de Matemática

EUREKA! N°22, 2005

32

PROBLEMA 2:A seqüência de algarismos

1, 2, 3, 4, 0, 9, 6, 9, 4, 8, 7, …

é construída da seguinte maneira: cada elemento, a partir do quinto, é igual ao últimoalgarismo da soma dos quatro anteriores.a) Os algarismos 2, 0, 0, 4, juntos e nesta ordem, aparecem na seqüência?b) Os algarismos iniciais 1, 2, 3, 4, juntos e nesta ordem, aparecem novamente naseqüência?

PROBLEMA 3:Esmeralda tem uma pilha com 100 pedras. Ela divide essa pilha em duas novaspilhas e em seguida multiplica as quantidades de pedras nessas duas novas pilhas eescreve o produto em um quadro. Ela então escolhe uma pilha com mais de umapedra e repete esse procedimento: a pilha é dividida em duas, as quantidades depedras nessas duas pilhas são multiplicadas e o produto escrito no quadro. Estaoperação é realizada até se obter apenas pilhas com 1 pedra cada.Quais são os possíveis valores da soma de todos os produtos escritos no quadro?

PROBLEMA 4:Em um jogo para dois participantes, Arnaldo e Bernaldo alternadamente escolhemum número inteiro positivo. A cada jogada, deve-se escolher um número maior que oúltimo número escolhido e menor que o dobro do último número escolhido.Nesse jogo, vence o jogador que conseguir escolher o número 2004. Arnaldo jogaprimeiro e inicia com o número 2. Qual dos dois tem estratégia vencedora, ou seja,consegue escolher o número 2004 independentemente das jogadas do adversário?

PROBLEMA 5:Seja D o ponto médio da hipotenusa AB de um triângulo retângulo ABC. Sejam O1 eO2 os circuncentros dos triângulos ADC e DBC, respectivamente.

a) Mostre que 1 2ˆO DO é reto.

b) Mostre que AB é tangente ao círculo de diâmetro 1 2O O .

PROBLEMA 6:Considere todas as maneiras de colocarmos nas casas de um tabuleiro 1010×exatamente dez vezes cada um dos algarismos 0, 1, 2, …, 9.Encontre o maior inteiro n com a propriedade de que, em cada tabuleiro, algumalinha ou alguma coluna contenha pelo menos n algarismos diferentes.

Page 95: Eureka 2005

Sociedade Brasileira de Matemática

EUREKA! N°22, 2005

33

PROBLEMAS – NÍVEL 3

PROBLEMA 1:Seja ABCD um quadrilátero convexo. Prove que os incírculos de ABC, BCD, CDA eDAB têm um ponto em comum se, e somente se, ABCD é um losango.

PROBLEMA 2:Determine todos os valores de n tais que é possível dividir um triângulo em ntriângulos de modo que não haja três vértices alinhados e em cada vértice incida omesmo número de segmentos.

Mostramos a seguir tal divisão para n = 7. Observe que em cada um dos seis vérticesincidem quatro segmentos.

PROBLEMA 3:Seja 200421 ,,, xxx � uma seqüência de números inteiros satisfazendo

kkkk xxxx 123 +++ += , 20011 ≤≤ k .

É possível que mais da metade de seus termos sejam negativos?

PROBLEMA 4:Considere todas as maneiras de colocarmos nas casas de um tabuleiro 1010×exatamente dez vezes cada um dos algarismos 0, 1, 2, …, 9.

Encontre o maior inteiro n com a propriedade de que, em cada tabuleiro, algumalinha ou alguma coluna contenha pelo menos n algarismos diferentes.

Page 96: Eureka 2005

Sociedade Brasileira de Matemática

EUREKA! N°22, 2005

34

PROBLEMA 5:

Considere a seqüência Nnna ∈)( com 13210 ==== aaaa e 22314 −−−− += nnnnn aaaaa .

Mostre que todos os termos dessa seqüência são números inteiros.

PROBLEMA 6:Sejam a e b números reais. Considere a função baf , : R2 → R2 definida por

);();( 2, xxbyayxf ba −−= . Sendo 2);( RyxP ∈= , definimos PPf ba =)(0

, e

))(()( ,,1

, PffPf kbaba

kba =+ , para k inteiro não negativo.

O conjunto per(a; b) dos pontos periódicos da função baf , é o conjuntos dos pontos

P de R2 para os quais existe um inteiro positivo n tal que PPf nba =)( , .

Fixado o real b, prove que o conjunto }0),(|{ /≠∈= baperRaAb tem um menorelemento. Calcule esse menor elemento.

SOLUÇÕES – NÍVEL 1

PROBLEMA 1: SOLUÇÃO DE DIANA VAISMAN (RIO DE JANEIRO - RJ)

Números ímpares que são quadrados perfeitos Soma dos quadrados dos algarismos

121 1 + 4 + 1 = 6169 1 + 36 + 81 = 118225 4 + 4 + 25 = 33289 4 + 64 +81 = 149361 9 + 36 + 1 = 46441 16 + 16 + 1 = 33529 25 + 4 + 81 = 110625 36 + 4 + 25 = 65729 49 + 4 + 81 = 134841 64 + 16 + 1 = 81961 81 + 36 + 1 = 118

Resposta: 841

Page 97: Eureka 2005

Sociedade Brasileira de Matemática

EUREKA! N°22, 2005

35

PROBLEMA 2: SOLUÇÃO DE HUGO FONSECA ARAÚJO (JUIZ DE FORA - MG):Posso pintar de 33 modos.Com 1 cor o desenho é este:

z

z z z

sendo z uma das três cores.Com 2 cores tenho estes desenhos:

x

x z x

x

z z x

x

z x x

Sendo x uma das três cores e z também.Com 3 cores tenho estes desenhos

z

x y y

z

y x y

Sendo x uma das três cores e y e z também.Com 1 cor tenho 3 × 1 possibilidades.Com 2 cores tenho 3 × 2 × 3 = 18 possibilidades.Com 3 cores tenho 3 × 2 × 1 × 2 = 12 possibilidades.No total tenho 12 + 18 + 3 = 33 possibilidades.

PROBLEMA 3: SOLUÇÃO DE ILLAN FEIMAN HALPERN (ITATIAIA - RJ):

O número 2004(2 2)2 1+ + é equivalente a

2003(2 1)4 1.+ +Como toda potência de 2 é par então 20032 1+ será ímpar.Como 4 elevado a um número ímpar dá um número cujo último algarismo é 4, então

2003(2 1)4 1+ + terá 5 como último algarismo.Como todo número que termina com 5 é múltiplo de 5 então o número

2004(2 2)2 1+ + será múltiplo de 5 e poderá ser escrito como 5x , com x maior do que 1, o

que prova que 2004(2 2)2 1+ + é composto.

Page 98: Eureka 2005

Sociedade Brasileira de Matemática

EUREKA! N°22, 2005

36

PROBLEMA 4: SOLUÇÃO DE JAMES JUN HONG (SÃO PAULO – SP):Para n ímpar, Arnaldo tem a estratégia vencedora. Para n par, Bernaldo tem aestratégia vencedora.Quando o número n for ímpar, basta Arnaldo começar com um dominó na vertical.Sejam quais forem as jogadas de Bernaldo, Arnaldo vencerá. Se Bernaldo puser umdominó na vertical, Arnaldo deverá por na vertical também, em qualquer lugar dotabuleiro. Se Bernaldo puser na horizontal, Arnaldo deverá por um exatamente acimaou abaixo da peça de Bernaldo. Seguindo as regras, Arnaldo vencerá porque nãosobrará nenhum espaço e o número de jogadas será ímpar. Como Arnaldo começa,ele também termina e Bernaldo não poderá colocar uma peça no tabuleiro cheio.Se o número n for par, Bernaldo vence. As regras para preenchimento são as mesmasjá citadas: se Arnaldo puser um dominó na vertical, Bernaldo deverá fazer o mesmo.Se Arnaldo puser na horizontal, Bernaldo deverá colocar na horizontal, acima ouabaixo da peça de Arnaldo. Como o número de jogadas, seguindo as regras, será par,Bernaldo terminará de preencher o tabuleiro porque Arnaldo começa.Conseqüentemente, Arnaldo não poderá colocar nenhuma peça e perderá.Lembramos que sendo o tabuleiro 2 × n, n é o número máximo de jogadas pois odominó ocupa 2 casas do tabuleiro.a) sendo 2004 par, Bernaldo vence.b) sendo 2005 ímpar, Arnaldo vence.

PROBLEMA 5:SOLUÇÕES DE WALLACE J. INOCÊNCIO e CAROLINE RIGOLON VEIGA (RIO DE JANEIRO - RJ)

Page 99: Eureka 2005

Sociedade Brasileira de Matemática

EUREKA! N°22, 2005

37

SOLUÇÕES – NÍVEL 2

PROBLEMA 1: SOLUÇÃO DE VALTER BARBALHO LIMA FILHO :

D

C

A E B

F

72° 36°

72° 72°

36° 36°

36° 72° 72°

2

2

2

2

a) I) � �

180 3672 36

2AD DE DEA DAE DAF

° − °= ⇒ = = = ° ⇒ = °

II) � �

180 3672 36

2AD AC ADC ACD EDC

° − °= ⇒ = = = ° ⇒ = °

b) I) ( ) 2.ADE ACB LAL AE∆ ≡ ∆ ⇒ =II) 72 2AFE AE AF∠ = ° ⇒ = =III) � � 36 2ADF F AD AF FD= = ° ⇒ = =IV) � � 72 2DFC DCF DC FD= = ° ⇒ = =

c) I) Seja .AC AB x= =

Temos2 4

~ 42

AE FE FEAFE ACD x FE FE

AB CB x x∆ ∆ = = ⇒ = ⇒ ⋅ = ⇒ =

II) 24 4x

DE x DF DE FE DF x DFx x

−= ⇒ = − ⇒ = − ⇒ =

III) 2

242 2 2 4 0 1 5

xDF AF x x x

x

−= = ⇒ = ⇒ − − = ⇒ = +

IV) 1 5.AC DE x= = = +

PROBLEMA 2: SOLUÇÃO DE RÉGIS PRADO BARBOSA (FORTALEZA - CE):a) note que se um natural n é par ou ímpar, n – 10k terá mesma paridade pois

par =2 2 10 2( 5 ) par

ímpar =2 1 2 1 10 2( 5 ) 1 par

n t t k t k

n t t k t k

⇒ − = − → + ⇒ + − = − + →

Page 100: Eureka 2005

Sociedade Brasileira de Matemática

EUREKA! N°22, 2005

38

logo se a soma de quatro números da seqüência tiver certa paridade o quinto númeroterá a mesma.Seja agora: i = No. ímpar p = No. par1, 2, 3, 4, 0, 9, 6, 9, 4, 8, 7, 8, 7, 0, 2…i, p, i, p, p, i, p, i, p, p, i, p, i, p, p…

Note que as paridades se repetem de 5 em 5 números. Provemos que de fato ela ésempre assim.

Por indução: 1, 2, 3, 4, 0, 9, 6, 9, 4, 8…Caso inicial:

1 2

, , , , , , , , , ...a a

i p i p p i p i p p����� �����

Hipótese: suponha que dá certo até o ka −ésimo período.

Passo indutivo: ... , , , ,

ka

i p i p p�����

a, b, c, d, e…

( 10),a p i p p i mod≡ + + + ≡ logo a é ímpar.( 10),b i p p a i p p i p mod≡ + + + ≡ + + + ≡ logo b é par.( 10),c p p a b p p i p i mod≡ + + + ≡ + + + ≡ logo c é ímpar.( 10),d p a b c p i p i p mod≡ + + + ≡ + + + ≡ logo d é par.( 10),e a b c d i p i p p mod≡ + + + ≡ + + + ≡ logo e é par.

Assim, a, b, c, d, e =

1

, , , ,

ka

i p i p p

+

�����, e logo nunca poderemos chegar a 2, 0, 0, 4, pois não

poderemos ter 4 p's seguidos.

b) Veja que, para esta seqüência, a seqüência dos grupos de 4 termos consecutivosdela não poderá ter infinitos termos diferentes, pois não temos infinitaspossibilidades para a, b, c, d: serão no máximo

� � � �

4

10 10 10 10

, , , 10a b c d = possibilidades (aqui são contadas possibilidades que, assim

como vimos no item a), não podem aparecer, mas o que queremos com isso não éachar um número exato mas sim um máximo e mostrar que as possibilidades sãofinitas). Assim, num certo ponto começarão a se repetir os números formando umperíodo, assim:

Page 101: Eureka 2005

Sociedade Brasileira de Matemática

EUREKA! N°22, 2005

39

a, b, c, d y

x 1, 2, 3, 4 …

Mas note que se: y é o último número antes de começar a repetição, x é o últimonúmero do período e a, b, c, d são os 4 primeiros números do período, teremos y + a+ b + c ≡ d (mod 10) e x + a + b + c ≡ d (mod 10)

logo y ≡ d – a – b – c ≡ x (mod 10), e x ≡ y (mod 10), 0 ≤ x ≤ 9 e 0 ≤ y ≤ 9 ⇒ x = y.Se fizermos isso várias vezes veremos que na verdade o período é:

…1, 2, 3, 4 …

Logo ele aparecerá novamente.

PROBLEMA 3: SOLUÇÃO DE RÉGIS PRADO BARBOSA (FORTALEZA - CE):Note que se temos uma pilha com a pedras e fazemos o processo dividindo-a emduas pilhas com b e e pedras o número escrito será b × e, mas se a foi dividido em be, temos:

2 2 22 2 2 2 2 2 2 2( ) 2 2

2

a b eb e a b e a b be e a be a b e be

− −+ = ⇒ + = ⇒ + + = ⇒ = − − ⇒ = .

Assim note que a soma será:

1 2 3 4 1... i ib b b b b b S+⋅ + ⋅ + ⋅ = ( ib = No. de pedras na pilha i)2 2 2 2 2 22 2 2

1 3 4 1 11 2100...

2 2 2i i ib b b b b bb b

S− +− − − −− − + + =

Page 102: Eureka 2005

Sociedade Brasileira de Matemática

EUREKA! N°22, 2005

40

veja só que se 1tb > ele ainda será dividido em mais pilhas, ou seja quando ele

aparecer será: 2 2 2

2x t yb b b− −

mas se é 1tb > aparecerá também 2 2 2

2t z wb b b− −

. Assim, os

2tb terão soma 0, a não ser quando 1tb = e, como a pilha inicial tem 100 pedras, no

fim são 100 pilhas com 1 pedra cada, e teremos:2 2 2 2

2100 vezes

100 1 1 ... 110000 100 1 9900

.2 2 2

S

− − − −− ⋅= = =

�������

99004950

2S S= ⇒ = e esta é a única possibilidade.

PROBLEMA 4: SOLUÇÃO DE HENRIQUE PONDÉ DE OLIVEIRA PINTO (SALVADOR - BA):Seja uma Rodada definida como a jogada de cada jogador assim na 1ª RodadaArnaldo escolheu o número 2 e na 2ª Rodada Bernaldo será obrigado a escolher umnúmero maior que 2 e menor que 2 × 2 = 4 logo na 2ª Rodada será escolhido onúmero 3. Sejam os dois jogadores do enunciado os jogadores X e Y.

1. Suponha que o jogador X escolhe 2004 e ganha na Rodada n.2. Para isso Y teria que ter escolhido qualquer número entre 1003 e 2003 na

Rodada n – 1 e isso é visto facilmente.3. A jogada de X na Rodada n – 2 teria que ser 1002, pois se fosse maior ou

menor que 1002 Y não seria obrigado a escolher um número entre 1003 e2003 na Rodada n – 1.

4. Para X ter escolhido 1002 na Rodada n – 2 Y teria que ter escolhido umnúmero entre 502 e 1001 na Rodada n – 3.

5. A jogada de X na Rodada n – 4 teria que ser 501, pois se fosse maior oumenor que 501, Y não seria obrigado a escolher um número entre 502 e 1001na Rodada n – 3.

6. Para X ter escolhido 501 na Rodada n – 4, Y teria que ter escolhido umnúmero entre 251 e 500 na Rodada n – 5.

7. A jogada de X na Rodada n – 6 teria que ser 250, pois se fosse maior oumenor que 250, Y não seria obrigado a escolher um número entre 251 e 500na Rodada n – 5.

Agora que o raciocínio da questão já foi monstrado podemos continuar sem escrevertanto.

Rodada n – 6: (X): 250 (Como 7. mostrou)Rodada n – 7: (Y): Entre 126 e 249Rodada n – 8: (X): 125

Page 103: Eureka 2005

Sociedade Brasileira de Matemática

EUREKA! N°22, 2005

41

Rodada n – 9: (Y): Entre 63 e 124Rodada n – 10: (X): 62Rodada n – 11: (Y): Entre 32 e 61Rodada n – 12: (X): 31Rodada n – 13: (Y): Entre 16 e 30Rodada n – 14: (X): 15Rodada n – 15: (Y): Entre 8 e 14Rodada n – 16: (X): 7Rodada n – 17: (Y): Entre 4 e 6Rodada n – 18: (X): 3Rodada n – 19: (Y): 2

Logo X = Bernaldo e Y = Arnaldo, e como X ganha, Bernaldo ganha.

Obs: Quando falo números entre r e s, r e s estão incluídos neste intervalo.

PS: Observe as jogadas de Bernaldo: 3, 7, 15, 31, 62, 125, 250, 501, 1002, 2004. Seele escolhe x na Rodada a escolhe 2x ou 2x + 1 na Rodada a + 2. Isso nos leva auma observação que se Bernaldo escolhe x na Rodada a, Arnaldo pode escolherentre x + 1 e 2x – 1 na Rodada a + 1, o que implica que Bernaldo pode escolher 2xou 2x + 1 na Rodada a + 2, SEMPRE. Isso nos leva a pensar no sistema binário denumeração. Ou seja, se na Rodada a o Jogador escolheu o número AnAn –1...A0,na Rodada a + 2 pode escolher os números AnAn–1.....A00 ou AnAn–1...A01. Logo elepode ir “adicionando” algarismos à direita da representação binária do número. Vejaque os dois algarismos à esquerda de um número no sistema binário são 10 ou 11 eque o 1º número do 1º jogador é 2 = (10)2 e o do 2º é 3 = (11)2. Logo se K é onúmero a ser alcançado, se K começar com 10 no sistema binário o 1º ganha e se Kcomeçar com 11, no sistema binário o 2º ganha. Como a representação binária de2004 é 11111010100 que começa com 11 temos que o 2º ganha.

PROBLEMA 5: SOLUÇÃO DE LÚCIO EIJI ASSAOKA HOSSAKA (CURITIBA - PR):a) Primeiro, é necessário dizer que ABC∆ pode ser inscrito em uma circunferência

de raio AD, onde AB é o diâmetro, pois �ACB e reto e deve estar compreendendo um

arco � 180AB = ° , que é uma semi-circunferência. Logo, por D ser centro dessacircunferência, ,AD BD CD= = e os triângulos ACD e BCD são isósceles, tendo ACe BC como base, respectivamente.Logo, a mediatriz de AC contém 1O assim como D (pois D está à mesma distânciade A e C, assim como todos os pontos da mediatriz). Da mesma maneira, a mediatriz

Page 104: Eureka 2005

Sociedade Brasileira de Matemática

EUREKA! N°22, 2005

42

de CB contém 2O e D (lembrando que o circuncentro é o encontro das mediatrizesdos lados do triângulo).A mediatriz de AC é perpendicular a AC, e a mediatriz de CB é perpendicular a CB.Logo, ambas são perpendiculares entre si, pois AC e BC também o são. Comoambas contém D e são retas, elas se interceptam em D, e além disso cada uma

contém um dos circuncentros, o que mostra que �

1 2O DO é reto.

b) Seja E o ponto médio de AD, e F o ponto médio de BD. A mediatriz de AD

contém E e 1O , e portanto �

1O ED é um ângulo reto. Analogamente, �

2DFO é retotambém.Como E, D e F são colineares (AB contém os três), então 1EO e 2FO são segmentos

paralelos entre si. Seja G o ponto médio de 1 2.O O Como 1 2O O D é um triânguloretângulo G é o centro do círculo que deveremos provar que é tangente a AB.Perceba que G é o ponto médio de 1 2O O assim como D é o ponto médio de EF (é

óbvio que ED = DF, pois D é o ponto médio de AB), e que portanto 1

2

.O GED

DF GO= DG

é, portanto, paralelo a 1EO e 2FO , pelo teorema de Tales. Assim, DG é

perpendicular a AB, e como DG é raio do círculo de diâmetro 1 2,O O AB tangenciaesse círculo.

PROBLEMA 6: SOLUÇÃO DE LÚCIO EIJI ASSAOKA HOSSAKA (CURITIBA - PR):Se são 10 linhas e 10 colunas, então há espaço para 20 "presenças" de alguns dos 10algarismos. "Presença" significa que cada vez que um número aparece em algumalinha ou coluna, significa uma "presença". No caso das linhas, por exemplo. Se umalgarismo estiver distribuído dez vezes em duas linhas, são duas "presenças".Assim como se algarismo aparecer apenas 3 vezes em 3 linhas, são 3 "presenças". né maior que 3. Como já disse, cada algarismo ocupa pelo menos 3 linhas ou 3colunas. Se um algarismo ocupar, por exemplo, 3 linhas, ocupará 4 colunas, pois sefossem 3 linhas e 3 colunas, apareceria, no máximo, 9 vezes. E vice-versa. O melhorjeito de distribuir os algarismos é fazer com que cada um tenha o mínimo depresenças. A soma das presenças de cada algarismo é no mínimo 7 (3 linhas e 4colunas, e vice-versa, e 2 linhas e 5 colunas, e vice-versa), sendo 7 × 10 = 70presenças no total. Se são 60 disponíveis (10 linhas e 10 colunas, cada uma podendocomportar 3 algarismos distintos), é impossível criar um tabuleiro com no máximo 3algarismos diferentes em cada coluna e cada linha. portanto, n é maior que 3.Assim, n é 4, pois é possível distribuir os números de forma e que cada linha e cadacoluna tenha até 4 algarismos diferentes:

Page 105: Eureka 2005

Sociedade Brasileira de Matemática

EUREKA! N°22, 2005

43

Assim, existe um arranjo que limita o valor de n, e esse valor é 4.

9 0 9 0 9 0 9 0 9 09 0 9 0 9 0 9 0 9 06 6 5 5 5 7 7 7 8 86 6 5 5 5 7 7 7 8 86 6 5 5 6 8 7 7 8 86 6 5 5 6 8 7 7 8 81 2 2 2 2 3 3 3 3 41 1 2 2 2 3 3 3 4 41 1 1 2 2 3 3 4 4 41 1 1 1 2 3 4 4 4 4

SOLUÇÕES – NÍVEL 3

PROBLEMA 1: SOLUÇÃO DE EDSON AUGUSTO BEZERRA LOPES (FORTALEZA - CE):Parte 1)

A B

D C

O

Se ABCD é um losango, então AB = BC = CD = DA, ou seja, os triângulos ABC,BCD, CDA, DAB são todos isósceles. Veja também que .AC BD⊥ SejaAC BD O∩ = . Como em um triângulo isósceles a altura relativa à base é tambémbissetriz. AO, OB, OC, OD possam pelos incentros de DAB, ABC, BCD e CDArespectivamente.Como ao traçarmos uma perpendicular ao lado pelo incentro obtemos o ponto detoque do incírculo a esse lado, O está contido nos 4 incirculos, já que é o ponto detoque do incírculo na base dos 4 triângulos.

Page 106: Eureka 2005

Sociedade Brasileira de Matemática

EUREKA! N°22, 2005

44

Parte 2)

A B

D C

O

Suponhamos agora que os 4 círculos têm um ponto em comum. Temos então que osincírculos dos triângulos ABC e ACD tocam AC no mesmo ponto, pois do contrárionão teriam nenhum ponto em comum. Seja O esse ponto. Veja que se O estiver nointerior do triângulo ABC o incírculo do triângulo BDC não o conterá, e seguindo omesmo raciocínio vemos que O não está no interior do triângulo BDC. Logo O estásobre .BD AC BD O⇒ ∩ =Sejam 1 2 3, ,O O O e 4O Os incentros dos triângulos DAB, ABC, BCD e CDA,respectivamente. Suponhamos que AC e BD não são perpendiculares. Suponhamos

agora, sem perda de generalidade, que �AOB e �DOC são obtusos e �BOC e �AODsão agudos. Claramente 2O está no interior do triângulo AOB, pois já que

2O O AC⊥ , temos � �

2 .O OC BOC> Com o mesmo raciocínio encontramos que 1O e

2O estão no interior do triângulo AOB e que 3O e 4O estão no interior do triângulo

COD. Seja 1 1 2.G AO BO= ∩

Claramente �

1 2DAB

G AB = e �

� � �

11 1802 2

ABC DAB ABCG BA AG B

+= ⇒ = °−

� � � � � � �

1 1180 ( ) 180 2(180 ) 180 ,OAG OBG AOB AOB DAB ABC AOB= °− + + < ° − ⇒ + < °− < °

pois � 90 .AOB > ° De modo análogo temos� � 180BCD CDA+ < ° � � � � 360 360 360DAB ABC BCD CDA⇒ + + + < ° ⇒ ° < ° . Absurdo!

Assim, ,AC BD⊥ donde 1 2 3 4, , , , ,O AO O BO O CO O DO AO BO CO∈ ∈ ∈ ∈ ⇒ e DOsão, além de alturas, bissetrizes , , ,ABC BCD CDA DAB⇒ são isósceles

AB BC CD DA⇒ = = =ABCD⇒ é um losango.

Page 107: Eureka 2005

Sociedade Brasileira de Matemática

EUREKA! N°22, 2005

45

PROBLEMA 2: SOLUÇÃO DE LARISSA CAVALCANTE QUEIROZ DE LIMA (FORTALEZA - CE):Seja N tal que o triângulo está dividido em N triângulos de modo que não há 3vértices colineares e em cada vértice incida exatamente k segmentos.Seja M o total de vértices (incluindo os vértices do triângulo dividido ABC).Contando a soma dos ângulos de todos os triângulos temos:

� � �

soma dos ângulospois há N 's soma dos ângulos de cadavértice do interior de

180 360 ( 3) 180

A B CABC

N M∆

+ +

° ⋅ = ° − + °����� �������

2( 3) 1 2 6 1 2 5

5

2

N M M M

NM

⇒ = − + = − + = −+⇒ =

Contando o total de segmentos:

há vértices degrau cada

3 3

2 2m

k

M k N⋅ ⋅ + =

há N triângulos e cada segmento é lado de dois triângulos, exceto os três segmentos: AB, BC, CA.

3 3 3 3 6 63 3

( 5) / 2 5

N N NM k N k

M N N

+ + +⇒ ⋅ = + ⇒ = = = ∈+ +

6( 5) 24 246 .

5 5 5

Nk

N N N

+⇒ = − = −+ + +

como k é inteiro, N + 5 deve ser um divisor de 24.Assim, 5 6,8,12N + = ou 24 (pois N é inteiro positivo)

1,3,7N⇒ = ou 19.

De fato, para cada um desses valores, há uma divisão do triângulo:

N = 1: N = 3 :

N = 7: N = 19:

Page 108: Eureka 2005

Sociedade Brasileira de Matemática

EUREKA! N°22, 2005

46

Observação: as duas contagens feitas pela Larissa, se generalizadas, levam a umademonstração do Teorema de Euler: sendo V, A, e F o número de vértices, arestas efaces, respectivamente, de um poliedro (ou se você quiser, um grafo plano), entãoV – A + F = 2. No problema 2, F = n + 1, V = M e A é o número de segmentos.Em contrapartida, também é possível resolver este problema usando o Teorema deEuler.

PROBLEMA 3: SOLUÇÃO DE FÁBIO DIAS MOREIRA (RIO DE JANEIRO - RJ):Sim. Se tomarmos a suficientemente grande e 1 2 3( , , ) ( , , ),x x x a a a= − é trivial ver que

cada ix é um polinômio em a. Em particular, como a seqüência possui um númerofinito de termos, podemos tomar a suficientemente grande de tal forma que cadatermo ix da seqüência tenha o sinal do coeficiente líder do polinômio.Mas é fácil ver que a seqüência dos termos líderes é:

2 2 3 4 5 7 9 12( , , , , 2 , ,2 ,2 , 2 ,4 , 4 ,...)a a a a a a a a a a a− − − − − − e é fácil provar, por indução que

dois termos líderes nunca se cancelam (basta notar que a partir de 124a− , osexponentes dos dois termos anteriores são sempre maiores que metade do expoente

do termo atual: isso é verdade para 124a− , e se os expoentes são a, b e c, com 2

ca >

e ,2

cb a b c> < < (já que a seqüência dos expoentes é crescente), então

2

a bb

+> pois

b > a e 2

a bc

+> pois c > a e c > b). Mas a seqüência dos sinais dos termos líderes é

claramente periódica de período 7:

( , , , , , , , , , , , , , , , , ,...)+ − + − − − + + − + − − − + + − +

Logo a seqüência definida com o a supracitado e 1 2 3, ,x a x a x a= = − = tem pelo

menos 4

2004 10027

⋅ > termos negativos.

PROBLEMA 4:Veja a solução do problema No. 6 do nível 2.

Page 109: Eureka 2005

Sociedade Brasileira de Matemática

EUREKA! N°22, 2005

47

PROBLEMA 5: SOLUÇÃO DE FÁBIO DIAS MOREIRA (RIO DE JANEIRO - RJ):Vamos demonstrar que na é sempre inteiro por indução: isto é evidentemente

verdadeiro para n = 0, 1, 2, 3, 4, 5, 6, 7 (pois 4 5 6 72, 3, 7 e 23)a a a a= = = = .

Suponha então que 8n ≥ , e 0 1 2 1, , ,..., na a a a − são todos inteiros.

Lema: Se 0 ,k n< < então 1( , ) 1.k ka a − =Prova: O resultado é evidentemente verdadeiro para k =1.Suponha-o verdadeiro para k – 1. Então

22 21 3 2

1 1 1 3 2 1 2 14

( , ) , ( , ) ( , ) 1,k k kk k k k k k k k k

k

a a aa a a a a a a a a

a− − −

− − − − − − − −−

+= ≤ + = =

pois

1 2( , ) 1.k ka a− − = Logo o resultado é válido para todo 0 .k n< <Corolário: 21 ( , ) 1k kk n a a −< < ⇒ = e 32 ( , ) 1.k kk n a a −< < ⇒ =Prova: Basta notar que

22 1 3 2 2 1 3 2 1 2 3 2( , ) ( , ) ( , ) ( , ) ( , ) 1 1 1k k k k k k k k k k k k ka a a a a a a a a a a a a− − − − − − − − − − − −≤ + = ≤ ⋅ = ⋅ =

e 3( , )k ka a −2 2 2 2

1 3 2 3 2 3 2 3( , ) ( , ) ( , ) 1 1.k k k k k k k ka a a a a a a a− − − − − − − −≤ + = ≤ = =Para provar que na é inteiro, basta provar que 2

4 1 3 2| .n n n na a a a− − − −+Em outras palavras, temos que demonstrar que

23 1 2 0n n na a a− − −+ ≡ (mod 4na − ) ⇔

22 2 26 4 5 4 2 3 5 3 4

7 5 6

0n n n n n n n n n

n n n

a a a a a a a a a

a a a− − − − − − − − −

− − −

+ ⋅ + +⋅ + ≡

(mod 4na − ) ⇔

2 2 2 2 26 6 4 5 4 2 3 5 7 5 3 4

25 6 7

( )( ) ( )0n n n n n n n n n n n n

n n n

a a a a a a a a a a a a

a a a− − − − − − − − − − − −

− − −

+ + + +≡ (mod 4na − )

Pelo Lema, podemos multiplicar por 25 6 7n n na a a− − − e manter a equivalência, logo basta

provar que2 2 2 2 2

6 6 4 5 4 2 3 5 7 5 3 4 4( )( ) ( ) 0(mod )n n n n n n n n n n n n na a a a a a a a a a a a a− − − − − − − − − − − − −+ + + + ≡2 2 2

5 3 6 7 5 4( ) 0(mod )n n n n n na a a a a a− − − − − −⇔ + ≡ ⇔2 2

5 3 4 8 40(mod ) 0 0n n n n na a a a a− − − − −⇔ ≡ ⇔ ≡ (mod 4na − ).

PROBLEMA 6: SOLUÇÃO DE GABRIEL TAVARES BUJOKAS (SÃO PAULO - SP):

Resposta: 2( 1)

.4

b− +

Seja k o tamanho do ciclo, 1 0( , ),P x x= , 1( ) ( , ),ra b r rf P x x+= para todo r ∈ � .

Page 110: Eureka 2005

Sociedade Brasileira de Matemática

EUREKA! N°22, 2005

48

Logo

20 2 1

21 1 0

22 0 1

21 3 2

k k

k

k k k

x a bx x

x a bx x

x a bx x

x a bx x

− −

− − −

= − −

= − −

= − −

= − −�

21 2 1 2 Sendo , , temos ( 1)i i aS x S x S b S k⇒ = = + + =∑ ∑

22 1 1

2i

MQ MA

xS S SS

k k k k≥

≥ ≥ ⇒ ≥∑���������

Logo:2 21

1( 1) ( 1)S x

ka S b x b kak k

≥ + + ⇒ + + − tem raiz 2

2 ( 1)( 1) 4 0 .

4

bb a a

− +⇒∆= + + ≥ ⇒ ≥

Para 2

,

( 1) ( 1) ( 1) ( 1) ( 1): ; ;

4 2 2 2 2a b

b b b b ba f

− + + + + + = − − = − − ⇒ per ( , ) .a b ≠∅

Page 111: Eureka 2005

Sociedade Brasileira de Matemática

EUREKA! N°22, 2005

49

XXVI OLIMPÍADA BRASILEIRA DE MATEMÁTICAProblemas e Soluções da Primeira Fase – Nível Universitário

PROBLEMA 1:

Considere a matriz complexa

1 0

0 0 0 .

0 1

i

A

i

=

Calcule 2004.A

PROBLEMA 2:

Calcule a integral: 20041

11 x

xdx

e− +∫

PROBLEMA 3:Determine a equação da reta que tangencia a curva de equação 4 33 4y x x= − emdois pontos distintos.

PROBLEMA 4:Quantas triplas ordenadas (A, B, C) de subconjuntos de {1, 2, ..., }n existem para as

quais ; ; ?A B C A B A C∩ ∩ = ∅ ∩ ≠ ∅ ∩ ≠ ∅

PROBLEMA 5:Considere a matriz A n × n definida por ( 1)ija n i j= − + , para todos 1 , .i j n≤ ≤As interseções de k linhas e k colunas quaisquer de A determinam uma submatriz deordem k de A. Seja ( )nϕ a soma dos determinantes de todas as submatrizes de A.

a) Determine λ real de forma que ( )

limn

n

nλϕ

→+∞ exista e seja não nulo.

b) Determine o valor do limite acima para o valor de λ encontrado.

PROBLEMA 6:

Calcule 0

1

(3 1)(3 2)(3 3)k k k k

= + + +∑ .

Page 112: Eureka 2005

Sociedade Brasileira de Matemática

EUREKA! N°22, 2005

50

SOLUÇÕES – PRIMEIRA FASE – NÍVEL UNIVERSITÁRIO

SOLUÇÃO DO PROBLEMA 1:

Temos 4

4 0 0

0 0 0

0 0 4

A

− = −

, donde 4

( 4) 0 0

0 0 0

0 0 ( 4)

k

k

k

A

− = −

.

Em particular,

1002

2004

1002

2 0 0

0 0 0

0 0 2

A

− = −

.

Soluções cujo único erro se refere aos sinais:

SEGUNDA SOLUÇÃO DO PROBLEMA 1:

Diagonalizando A, temos 1A XDX −= onde

1 0 1

0 1 0

1 0 1

X

= −

,

1 0 0

0 0 0

0 0 1

i

D

i

+ = −

Assim,2004 1002 1002

2004 1 1

2004 1002 1002

(1 ) 0 0 2 0 0 2 0 0

0 0 0 0 0 0 0 0 0 .

0 0 (1 ) 0 0 2 0 0 2

i

A X X X X

i

− −

+ − − = = = − − −

SOLUÇÃO DO PROBLEMA 2:

Por substituição, 2004 20041 1

1 11 1x x

x xdx dx

e e−− −=

+ +∫ ∫ .

Assim2004 2004 20041 1 1 1 2004

1 1 1 1

1 1 1 1

1 2 1 1 2 1 1x x x x x

x x xdx dx dx x dx

e e e e e− −− − − −

= + = + + + + + + ∫ ∫ ∫ ∫1 12004 2004

1 0

1 1.

2 2005x dx x dx

−= = =∫ ∫

SOLUÇÃO DO PROBLEMA 3:Queremos encontrar a e b tais que, 4 33 4P x x ax b= − − − tenha duas raízes reaisduplas; em particular, P deve ser um quadrado perfeito.

Page 113: Eureka 2005

Sociedade Brasileira de Matemática

EUREKA! N°22, 2005

51

4 3 2 2

2 4 3 2 2 2

3 4 ( )

2 (2 ) 2

x x ax b cx dx e

c x cdx ce d x dex e

− − − = + +

= + + + + +

implica em 3, 2 ,3

cc d= ± = − 2

9

ce = − donde

8 4, .

9 27a b

− −= =

Assim a reta tem equação 8 4

.9 27

y x= − −

SOLUÇÃO DO PROBLEMA 4:

Vamos contar inicialmente o número de escolhas tais que A B C∩ ∩ = ∅ .

S1 S1 S2

S5 S6

S3

A B

C

Cada elemento de {1,2,…,n} pode ser colocado em um dos 7 subconjuntos indicadosacima

(S1, S2, …,S7). Logo há 7n tais escolhas.

Fazendo um raciocínio similar, temos, dentre essas escolhas, 6n com A B∩ = ∅ ;6n com A C∩ = ∅ e 5n com A B∩ = ∅ e .A C∩ = ∅

Portanto, pelo princípio de Inclusão-Exclusão, há 7 2 6 5n n n− ⋅ + maneiras deescolher A, B, C.

SOLUÇÃO ALTERNATIVA DO PROBLEMA 4:

Fixe o número k de elementos de BA ∩ (1 � k � n), e j de CA ∩ (1 � j� n – k).

Page 114: Eureka 2005

Sociedade Brasileira de Matemática

EUREKA! N°22, 2005

52

k

n modos de escolher esses k elementos,

−j

kn modos de escolher os j e 5

regiões permitidas para cada um dos outros n – k – j elementos, de forma que aresposta é:

R = ∑ ∑∑ ∑=

=

−−

=

=

−−

−⋅

=

n

k

kn

j

jknn

k

kn

j

jkn

j

kn

k

n

j

kn

k

n

1 11 1

55

Utilizando o binômio de Newton, tem-se

knknjknkn

j j

kn −−−−−

==+=

−∑ 6)51(50

.

Portanto,

nnnnnnnn

k

knn

k

kn

k

n

k

nR 5627)56(6756

11

+⋅−=−−−=

= ∑∑

=

=

− .

SOLUÇÃO DO PROBLEMA 5:Se 3k ≥ , qualquer submatriz de ordem k de A determinada por k linhas e k colunastem determinante O. De fato, se 1 2 3j j j< < são índices de 3 das k colunas da

submatriz, denotando essas colunas por 1 2 3, e ,j j jC C C temos que todas as entradas de

2 1j jC C− são iguais a 2 1j j− e todas as entradas de 3 2j jC C− são iguais a 3 2 ,j j−

donde 3 2 2 1

3 2

2 1

( ),j j j j

j jC C C C

j j

−− = ⋅ −

− e logo

3 2 1

3 1 3 2

2 1 2 1

.j j j

j j j jC C C

j j j j

− −= ⋅ − ⋅

− −

Assim, ( )nϕ é a soma dos determinantes das submatrizes de A de ordem 1 ou 2. A

soma dos determinantes das submatrizes de A de ordem 1 é 2 2 2

1

( 1).

2

n

m

n nm

=

+=∑

As submatrizes de ordem 2 são obtidas da seguintes forma:Dados 1 a b n≤ < ≤ e 1 c d n≤ < ≤ , associamos a seguinte submatriz de ordem 2:

( 1) ( 1),

( 1) ( 1)

n a c n a d

n b c n b d

− + − + − + − +

cujo determinante é

(( 1) ( 1) ( 1) ( 1) ) ( )( ).n a d b c b d a c n a b d c− + − − − − − = − − Assim, a soma dosdeterminantes das submatrizes de ordem 2 é

Page 115: Eureka 2005

Sociedade Brasileira de Matemática

EUREKA! N°22, 2005

53

2 2 21

1 1 1 1 1 1

( 1)( ) ( ) ( )

2

jn n

a b n c d n i j n j r j

j jn a b d c n j i n r n

≤ < ≤ ≤ < ≤ ≤ < ≤ = = =

− − − = − − = − = − =

∑ ∑ ∑ ∑∑ ∑23

( ) ,4 3

n ng n

= − +

onde

3

( )lim 0n

g n

n→∞= (de fato, ( )g n é um polinômio de grau 2).

Assim, como 2 2

7

1 ( 1)lim 0

2n

n n

n→∞

+ =

e

2 23

7 3

1 1 1 ( ) 1lim ( ) lim ,

4 3 4 3 36n n

n n g ng n

n n→∞ →∞

⋅ − + = − ⋅ + = −

Temos 7λ = e 7

( ) 1lim .

36n

n

n

ϕ→∞

= −

SOLUÇÃO DO PROBLEMA 6:

Decompondo em frações parciais, procuramos constantes A, B e C tais que:

332313)33)(23)(13(

1

++

++

+=

+++ k

C

k

B

k

A

kkk (1)

Comparando os numeradores, verifica-se que (1) é identidade para

2

1,1,

2

1 =−== CBA .

Sendo S a soma procurada: ∑∞

= ++

+−

+=

0 33

1

23

2

13

12

k kkkS .

Como ∫=+

1

0

3

13

1dxx

kk , tem-se ( )∑∫

=

++ +−=0

1

0

23133 22k

kkk dxxxxS .

Trocando a integral com o somatório e somando a PG infinita:

∫ ∫∑∫ ∑ −−=

⋅+−=

+−=

=

=

++1

0

1

03

2

0

321

0 0

23133

1

)1()21(22 dx

x

xdxxxxdxxxxS

k

k

k

kkk

Page 116: Eureka 2005

Sociedade Brasileira de Matemática

EUREKA! N°22, 2005

54

1 1

2 20 0

1 1

22 20 0

1 1 (2 1) 32

1 2 1

1 2 1 3

2 1 2 1 32 2

x xS dx dx

x x x x

x dxdx

x xx

− − + += =+ + + +

+= − ++ + + +

∫ ∫

∫ ∫

1

2

0

11 3 2 12ln( 1) arctan ln3 3 ( )2 2 2 3 63 3

2

xx x

π π+= − + + + ⋅ ⋅ = − + ⋅ −

Logo, 3 3ln 3

12S

π −=

Page 117: Eureka 2005

Sociedade Brasileira de Matemática

EUREKA! N°22, 2005

55

XXVI OLIMPÍADA BRASILEIRA DE MATEMÁTICAProblemas e Soluções da Segunda Fase – Nível Universitário

PROBLEMA 1:A função derivável :f → tem as seguintes propriedades:

a) (0) 0f = e (2) 2.f =b) Para todo \{0},a ∈ a reta tangente ao gráfico de f no ponto ( , ( ))P a f a=

corta o eixo x em um ponto A e o eixo y em um ponto B de tal forma que A é oponto médio do segmento BP.

Calcule f (3).

PROBLEMA 2:

Prove que não existe um conjunto 2A ⊆ tal que:

• Para todo x ∈ , { ( , ) }y x y A∈ ∈ é finito.

• Para todo y ∈ , { ( , ) }x x y A∈ ∉ é enumerável.

Obs: Um conjunto A é dito enumerável se A = ∅ ou existe uma função sobrejetiva: .f A→�

PROBLEMA 3:

Seja A uma matriz real inversível de ordem n e tA a sua transposta. Sejam

1 2 ... 0nλ λ λ≥ ≥ ≥ > os autovalores de .tA A Definimos a norma de A por 1A λ= e

o fator de dilatação de A por 1

2( ) .d A λ

λ= Prove que, para quaisquer matrizes

reais inversíveis A e B, ( ) ( )AB

d AB d BA B

≥ ⋅⋅

.

PROBLEMA 4:

Seja 1 2 1 2{( , ,..., ); , ,..., }.nn nx x x x x x= ∈� �

Seja p um primo, k um inteiro positivo e 1 2, ,..., , nkP P P Q ∈ � tais que para todo j,

1 , .j nP Q

j kp

−≤ ≤ ∉ �

Page 118: Eureka 2005

Sociedade Brasileira de Matemática

EUREKA! N°22, 2005

56

Prove que existe um polinômio 1( ,..., )nf x x com coeficientes inteiros com ( ) 0jf P =

para todo j, 1 ,j k≤ ≤ e ( )

.f Q

p∉ �

PROBLEMA 5:Seja 2m ≥ um inteiro.Arnaldo e Bernaldo disputam o seguinte jogo: cada jogador recebe, alternadamente,

um número kN e devolve para o outro jogador ou 1 1k kN N+ = − ou 1 .kk

NN m+ =

Arnaldo começa recebendo um número inteiro positivo 0.N Quem devolver zerovence o jogo.Seja (resp. )n nA B o conjunto dos valores de 0 0,N N n< , tais que Arnaldo (resp.Bernaldo) tem estratégia vencedora.

Calcule lim n

nn

A

B→∞ em função de m.

PROBLEMA 6:Seja 2:[0,2 ]γ π → uma função derivável, com derivada contínua, com

'( ) 1tγ = para todo t e cuja imagem é uma curva simples fechada, isto é,

0 1 0 1 0 1( ) ( ), 0, 2t t t t t tγ γ π= < ⇔ = = .

Prove que existem 0 10 2t t π≤ < < tais que

0 1 1 0 0 1

2( ) ( ) min { ,2 }t t t t t tγ γ π

π− ≤ − + − .

Page 119: Eureka 2005

Sociedade Brasileira de Matemática

EUREKA! N°22, 2005

57

PROBLEMA 1: SOLUÇÃO DE LEONARDO AUGUSTO ZÃO (NILÓPOLIS – RJ)De acordo com o item (b) do enunciado, ao traçarmos a reta tangente ao gráfico noponto (a, f(a)), com a ≠ 0, obtemos:

A = (xA, 0)

P = (a, f(a))

B = (0, yB)

α

f (x)

x

Reta tangente ao gráfico de f em (a, f(a))

Como A é ponto médio de BP, então: 2

B PA

+=

Assim, 2A

ax = e ( )By f a= − .

Portanto, o coeficiente angular da reta é dado por ( ( ) ) 2 ( )

( / 2)Bf a y f a

tga a a

−= =−

α

Assim, 2 ( )

'( ) , *.f a

f a aa

= ∀ ∈

Então chegamos à equação diferencial: 2 ( )

'( ) , *f x

f x xx

= ∈

Resolvendo, temos: [ ] [ ] *'( ) 2 ( ) ' 2 ',

( )

f xln f x ln x x

f x x += ⇒ = ∈

Então 2 2 * ( ) 2 ( ) ( ) , .k kln f x ln x k ln f x ln x e f x e x x += + ⇒ = ⋅ ⇒ = ⋅ ∀ ∈

Para 2, ( ) 2.x f x= = Então 2 1(2) 2 .

2k kf e e= ⋅ ⇒ =

Assim, a função f obedece 21( ) ,

2f x x= para *x +∈ .

Então, 21(3) 3 ,

2f = ⋅ e finalmente,

9(3) .

2f =

Page 120: Eureka 2005

Sociedade Brasileira de Matemática

EUREKA! N°22, 2005

58

PROBLEMA 2: SOLUÇÃO DE DAVI MÁXIMO ALEXANDRINO NOGUEIRA (FORTALEZA – CE)Sejam

{ | ( , ) }yA x x y A= ∈ ∈

{ | ( , ) }xB y x y A= ∈ ∈

Logo xB é finito, x∀ ∈ e yA− é enumerável, y∀ ∈ .

Agora sejam 1 2 3 ...,y y y< < < uma seqüência crescente infinita em .

Logo, 1 1

( ) ,i iy y

i i

A A∞ ∞

= =

= − − ≠ ∅ � � pois, do contrário, teríamos

1

( ),iy

i

A∞

=

= − � mas cada iyA− é enumerável

1

( )iy

i

A∞

=

⇒ −� enumerável

⇒ enumerável, absurdo!

Agora, tome 1 21

. Temos { , ,...,} ,i xi

x Ay y y B∞

=

∈ ⊂� absurdo! (pois xB é finito). Logo A

não existe.

PROBLEMA 3: SOLUÇÃO DE ALEX CORRÊA ABREU (NITERÓI – RJ)

Sejam 0...;0...;0... 212121 >≥≥≥>≥≥≥>≥≥≥ nnn αααβββλλλ os

autovalores de ( ) , t tAB AB B B e tA A respectivamente. A desigualdade pedida é

claramente equivalente a 212 βαλ ⋅≤ .

Como XTX é sempre simétrica e >>=<< XuXuuXuX T ,, , temos pelo teorema domin-max (que provaremos no fim desta solução) que:

2 dim 1 dim 1 \{0}

, , ,min max min max .

, , ,S n S nx S x S

ABx ABx ABx ABx Bx Bx

x x Bx Bx x x= − = −∈ ∈

< > < > < > λ = = ⋅ < > < > < > Por outro lado, como só há um subespaço de dimensão n (e ele inclui todos osvetores), temos:

1\{0}

,max

,nx

Ax Ax

x x∈

< >α =< >�

, e 2 dim 1 \{0}

,min max

,S n x S

Bx Bx

x x= − ∈

< > β = < > .

Seja S o subespaço no qual 2β é atingido, i.e, 2\{0}

,max

,x S

Bx Bx

x x∈

< >β =< >

.

Temos então:

2\{0} \{0} \{0}

, , , ,max max max

, , , ,x S x S x S

ABx ABx Bx Bx ABx ABx Bx Bx

Bx Bx x x Bx Bx x x∈ ∈ ∈

< > < > < > < > λ ≤ ⋅ ≤ ⋅ < > < > < > < > ,

Page 121: Eureka 2005

Sociedade Brasileira de Matemática

EUREKA! N°22, 2005

59

(usamos respectivamente que o mínimo é � ��� �������� ������ �� ���� �

vuuv maxmax)max( ⋅≤ )

O segundo termo do produto é exatamente 2β . Fazendo Bxx =' e notando que

( ) nB S ⊂ , tem-se:

1\{0} ' \{0}

, ', 'max max

, ', 'nx S x

ABx ABx Ax Ax

Bx Bx x x∈ ∈

< > < >≤ α< > < >�

, e juntando tudo obtemos 212 βαλ ⋅≤

Lema: (Teorema do Min Max ): 1 1max , ,

xAx x

=λ = < >

2 dim 1 1

min max ,S n x S

x

Ax x= − ∈

=

λ = < > , para toda A matriz simétrica, onde 1 2 ... nλ ≥ λ ≥ ≥ λ são

seus autovalores.

Prova: Como A é simétrica existe base ortonormal 1,..., nx x tal que i i iAx x= λ logo,

se 1 1 ... ,n nx a x a x= + + 2 2 2 2 21 1 2 2 1 1 1, ... ( ... )n n nAx x a a a a a< > = λ + λ + + λ ≤ λ + + = λ ⇒

1max , Ax x< > ≤ λ , mas, tomando 1 1 1 1, , x x Ax x= < > = λ . Para o 2λ , seja L

subespaço gerado por 1x e 2.x

Assim, 1 1

dim 1 max , max ,x S x L Sx x

S n Ax x Ax x∈ ∈ ∩= =

= − ⇒ < > ≥ < > , mas se 1 1 2 2y a x a x= + L S∈ ∩ ,

2 2 2 2 2 21 2 1 1 2 2 2 1 2 2

1

1 max , , ( )x L S

x

a a Ax x Ay y a a a a∈ ∩

=

+ = ⇒ < > ≥ < > = λ +λ ≥ λ + = λ ⇒

2dim 1min max ,

S n x SAx x

= − ∈< > ≥ λ . Agora, tomando 1( )S x ⊥= temos que o mínimo é

atingido.

PROBLEMA 4: SOLUÇÃO DE YURI GOMES LIMA (FORTALEZA - CE)Seja 1( ,..., ).nQ b b= Sejam também 1( ,..., ), , 1 , 1 .j j jn jP a a a j k n= ∈ ≤ ≤ ≤ ≤

�� �

Observe que 1 1 ,...,j j jn nn nP Q a b a b

p p p

− − − ∉ ⇔ ∉ ⇔

� � existe {1,2,..., }ji n∈ tal

que |j jji ia bp

p

−∉ ⇔� ( )

j jji ia b−

Tome então

11

( ,..., ) ( )j j

k

n i jij

f x x x a=

= −∏Note que:

Page 122: Eureka 2005

Sociedade Brasileira de Matemática

EUREKA! N°22, 2005

60

1 11

( ,..., ) ( ) ( ) ( ) ( ,..., )j j i j i j

k

n i ji i j i j j n

j

f x x x a x a x a R x x=≠

= − − = − ⇒∏� �

1( ) ( ,..., ) ( ) ( ,..., ) 0 ( ) 0j jj j jn ji ji j j jn jf P f a a a a R a a f P⇒ = = − = ⇒ =

Ademais, 11

( ) ( ,..., ) ( ).j

k

n ij jij

f Q f b b b a=

= = −∏ Como |p ( ), ,jij jib a j− ∀ e p é primo

segue que |p( )

( )f Q

f Qp

⇒ ∉ � (c.q.d.)

PROBLEMA 5: SOLUÇÃO OFICIALConsideremos a seqüência 0 1 2, , ,...a a a com {0,1},na n∈ ∀ ∈ � tal que 1na = se

Arnaldo tem estratégia vencedora recebendo 0N n= e 0,na = caso contrário, i.e.,

caso Bernaldo tenha estratégia vencedora se Arnaldo recebe 0N n= . É fácil ver que,

para 1,k ≥ se 0,ka = 1km ra + = para 0 1r m≤ ≤ − (nesse caso km r

km+ =

), e, se

1ka = caso 1 1,kma − = temos 0, se é par1 ( 1)

1, se é ímpar2

r

km r

ra

r+− −

= =

para 0 1,r m≤ ≤ − e,

caso 1 0,kma − = temos, para 1, se é par1 ( 1)

0 1,0, se é ímpar2

r

km r

rr m a

r++ −

≤ ≤ − = =

.

Suponhamos inicialmente que m é par. Assim, pela discussão acima, se

1,ka = para 2

m valores de r, com 0 1,r m≤ ≤ − temos 0,km ra + = e para os demais

2

m

valores de r nesse intervalo temos 1.km ra + = Assim, se {0 0}n kB k n a= ≤ < = e

{0 1},n kA k n a= ≤ < = temos ,n nB A n+ = 12mn n

mB V= ⋅ + e

1, 1,2mn n n

mA m U V n= ⋅ + ⋅ − ∀ ≥ donde, como ,n nA n B= − temos

( ) 1,2mn n

mB n B= ⋅ − + e logo

1 11 .

2mn nB B

mn n mn

= ⋅ − +

Como a solução de

( )11

2t t= − é

1,

3t = fazemos

1,

3n

n

B

n= +δ e obtemos

1 1.

2mn n mn= − +δ δ De

( 1)mn mn r mnB B B m+≤ ≤ + − para 0 1,r m≤ ≤ − obtemos 1

,( 1)

mn mn r mnB B B m

m n mn r mn+ + −

≤ ≤+ +

Page 123: Eureka 2005

Sociedade Brasileira de Matemática

EUREKA! N°22, 2005

61

donde, para 0 1,r m≤ ≤ − 1,

1mn r mn

m

n mn r+ − < <+ +

δ δ desde que 1.n m≥ − Assim,

se / /

2 1( 1), ,

2n m N m N m

m mN m m

N N ≥ − ≤ + ⋅ ≤ +δ δ δ de onde é fácil provar por

indução em k que, se 2k ≥ e 1,k km n m +≤ < temos 2/ 2 ,kn k −≤δ donde lim 0.n

n→∞=δ

Portanto, se m é par 1

lim ,3

n

n

B

n→∞= donde

2lim lim 1 lim ,

3n n n

n n n

A n B B

n n n→∞ →∞ →∞

−= = − = e

2 3lim lim 2.

1 3n n

n nn n

A A n

B B n→∞ →∞= = =

Suponha agora m ímpar. Para cada seqüência finita α de 0's e 1's denotaremos pornα a seqüência

vezes

...n���αα α obtida pela concatenação de n cópias da seqüências α .

No que se segue, usaremos o termo bloco para denotar uma seqüência do tipo 1 0r ,com 1.r ≥ Veremos agora que 1( )n n ma ≥ + é obtida por uma concatenação de blocos

dos tipos 10, 1 0m e 11 0.m+

A seqüência 1 2 ( 1)...m m m ma a a+ + + é ( 1)

1 12(10) 1 0.m m

m− − +⋅ Vamos mostrar por indução que,

para todo 2,k ≥ a seqüência 1 11 1

, 11 1 1

k k k

k j

m m m ma j

m m m

+ + − − −= ≤ ≤ = − − − − β é obtida

por uma concatenação de blocos dos tipos 10 e 1 0m , seguida por um único bloco de

tipo 11 0m+ (note que 2 1

11

mm

m

− = +−

e 3

( 1),1

m mm m

m

− = +−

e logo já verificamos o

caso k = 2).

Se 1 0,ja − = 1j ta + = para 0 1t r≤ ≤ − e 0,j ra + = temos ( ) ( 1) 1m j r s m j sa a+ + + += = para

0 1,s m≤ ≤ − e, para 0, se é par1 ( 1)

0 1,1, se é ímpar2

s

mj s

ss mr a

s+− −≤ ≤ − = =

. Assim, se

r = 1 ou r = m, 0 1( )mj s s mra + ≤ ≤ − começa e termina por 0, e, se r = m + 1, 0 1( )mj s s mra + ≤ ≤ −

começa por 0 e termina por 1. Logo, blocos dos tipos 10 e 1 0m geram apenas blocosdesses mesmos tipos, e blocos do tipo 11m+ só podem ser gerados por blocos do tipo

11 0m+ (eles aparecem no fim da seqüência gerada por 11 0m+ ). Essas observaçõesprovam a nossa afirmação.

Page 124: Eureka 2005

Sociedade Brasileira de Matemática

EUREKA! N°22, 2005

62

Como os únicos blocos diferentes desses que aparecem são os do tipo 11 0m+ de

tamanho total m + 2, os quais só aparecem entre as posições 1

1

km mm

m

+ − −−

e

1

,1

km m

m

+ −−

2,k ≥ e como 1

,1

kkm m

m mm

+ − > +−

temos

log ( 2) log2 ( 1) ( 1) ( 2) 1 ,

log logn n

n m nu m v n m m n

m m

++ + ≥ − + − + ⋅ − ≥ −

donde

2 ( 1)lim 1.n n

n

u m v

n→∞

+ += Além disso, pela discussão acima, seqüências dos tipos 10 e

1 0m geram seqüências desses mesmos tipos, donde21 1

2 2mn n n n

m mu u v r

− − = + + e ,mn n n nv u v s= + + para 1,n m≥ + onde

2 log

logn

m nr

m≤ e

log

logn

ns

m≤ os desvios nr e ns se devem a blocos gerados por

blocos incompletos e pelos blocos do tipo 11 0,m+ que são no máximo log

log

n

m em

( ) ).k k na ≤ Como vimos acima, 2 ( 1) ,n n nu m v n w+ + = − onde ( 2) log

.logn

m nw

m

+≤

Sejam nn

ux

n= e .n

n

vy m

n= Temos ,n n n n n nu mv A u mv w+ ≤ ≤ + + donde

log,n

n n

A nx y O

n n

+ = + e, de 2 ( 1) ,n n nu m v n w+ + = − segue que

1 log2 1 .n n

m nx y O

m n

+ + ⋅ = −

De mn n n nV u v s= + + e de 1N Nm

m

v v

− ≤ segue que (log ),N N N

m m

v u v O N

= + +

donde 1 log 1 1 1 log

12N N N N N

m m m m

N m Ny x y O y y O

m N m m N

+ = + + = − ⋅ + + =

Page 125: Eureka 2005

Sociedade Brasileira de Matemática

EUREKA! N°22, 2005

63

1 1 log.

2 2 Nm

m Ny O

m N

− = − + Escrevendo

3 1n n

my

m= +

−ε (note que a solução de

1 1

2 2

mt t

m

−= − ⋅ é )3 1

mt

m=

−, temos

1 log,

2N N

m

m NO

m N

− = − ⋅ + ε ε donde

0 limsup 1nn→∞

≤ ≤ε e 1

limsup limsup 0,2n n

n n

m

m→∞ →∞

−≤ ⋅ +ε ε e logo limsup 0,nn→∞

=ε ou

seja, lim 0nn→∞

=ε , donde lim3 1n

n

my

m→∞=

− e, de

1 1 log1 ,

2n n

m nx y O

m n

+ = − ⋅ +

segue que 1 1 1

lim 1 .2 3 1 3 1nn

m m mx

m m m→∞

+ − = − ⋅ = − − De

log,n

n n

A nx y O

n n

+ = +

segue que ( ) 2 1lim lim ,

3 1n

n nn n

A mx y

n m→∞ →∞

−= + =−

e lim 1 lim3 1

n n

n n

B A m

n n m→∞ →∞= − =

−.

Portanto, se m é ímpar, (2 1) (3 1) 2 1

lim lim .(3 1)

n n

n nn n

A A n m m m

B B n m m m→∞ →∞

− − −= = =−

PROBLEMA 6: SOLUÇÃO OFICIAL(do Prof. Alexandre Fernandes, comunicada pelo Prof. Lev Birbrair, de Fortaleza, CE)Suponha por absurdo que exista um contra-exemplo .γ Defina 2:[0,2 ] →α π por

( ) ( ) ( ),t t t= − +α γ γ π para 0 t≤ ≤π e ( ) ( ) ( )t t t= − −α γ γ π para 2 .t≤ ≤π πPela hipótese de absurdo temos ( ) 2t >α para todo t. A função α é contínua (pois

(0) (2 ))=γ γ π e derivável exceto talvez para 0,t = π .

Temos ainda '( ) '( ) '( ) 2t t t≤ + + =α γ γ π para todo t, donde o comprimento da

curva α é menor ou igual a 4 .πMas a imagem de α é uma curva fechada simétrica em relação à origem (pois

( ) ( ), [0, ]),t t t+ = − ∀ ∈α π α π que está no exterior da bola de raio 2, donde seucomprimento é maior que 4 ,π absurdo.

α(π) = – α (0)

α (0)

2

Page 126: Eureka 2005

Sociedade Brasileira de Matemática

EUREKA! N°22, 2005

64

XXVI OLIMPÍADA BRASILEIRA DE MATEMÁTICAResultado – Nível 1 (5a. e 6a. Séries)

NOME CIDADE – ESTADO PRÊMIOIllan Feiman Halpern Itatiaia – RJ OuroLeonardo Pereira Stédile São Paulo – SP OuroJames Jung Hong São Paulo – SP OuroVictor Reis de Abreu Cavalcanti Maceió – AL OuroWagner Carlos Morêto Loyola Filho Vitória – ES OuroRafael Alves da Silva Teresina – PI OuroMarcelo Tadeu de Sá Oliveira Sales Barreiras – BA PrataLereno Soares Netto Rio de Janeiro – RJ PrataDan Zylberglejd Rio de Janeiro – RJ PrataClarissa Maria Grosso Fabris São Paulo – SP PrataMatheus Barros de Paula Taubaté – SP PrataRenan Henrique Finder Joinville – SC PrataLeonardo Shinizu Yojo São Paulo – SP PrataAna Luiza Ferron Zanella Pato Branco – PR PrataFernanda Daltro Costa Knoblauch Salvador – BA PrataThiago Ribeiro Ramos Varginha – MG PrataLucas Gouveia Omena Lopes Maceió – AL PrataJosé Rodolfo Bezerra Mesquita Araújo João Pessoa – PB Prata

Débora Kiame Paias Atibaia – SP BronzeDaniel Caueh Dunaiski Figueira Leal Curitiba – PR BronzeAlessandro D'Alessio de Souza São Paulo – SP BronzeEduardo Kaiser Urubahy Nunes Fonseca Rezende – RJ BronzeCaroline Rigolon Veiga Rio de Janeiro – RJ Bronze

Erick Magno Costa Alonso Uberaba – MG BronzeVinicius Tineli Paiva Vitória – ES BronzeMarcelo Aires Moreira João Pessoa – PB BronzeRafael Fernandes Paixão Rio de Janeiro – RJ BronzeDiana Vaisman Rio de Janeiro – RJ BronzeCamila Miraglia Ribeiro Curitiba – PR BronzeStefano Tonnasini Itu – SP BronzeVitor Cruth Sturn São Paulo – SP BronzeHugo Fonseca Araújo Juiz de Fora – MG BronzeBárbara Seccato Ruis Chagas Vitória – ES BronzeRafael Sarto Müller Vitória – ES Menção HonrosaThiago Gonçales Piracicaba – SP Menção HonrosaJosé Nilo Alves de Souza Neto Fortaleza – CE Menção Honrosa

Vitor Costa Fabris Criciúma – SC Menção HonrosaAna Luísa de Almeida Losnak São Paulo – SP Menção HonrosaRebeca Meirelles de Araújo Assis Salvador – BA Menção HonrosaGabriela Barini de Oliveira São José dos Campos – SP Menção HonrosaÍtalo Leite de Camargo Assis – SP Menção HonrosaArthur Lerer Rio de Janeiro – RJ Menção HonrosaAndré Saraiva Nobre dos Santos Fortaleza – CE Menção HonrosaJosé Ailton Azevedo Araújo Filho Fortaleza – CE Menção HonrosaCésar Ilharco Magalhães Juiz de Fora – MG Menção HonrosaMariana Hollos Fiorencio Rio de Janeiro – RJ Menção HonrosaFernando Fonseca Andrade Oliveira Belo Horizonte – MG Menção HonrosaBeatriz Arruda Asfora Recife – PE Menção HonrosaJoão Paulo Sousa Lucas Fortaleza – CE Menção HonrosaTayrini da Cruz Beligoli Juiz de Fora – MG Menção HonrosaMônica Monteiro Gondim Fortaleza – CE Menção HonrosaIzabela Karennina Travizani Maffra Belo Horizonte – MG Menção HonrosaMaíra Islena Tavares da Silva Belo Horizonte – MG Menção HonrosaCaio Menezes Facó Fortaleza – CE Menção HonrosaAndré Yoshio Oshiro Bastos São Paulo – SP Menção HonrosaNatasha Scaranello Cartolano Rio Claro – SP Menção HonrosaLaura Andrade Santiago Fortaleza – CE Menção HonrosaGuilherme Alencar Sorensen Colares Fortaleza – CE Menção HonrosaEduardo de Almeida Grande Valinhos – SP Menção HonrosaBreno Augusto Cardoso Barroso Fortaleza – CE Menção Honrosa

Page 127: Eureka 2005

Sociedade Brasileira de Matemática

EUREKA! N°22, 2005

65

Resultado – Nível 2 (7a. e 8a. Séries)NOME CIDADE – ESTADO PRÊMIOLucio Eiji Assaoka Hosaka Curitiba – PR OuroRegis Prado Barbosa Fortaleza – CE OuroHenrique Pondé de Oliveira Pinto Salvador – BA OuroRafael Sampaio de Rezende Fortaleza – CE OuroWilson Camara Marriel Rio de Janeiro – RJ OuroRafael Tupynambá Dutra Belo Horizonte – MG PrataAdenilson Arcanjo de Moura Júnior Fortaleza – CE PrataJosé Airton Coêlho Lima Filho Fortaleza – CE PrataFilipe Alves Tomé Fortaleza – CE PrataRafael Moura e Sucupira Fortaleza – CE PrataRamon Moreira Nunes Fortaleza – CE PrataRodrigo Clemente de Brito Pereira João Pessoa – PB PrataSérgio Ricardo Furtado Sampaio Filho Fortaleza – CE Prata

Giuliano Pezzolo Giacaglia Santo André – SP PrataDaniel Ungaretti Borges Belo Horizonte – MG BronzeGabriel Marcos Pasmanik Eisencraft São Paulo – SP BronzeAnna Kelly Krislane de Vasconcelos Fortaleza – CE BronzePaulo Vitor de Souza Albuquerque Rio de Janeiro – RJ BronzeArtur de Almeida Losnak São Paulo – SP BronzeRicardo Turolla Bortolotti Rio Claro – SP BronzeHenrique Hiroshi Motoyama Watanabe São Paulo – SP BronzeGuilherme Philippe Figueiredo Fortaleza – CE BronzeAlfredo Roque de Oliveira Freire Filho Salvador – BA Bronze

Lucas Zanotto Portela Curitiba – PR BronzeNathana Alcântara Lima Fortaleza – CE Bronze

Flávio Domingos de Azevedo Quadros Osasco – SP BronzeMateus Oliveira de Figueiredo Fortaleza – CE BronzeMarilia Valeska Costa Medeiros Fortaleza – CE BronzeGrazielly Muniz da Cunha Fortaleza – CE Menção HonrosaFábio Yoshiteru Nukui São Paulo – SP Menção HonrosaAna Luiza Viegas de Almeida Marília – SP Menção HonrosaVinicius Marques Regitano Piracicaba – SP Menção HonrosaAndré Antonio Battagello Araçatuba – SP Menção HonrosaAnderson Unlin Tsai São Paulo – SP Menção HonrosaMateos Kruchelski Tschá Curitiba – PR Menção HonrosaCaio Gustavo Mesquita Angelo Taguatinga – DF Menção HonrosaGuilherme Zatieff Topolski Chaves São Paulo – SP Menção HonrosaMarlen Lincoln da Silva Fortaleza – CE Menção HonrosaJanailson Rodrigues Lima Fortaleza – CE Menção HonrosaGilson Mauro Costa Fernandes Filho João Pessoa – PB Menção HonrosaDiogo Bonfim Moraes Morant de Holanda Rio de Janeiro – RJ Menção Honrosa

Paulo Wayner Carvalho dos Santos Rio de Janeiro – RJ Menção HonrosaDouglas Tirre Carnevale Oliveira Rio de Janeiro – RJ Menção HonrosaIvan Hitoshi Oyama Osasco – SP Menção HonrosaMarlon Vieira de Lima Júnior Fortaleza – CE Menção HonrosaCássio Kendi Takamori São José dos Campos – SP Menção HonrosaJessica de Aguiar França Brasília – DF Menção HonrosaKarin Naomi Harada de Oliveira São Paulo – SP Menção HonrosaDiogo Soares Dórea da Silva Feira de Santana – BA Menção HonrosaMauricio Henrique Bezerra Cardoso Aracajú – SE Menção HonrosaÉrik Fernando de Amorim Araraquara – SP Menção HonrosaCaio Prado Siqueira Campos Fortaleza – CE Menção HonrosaLuan Leal Oliveira Tremembé – SP Menção HonrosaDiego Eloi Misquita Gomes Fortaleza – CE Menção HonrosaCintia Mayumi Sakurai Kimura São Paulo – SP Menção HonrosaCarolina Santos Andrade São Cristóvão – SE Menção Honrosa

Page 128: Eureka 2005

Sociedade Brasileira de Matemática

EUREKA! N°22, 2005

66

Resultado – Nível 3 (Ensino Médio)

NOME CIDADE – ESTADO PRÊMIOFábio Dias Moreira Rio de Janeiro – RJ OuroRafael Marini Silva Vila Velha – ES OuroGabriel Tavares Bujokas São Paulo – SP OuroRafael Daigo Hirama Campinas – SP OuroGuilherme Rodrigues Nogueira de Souza São Paulo – SP OuroTelmo Luis Correa Junior Santo André – SP OuroThiago Costa Leite Santos São Paulo – SP PrataThomás Yoiti Sasaki Hoshina Rio de Janeiro – RJ PrataLarissa Cavalcante Queiroz de Lima Fortaleza – CE PrataHenry Wei Cheng Hsu São Paulo – SP PrataElton Gomes Coriolano Fortaleza – CE PrataLarissa Rodrigues Ribeiro Fortaleza – CE PrataRodrigo Aguiar Pinheiro Fortaleza – CE PrataFelipe Rodrigues Nogueira de Souza São Paulo – SP PrataRicardo Mizoguchi Gorgoll São Paulo – SP PrataGuilherme Rodrigues Salerno Goiânia – GO BronzeEdson Augusto Bezerra Lopes Fortaleza – CE BronzeLuiz Adolfo Schiller Niterói – RJ BronzeMarcus Edson Barreto Brito Fortaleza – CE BronzeLucas Pavan Barros Vila Velha – ES BronzeDiogo dos Santos Suyama Belo Horizonte – MG BronzeFelipe Ferreira Villar Coelho Serra – ES BronzeJosé Marcos Andrade Ferraro São Paulo – SP BronzeFrancisco Bruno de Lima Holanda Fortaleza – CE BronzeLuty Rodrigues Ribeiro Fortaleza – CE BronzeIsrael Franklin Dourado Carrah Fortaleza – CE BronzeDouglas Bokliang Ang Cunha São José dos Campos – SP BronzeLevi Máximo Viana Fortaleza – CE BronzeLeandro Farias Maia Fortaleza – CE BronzeVictor de Andrade Lazarte São Paulo – SP BronzeRafael Mendes de Oliveira Mesquita – RJ Menção HonrosaRaphael Rodrigues Mata Salvador – BA Menção HonrosaThiago Jorge Marinho Vieira Fortaleza – CE Menção HonrosaAna Cláudia Onuchic São Paulo – SP Menção HonrosaPaulo Victor Teixeira Eufrásio Fortaleza – CE Menção HonrosaCarlos Augusto David Ribeiro Fortaleza – CE Menção HonrosaEduardo Fischer Encantado – RS Menção HonrosaEdson Leandro Finotti Bittar São Paulo – SP Menção HonrosaLeandro Farias Nogueira Recife – PE Menção HonrosaSheyne Cristina Leal Rio de Janeiro – RJ Menção HonrosaHugo Caetano da Silva Junior Teresina – PI Menção HonrosaErasto Villa Branco Neto Curitiba – PR Menção HonrosaAntonia Taline de Souza Mendonça Fortaleza – CE Menção HonrosaJosé Gerardo Arruda Junior Fortaleza – CE Menção HonrosaJosé Mário da Silva Filho Fortaleza – CE Menção HonrosaBruno Cardoso Vieira Rio de Janeiro – RJ Menção HonrosaPedro Henrique Milet Pinheiro Pereira Rio de Janeiro – RJ Menção HonrosaMateus Gomes Filgueiras Fortaleza – CE Menção HonrosaVicente Matheus Moreira Zuffo São Paulo – SP Menção Honrosa

Page 129: Eureka 2005

Sociedade Brasileira de Matemática

EUREKA! N°22, 2005

67

Resultado – Nível Universitário

NOME CIDADE - ESTADO PRÊMIO

Carlos Stein Naves de Brito São José dos Campos – SP OuroAlex Corrêa Abreu Niterói – RJ OuroHumberto Silva Naves São José dos Campos – SP OuroBernardo Freitas Paulo da Costa Rio de Janeiro – RJ OuroDaniel Massaki Yamamoto São Paulo – SP OuroMurilo Vasconcelos Andrade Rio de Janeiro – RJ OuroDavi Máximo Alexandrino Nogueira Fortaleza – CE PrataCarlo Pietro Sousa da Silva São Cristóvão – SE PrataYuri Gomes Lima Fortaleza – CE PrataTertuliano Franco Santos Franco Salvador – BA PrataRodrigo Roque Dias São Paulo – SP PrataDiêgo Veloso Uchôa Teresina – PI PrataPietro Kreitlon Carolino Campinas – SP PrataThiago Barros Rodrigues Costa Fortaleza – CE PrataThiago da Silva Sobral Fortaleza – CE PrataLeonardo Augusto Zão Nilópolis – RJ BronzeSamuel Barbosa Feitosa Fortaleza – CE BronzeEduardo Famini Silva Rio de Janeiro – RJ BronzeEinstein do Nascimento Júnior Fortaleza – CE BronzeEduardo Bertoldi Vila Velha – ES BronzeRafael Tajra Fonteles Teresina – PI BronzeDiogo Diniz Pereira da Silva e Silva Campina Grande – PB BronzeKellem Corrêa Santos Rio de Janeiro – RJ BronzeDomingos Dellamonica Junior São Paulo – SP BronzeLuis Daniel Barbosa Coelho São José dos Campos – SP BronzeJorge Peixoto de Morais Neto São José dos Campos – SP BronzeAnne Caroline Bronzi Ribeirão Preto – SP BronzeEvandro Makiyama de Melo São Paulo – SP Menção HonrosaHelder Oliveira de Castro Mogi das Cruzes – SP Menção HonrosaMoyses Afonso Assad Cohen Rio de Janeiro – RJ Menção HonrosaRogério de Assis Medeiros Franco da Rocha – SP Menção HonrosaRodrigo Angelo Muniz Cariacica – ES Menção HonrosaFabrício Siqueira Benevides Fortaleza – CE Menção HonrosaGiuliano Boava Florianópolis – SC Menção HonrosaFilipe Rodrigues de Souza Moreira São José dos Campos – SP Menção HonrosaAntonio Carlos Maldonado S. A. Munhoz Rio de Janeiro – RJ Menção HonrosaAnderson Rodrigues Ferreira Rio de Janeiro – RJ Menção HonrosaEduardo Ferraz Castelo Branco Ferreira Rio de Janeiro – RJ Menção HonrosaArthur Omar de Andrade Lazarte São Paulo – SP Menção HonrosaVitor Gabriel Kleine Mogi das Cruzes – SP Menção Honrosa

Page 130: Eureka 2005

Sociedade Brasileira de Matemática

EUREKA! N°22, 2005

68

AGENDA OLÍMPICA

XXVII OLIMPÍADA BRASILEIRA DE MATEMÁTICA

NÍVEIS 1, 2 e 3Primeira Fase – Sábado, 11 de junho de 2005

Segunda Fase – Sábado, 03 de setembro de 2005Terceira Fase – Sábado, 22 de outubro de 2005 (níveis 1, 2 e 3)

Domingo, 23 de outubro de 2005 (níveis 2 e 3 - segundo dia de prova).

NÍVEL UNIVERSITÁRIOPrimeira Fase – Sábado, 03 de setembro de 2005

Segunda Fase – Sábado, 22 e Domingo, 23 de outubro de 2005

XI OLIMPÍADA DE MAIO14 de maio de 2005

XVI OLIMPÍADA DE MATEMÁTICA DO CONE SUL23 a 28 de maio de 2005

Sucre, Bolívia

♦XLVI OLIMPÍADA INTERNACIONAL DE MATEMÁTICA

08 a 16 de julho de 2005Yucatán, México

♦XII OLIMPÍADA INTERNACIONAL DE MATEMÁTICA UNIVERSITÁRIA

22 a 28 de julho de 2005Blagoevgrad, Bulgária

♦XX OLIMPÍADA IBEROAMERICANA DE MATEMÁTICA

24 a 30 de setembro de 2005Cartagena de Índias, Colômbia

♦VIII OLIMPÍADA IBEROAMERICANA DE MATEMÁTICA UNIVERSITÁRIA

19 de novembro de 2005

♦♦♦

Page 131: Eureka 2005

Sociedade Brasileira de Matemática

EUREKA! N°22, 2005

69

COORDENADORES REGIONAIS

Alberto Hassen Raad (UFJF) Juiz de Fora – MGAmérico López Gálvez (USP) Ribeirão Preto – SPAmarísio da Silva Araújo (UFV) Viçosa – MGAna Paula Bernardi da Silva (Universidade Católica de Brasília) Brasília – DFAntonio Carlos Nogueira (UFU) Uberlândia – MGAli Tahzibi (USP) São Carlos – SPBenedito Tadeu Vasconcelos Freire (UFRN) Natal – RNCarlos Frederico Borges Palmeira (PUC-Rio) Rio de Janeiro – RJClaus Haetinger (UNIVATES) Lajeado – RSCleonor Crescêncio das Neves (UTAM) Manaus – AMCláudio de Lima Vidal (UNESP) S.J. do Rio Preto – SPEdson Roberto Abe (Colégio Objetivo de Campinas) Campinas – SPÉlio Mega (Colégio Etapa) São Paulo – SPÉder Luiz Pereira de Andrade (UNESPAR/FECILCAM) Campo Mourão – PREudes Antonio da Costa (Univ. do Tocantins) Arraias – TOFlorêncio Ferreira Guimarães Filho (UFES) Vitória – ESIvanilde Fernandes Saad (UC. Dom Bosco) Campo Grande– MSJacqueline Fabiola Rojas Arancibia (UFPB) João Pessoa – PBJanice T. Reichert (UNOCHAPECÓ) Chapecó – SCJoão Benício de Melo Neto (UFPI) Teresina – PIJoão Francisco Melo Libonati (Grupo Educacional Ideal) Belém – PAJosé Carlos dos Santos Rodrigues (Unespar) Campo Mourão – PRJosé Cloves Saraiva (UFMA) São Luis – MAJosé Gaspar Ruas Filho (ICMC-USP) São Carlos – SPJosé Luiz Rosas Pinho (UFSC) Florianópolis – SCJosé Vieira Alves (UFPB) Campina Grande – PBKrerley Oliveira (UFAL) Maceió – ALLicio Hernandes Bezerra (UFSC) Florianópolis – SCLuzinalva Miranda de Amorim (UFBA) Salvador – BAMário Rocha Retamoso (UFRG) Rio Grande – RSMarcelo Rufino de Oliveira (Grupo Educacional Ideal) Belém – PAMarcelo Mendes (Colégio Farias Brito, Pré-vestibular) Fortaleza – CEMarilane de Fraga Sant'Ana FACOS Osório – RSPablo Rodrigo Ganassim (Liceu Terras do Engenho) Piracicaba – SPRamón Mendoza (UFPE) Recife – PERaúl Cintra de Negreiros Ribeiro (Colégio Anglo) Atibaia – SPRonaldo Alves Garcia (UFGO) Goiânia – GOReginaldo de Lima Pereira (Escola Técnica Federal de Roraima) Boa Vista – RRReinaldo Gen Ichiro Arakaki (UNIVAP) SJ dos Campos – SPRicardo Amorim (Centro Educacional Logos) Nova Iguaçu – RJSérgio Cláudio Ramos (IM-UFRGS) Porto Alegre – RSSeme Guevara Neto (UFMG) Belo Horizonte – MGTadeu Ferreira Gomes (UEBA) Juazeiro – BATomás Menéndez Rodrigues (U. Federal de Rondônia) Porto Velho – ROValdenberg Araújo da Silva (U. Federal de Sergipe) São Cristovão – SEValdeni Soliani Franco (U. Estadual de Maringá) Maringá – PRVânia Cristina Silva Rodrigues (U. Metodista de SP) S.B. do Campo – SPWagner Pereira Lopes (CEFET – GO) Jataí – GO